• Search Menu
  • Volume 2024, Issue 4, April 2024 (In Progress)
  • Volume 2024, Issue 3, March 2024
  • Case of the Year
  • MSF Case Reports
  • Audiovestibular medicine
  • Cardiology and cardiovascular systems
  • Critical care medicine
  • Dermatology
  • Emergency medicine
  • Endocrinology and metabolism
  • Gastroenterology and hepatology
  • Geriatrics and gerontology
  • Haematology
  • Infectious diseases and tropical medicine
  • Medical ophthalmology
  • Medical disorders in pregnancy
  • Paediatrics
  • Palliative medicine
  • Pharmacology and pharmacy
  • Radiology, nuclear medicine, and medical imaging
  • Respiratory disorders
  • Rheumatology
  • Sexual and reproductive health
  • Sports medicine
  • Substance abuse
  • Author Guidelines
  • Submission Site
  • Open Access
  • Editorial Board
  • Advertising and Corporate Services
  • Journals Career Network
  • Self-Archiving Policy
  • Journals on Oxford Academic
  • Books on Oxford Academic

Article Contents

Answer to part 1, answer to part 2, answer to part 3, answer to part 4, answer to part 5.

  • < Previous

Educational Case: A 57-year-old man with chest pain

Contributed equally.

  • Article contents
  • Figures & tables
  • Supplementary Data

Nikhil Aggarwal, Subothini Selvendran, Vassilios Vassiliou, Educational Case: A 57-year-old man with chest pain, Oxford Medical Case Reports , Volume 2016, Issue 4, April 2016, Pages 62–65, https://doi.org/10.1093/omcr/omw008

  • Permissions Icon Permissions

This is an educational case report including multiple choice questions and their answers. For the best educational experience we recommend the interactive web version of the exercise which is available via the following link: http://www.oxfordjournals.org/our_journals/omcr/ec01p1.html

A 57 year-old male lorry driver, presented to his local emergency department with a 20-minute episode of diaphoresis and chest pain. The chest pain was central, radiating to the left arm and crushing in nature. The pain settled promptly following 300 mg aspirin orally and 800 mcg glyceryl trinitrate (GTN) spray sublingually administered by paramedics in the community. He smoked 20 cigarettes daily (38 pack years) but was not aware of any other cardiovascular risk factors. On examination he appeared comfortable and was able to complete sentences fully. There were no heart murmurs present on cardiac auscultation. Blood pressure was 180/105 mmHg, heart rate was 83 bpm and regular, oxygen saturation was 97%.

What is the most likely diagnosis?

An ECG was requested and is shown in figure 1.

How would you manage the patient? (The patient has already received 300 mg aspirin).

30 minutes later the patient's chest pain returned with greater intensity whilst waiting in the emergency department. Now, he described the pain as though “an elephant is sitting on his chest”. The nurse has already done an ECG by the time you were called to see him. This is shown in figure 2.

ECG on admission.

ECG on admission.

ECG 30 minutes after admission.

ECG 30 minutes after admission.

What would be the optimal management for this patient?

He was taken to the catheterization lab where the left anterior descending coronary artery (LAD) was shown to be completely occluded. Following successful percutaneous intervention and one drug eluding stent implantation in the LAD normal flow is restored (Thrombosis in myocardial infarction, TIMI = 3). 72 hours later, he is ready to be discharged home. The patient is keen to return to work and asks when he could do so.

When would you advise him that he could return to work?

One week later, he receives a letter informing him that he is required to attend cardiac rehabilitation. The patient is confused as to what cardiac rehabilitation entails, although he does remember a nurse discussing this with him briefly before he was discharged. He phones the hospital in order to get some more information.

Which of the following can be addressed during cardiac rehabilitation?

A - Acute coronary syndrome

Although the presentation could be attributable to any of the above differential diagnoses, the most likely etiology given the clinical picture and risk factors is one of cardiac ischemia. Risk factors include gender, smoking status and age making the diagnosis of acute coronary syndrome the most likely one. The broad differential diagnosis in patients presenting with chest pain has been discussed extensively in the medical literature. An old but relevant review can be found freely available 1 as well as more recent reviews. 2 , 3

C - Atorvastatin 80 mg, Clopidogrel 300 mcg, GTN 500 mcg, Ramipril 2.5 mg,

In patients with ACS, medications can be tailored to the individual patient. Some medications have symptomatic benefit but some also have prognostic benefit. Aspirin 4 , Clopidogrel 5 , Atenolol 6 and Atorvastatin 7 have been found to improve prognosis significantly. ACE inhibitors have also been found to improve left ventricular modeling and function after an MI. 8 , 9 Furthermore, GTN 10 and morphine 11 have been found to be of only significant symptomatic benefit.

Oxygen should only to be used when saturations <95% and at the lowest concentration required to keep saturations >95%. 12

There is no evidence that diltiazem, a calcium channel blocker, is of benefit. 13

His ECG in figure 1 does not fulfil ST elevation myocardial infarction (STEMI) criteria and he should therefore be managed as a Non-STEMI. He would benefit prognostically from beta-blockade however his heart rate is only 42 bpm and therefore this is contraindicated. He should receive a loading dose of clopidogrel (300 mg) followed by daily maintenance dose (75 mg). 14 , 15 He might not require GTN if he is pain-free but out of the available answers 3 is the most correct.

D - Proceed to coronary angiography

The ECG shows ST elevation in leads V2-V6 and confirms an anterolateral STEMI, which suggests a completely occluded LAD. This ECG fulfils the criteria to initiate reperfusion therapy which traditionally require one of the three to be present: According to guidance, if the patient can undergo coronary angiography within 120 minutes from the onset of chest pain, then this represents the optimal management. If it is not possible to undergo coronary angiography and potentially percutaneous intervention within 2 hours, then thrombolysis is considered an acceptable alternative. 12 , 16

≥ 1 mm of ST change in at least two contiguous limb leads (II, III, AVF, I, AVL).

≥ 2 mm of ST change in at least two contiguous chest leads (V1-V6).

New left bundle branch block.

GTN and morphine administration can be considered in parallel but they do not have a prognostic benefit.

E - Not before an exercise test

This patient is a lorry driver and therefore has a professional heavy vehicle driving license. The regulation for driving initiation in a lorry driver following a NSTEMI/ STEMI may be different in various countries and therefore the local regulations should be followed.

In the UK, a lorry driver holds a category 2 driving license. He should therefore refrain from driving a lorry for at least 6 weeks and can only return to driving if he completes successfully an exercise evaluation. An exercise evaluation is performed on a bicycle or treadmill. Drivers should be able to complete 3 stages of the standard Bruce protocol 17 or equivalent (e.g. Myocardial perfusion scan) safely, having refrained from taking anti-anginal medication for 48 hours and should remain free from signs of cardiovascular dysfunction during the test, notably: angina pectoris, syncope, hypotension, sustained ventricular tachycardia, and/or electrocardiographic ST segment shift which is considered as being indicative of myocardial ischemia (usually >2 mm horizontal or down-sloping) during exercise or the recovery period. 18

For a standard car driving license (category 1), driving can resume one week after successful intervention providing that no other revascularization is planned within 4 weeks; left ventricular ejection fraction (LVEF) is at least 40% prior to hospital discharge and there is no other disqualifying condition.

Therefore if this patent was in the UK, he could restart driving a normal car one week later assuming an echocardiogram confirmed an EF > 40%. However, he could only continue lorry driving once he has passed the required tests. 18

E - All of the above

Cardiac rehabilitation bridges the gap between hospitals and patients' homes. The cardiac rehabilitation team consists of various healthcare professions and the programme is started during hospital admission or after diagnosis. Its aim is to educate patients about their cardiac condition in order to help them adopt a healthier lifestyle. This includes educating patients' about their diet, exercise, risk factors associated with their condition such as smoking and alcohol intake and finally, about the medication recommended. There is good evidence that adherence to cardiac rehabilitation programmes improves survival and leads to a reduction in future cardiovascular events.​ 19 , 20

Oille JA . Differential diagnosis of pain in the chest . Can Med Assoc J . 1937 ; 37 (3) : 209 – 216 . http://www.ncbi.nlm.nih.gov/pmc/articles/PMC536075/ .

Google Scholar

Lee TH , Goldman L . Evaluation of the patient with acute chest pain . N Engl J Med . 2000 ; 342 (16) : 1187 – 1195 . http://www.nejm.org/doi/full/10.1056/NEJM200004203421607 .

Douglas PS , Ginsburg GS . The evaluation of chest pain in women . N Engl J Med . 1996 ; 334 (20) : 1311 – 1315 . http://www.nejm.org/doi/full/10.1056/NEJM199605163342007 .

Baigent C , Collins R , Appleby P , Parish S , Sleight P , Peto R . ISIS-2: 10 year survival among patients with suspected acute myocardial infarction in randomised comparison of intravenous streptokinase, oral aspirin, both, or neither. the ISIS-2 (second international study of infarct survival) collaborative group . BMJ . 1998 ; 316 (7141) : 1337 – 1343 . http://www.ncbi.nlm.nih.gov/pmc/articles/PMC28530/ .

Yusuf S , Zhao F , Mehta S , Chrolavicius S , Tognoni G , Fox K . Clopidogrel in unstable angina to prevent recurrent events trail investigators . effects of clopidogrel in addition to aspirin in patients with acute coronary syndromes without ST-segment elevation . N Engl J Med . 2001 ; 345 (7) : 494 – 502 . http://www.nejm.org/doi/full/10.1056/NEJMoa010746#t=articleTop .

Yusuf S , Peto R , Lewis J , Collins R , Sleight P . Beta blockade during and after myocardial infarction: An overview of the randomized trials . Prog Cardiovasc Dis . 1985 ; 27 (5) : 335 – 371 . http://www.sciencedirect.com/science/article/pii/S0033062085800037 .

Schwartz GG , Olsson AG , Ezekowitz MD et al.  . Effects of atorvastatin on early recurrent ischemic events in acute coronary syndromes: The MIRACL study: A randomized controlled trial . JAMA . 2001 ; 285 (13) : 1711 – 1718 . http://jama.jamanetwork.com/article.aspx?articleid=193709 .

Pfeffer MA , Lamas GA , Vaughan DE , Parisi AF , Braunwald E . Effect of captopril on progressive ventricular dilatation after anterior myocardial infarction . N Engl J Med . 1988 ; 319 (2) : 80 – 86 . http://content.onlinejacc.org/article.aspx?articleid=1118054 .

Sharpe N , Smith H , Murphy J , Hannan S . Treatment of patients with symptomless left ventricular dysfunction after myocardial infarction . The Lancet . 1988 ; 331 (8580) : 255 – 259 . http://www.sciencedirect.com/science/article/pii/S0140673688903479 .

Ferreira JC , Mochly-Rosen D . Nitroglycerin use in myocardial infarction patients . Circ J . 2012 ; 76 (1) : 15 – 21 . http://www.ncbi.nlm.nih.gov/pmc/articles/PMC3527093/ .

Herlitz J , Hjalmarson A , Waagstein F . Treatment of pain in acute myocardial infarction . Br Heart J . 1989 ; 61 (1) : 9 – 13 . http://www.ncbi.nlm.nih.gov/pmc/articles/PMC1216614/ .

Task Force on the management of ST-segment elevation acute myocardial infarction of the European Society of Cardiology (ESC), Steg PG, James SK, et al . ESC guidelines for the management of acute myocardial infarction in patients presenting with ST-segment elevation . Eur Heart J . 2012 ; 33 (20) : 2569 – 2619 . http://eurheartj.oxfordjournals.org/content/33/20/2569 .

The effect of diltiazem on mortality and reinfarction after myocardial infarction . the multicenter diltiazem postinfarction trial research group . N Engl J Med . 1988 ; 319 (7) : 385 – 392 . http://www.nejm.org/doi/full/10.1056/NEJM198808183190701 .

Jneid H , Anderson JL , Wright RS et al.  . 2012 ACCF/AHA focused update of the guideline for the management of patients with unstable angina/Non–ST-elevation myocardial infarction (updating the 2007 guideline and replacing the 2011 focused update) A report of the american college of cardiology foundation/american heart association task force on practice guidelines . J Am Coll Cardiol . 2012 ; 60 (7) : 645 – 681 . http://circ.ahajournals.org/content/123/18/2022.full .

Hamm CW , Bassand JP , Agewall S et al.  . ESC guidelines for the management of acute coronary syndromes in patients presenting without persistent ST-segment elevation: The task force for the management of acute coronary syndromes (ACS) in patients presenting without persistent ST-segment elevation of the european society of cardiology (ESC) . Eur Heart J . 2011 ; 32 (23) : 2999 – 3054 . http://eurheartj.oxfordjournals.org/content/32/23/2999.long .

O'Gara PT , Kushner FG , Ascheim DD et al.  . 2013 ACCF/AHA guideline for the management of ST-elevation myocardial infarction: Executive summary: A report of the american college of cardiology foundation/american heart association task force on practice guidelines . J Am Coll Cardiol . 2013 ; 61 (4) : 485 – 510 . http://content.onlinejacc.org/article.aspx?articleid=1486115 .

BRUCE RA , LOVEJOY FW Jr . Normal respiratory and circulatory pathways of adaptation in exercise . J Clin Invest . 1949 ; 28 (6 Pt 2) : 1423 – 1430 . http://www.ncbi.nlm.nih.gov/pmc/articles/PMC439698/ .

DVLA . Https://Www.gov.uk/current-medical-guidelines-dvla-guidance-for-professionals-cardiovascular-chapter-appendix .

British Heart Foundation . Http://Www.bhf.org.uk/heart-health/living-with-heart-disease/cardiac-rehabilitation.aspx .

Kwan G , Balady GJ . Cardiac rehabilitation 2012: Advancing the field through emerging science . Circulation . 2012 ; 125 (7) : e369–73. http://circ.ahajournals.org/content/125/7/e369.full .

Author notes

  • knowledge acquisition

Email alerts

Citing articles via, affiliations.

  • Online ISSN 2053-8855
  • Copyright © 2024 Oxford University Press
  • About Oxford Academic
  • Publish journals with us
  • University press partners
  • What we publish
  • New features  
  • Open access
  • Institutional account management
  • Rights and permissions
  • Get help with access
  • Accessibility
  • Advertising
  • Media enquiries
  • Oxford University Press
  • Oxford Languages
  • University of Oxford

Oxford University Press is a department of the University of Oxford. It furthers the University's objective of excellence in research, scholarship, and education by publishing worldwide

  • Copyright © 2024 Oxford University Press
  • Cookie settings
  • Cookie policy
  • Privacy policy
  • Legal notice

This Feature Is Available To Subscribers Only

Sign In or Create an Account

This PDF is available to Subscribers Only

For full access to this pdf, sign in to an existing account, or purchase an annual subscription.

  • Skip to content

MNWC

State Resources

National Resources

Nursing Organizations

  • MNWC Initiatives

Maryland Nursing Workforce Center

  • NextGen NCLEX

Faculty Case Studies

The purpose of this project was to develop a repository of NextGen NCLEX case studies that can be accessed by all faculty members in Maryland.

Detailed information about how faculty members can use these case students is in this PowerPoint document .

The case studies are in a Word document and can be modified by faculty members as they determine. 

NOTE: The answers to the questions found in the NextGen NCLEX Test Bank  are only available in these faculty case studies. When students take the Test Bank questions, they will not get feedback on correct answers. Students and faculty should review test results and correct answers together.

The case studies are contained in 4 categories: Family (13 case studies), Fundamentals and Mental Health (14 case studies) and Medical Surgical (20 case studies). In addition the folder labeled minireviews contains PowerPoint sessions with combinations of case studies and standalone items. 

Family  ▾

  • Attention Deficit Hyperactivity Disorder - Pediatric
  • Ectopic Pregnancy
  • Febrile Seizures
  • Gestational Diabetes
  • Intimate Partner Violence
  • Neonatal Jaundice
  • Neonatal Respiratory Distress Syndrome
  • Pediatric Hypoglycemia
  • Pediatric Anaphylaxis
  • Pediatric Diarrhea and Dehydration
  • Pediatric Intussusception
  • Pediatric Sickle Cell
  • Postpartum Hemmorhage
  • Poststreptococcal Glomerulonephritis Pediatric
  • Preeclampsia

Fundamentals and Mental Health  ▾

  • Abdominal Surgery Postoperative Care
  • Anorexia with Dehydration
  • Catheter Related Urinary Tract Infection
  • Deep Vein Thrombosis
  • Dehydration Alzheimers
  • Electroconvulsive Therapy
  • Home Safety I
  • Home Safety II
  • Neuroleptic Maligant Syndrome
  • Opioid Overdose
  • Post Operative Atelectasis
  • Post-traumatic Stress
  • Pressure Injury
  • Substance Use Withdrawal and Pain Control
  • Suicide Prevention
  • Tardive Dyskinesia
  • Transfusion Reaction
  • Urinary Tract infection

Medical Surgical  ▾

  • Acute Asthma
  • Acute Respiratory Distress
  • Breast Cancer
  • Chest Pain (MI)
  • Compartment Syndrome
  • Deep Vein Thrombosis II
  • End Stage Renal Disease and Dialysis
  • Gastroesphageal Reflux
  • Heart Failure
  • HIV with Opportunistic Infection
  • Ketoacidosis
  • Liver Failure
  • Prostate Cancer
  • Spine Surgery
  • Tension Pneumothorax
  • Thyroid Storm
  • Tuberculosis

Community Based  ▾

Mini Review  ▾

  • Comprehensive Review
  • Fundamentals
  • Maternal Newborn Review
  • Medical Surgical Nursing
  • Mental Health Review
  • Mini Review Faculty Summaries
  • Mini Review Training for Website
  • Mini Reviews Student Worksheets
  • Pediatric Review

Registered Nurse RN

Registered Nurse, Free Care Plans, Free NCLEX Review, Nurse Salary, and much more. Join the nursing revolution.

Next Generation NCLEX Case Study Sample Questions

One of the big changes on the Next Generation NCLEX exam is a shift toward case studies. Case studies often require a deeper level of critical thinking, and understanding diseases on a more in-depth level (especially the pathophysiology) will make these types of questions easier to answer.

In this article, you’ll be able to watch a free video to help you prepare for the new Next Generation NCLEX case study format. Nurse Sarah will walk you step-by-step through each scenario and help you understand how to use critical thinking and nursing knowledge to answer these types of questions.

Next Generation NCLEX Case Study Review Questions Video

NGN Case Study Sample Questions and Answers

First, let’s take a look at our case study summary below:

Case Study Summary:

A 68-year-old male is admitted with shortness of breath. He reports difficulty breathing with activity, lying down, or while sleeping. He states that in order to “breathe easier,” he has had to sleep in a recliner for the past week. The patient has a history of hypertension, myocardial infarction (2 years ago), and cholecystectomy (10 years ago). The patient is being transferred to a cardiac progressive care unit for further evaluation and treatment.

Question 1 of 6: The nurse receives the patient admitted with shortness of breath. What findings are significant and require follow-up? The options are listed below. Select all that apply.

To answer this first question in the NGN case study, let’s look at the information provided in the nursing notes and vital signs tabs provided:

next generation nclex, ngn case study, next generation nclex case study, next generation nclex questions and answers, ngn practice

This question is asking us to identify findings that are significant and require the nurse to follow-up. In other words, what is presenting that we can’t ignore but need to investigate further.

Therefore, let’s comb through the nursing notes and vital signs to see what is abnormal and requires follow-up.

First, the patient arrived to the room via stretcher. That’s fine and doesn’t necessarily require follow-up.

Next, the patient is alert and oriented x 4 (person, place, time, event). This tells us that the patient’s neuro status is intact so far. Therefore, the shortness of breath isn’t affecting the patient’s mental function yet (we have enough oxygen on board right now for brain activity).

However, the nurse has noticed the shortness of breath with activity and talking, which should not normally happen. This tells us something is wrong and is significant enough to require follow-up. We want to know why is this happening, is it going to get worse, etc.

The patient’s weight and vital signs were collected (this is good). Weight is 155 lbs. and BMI is within a healthy range (doesn’t tell us too much but may be useful later). The patient is also connected to a bedside monitor, so they need to be monitored constantly like on a progressive care unit.

The monitor shows sinus tachycardia . This is significant because it seems the patient’s shortness of breath is causing the heart to compensate by increasing the heart rate to provide more oxygen (hence the lungs may be compromised).

Then we find out that the lungs are indeed compromised because crackles are heard in both lungs , and this may be why our patient is short of breath. This is significant (could the patient have pulmonary edema?)

Then we find out the nurse has noted an S3. This is an extra heart sound noted after S2. And what jumps out to me about this is that it is usually associated with volume overload in the heart like in cases of heart failure . However, S3 may be normal in some people under 40 or during pregnancy, but that’s not the case with our patient based on what we read in the case summary.

Therefore, based on everything I’m reading in this case study, I’m thinking this patient may have heart failure, but we need those test results back (especially the echo and chest x-ray, and hopefully a BNP will be in there too).

We are also told that the patient has an 18 gauge IV inserted (which is good thing to have so we can give medications if required), orders have been received, labs drawn, and testing results are pending.

next generation nclex, nclex prep, nclex case study questions, nclex questions and answers, ngn review,

Now let’s look at the “Vital Signs” tab above, and ask yourself what is normal vs. abnormal for this patient (adult male).

  • The heart rate is high at 112 (tachycardia), and should normally be 60-100 bpm (see heart rhythms ).
  • Blood pressure is higher than normal (normal is 120/80), which indicates hypertension.
  • Oxygen saturation is 94% (this is on the low side as we’d normally want around 95% or higher, and the patient is on 4 L nasal cannula, which tells us the lungs are not okay).
  • Respiratory rate is increased (26 breaths per minute)…normal is 12-20 breaths per minute.

Based on the information we were provided, I’ve selected the answers below. These findings are significant and definitely require follow-up by the nurse.

next generation nclex questions and answers, next generation nclex answers, next generation nclex sample questions, ngn questions

When answering these NGN case study questions, it’s helpful to think of the ABCDE (airway, breathing, circulation, etc.) as all of these fall into that category. If we don’t follow-up on the shortness of breath, crackles, respiratory rate, o2 saturation (94% on 4 L nasal cannula), the respiratory system can further decline.

In addition, the sinus tachycardia, S3 gallop, and hypertension could indicate fluid overload in the heart. This may cause the heart to tire out and lead the lethal rhythm. On the other hand, temperature, pain, weight, and BMI are not abnormal and do not require follow-up.

See the Complete Next Generation NCLEX Case Study Review

Each question in the case study builds on the previous question. To see how these questions evolve based on the patient’s condition and labs, watch the entire Next Generation NCLEX Case Study Review video on our YouTube Channel (RegisteredNurseRN).

NCLEX Practice Quizzes

We’ve developed many free NCLEX review quizzes to test your knowledge on nursing topics and to help you prepare for the Next Generation NCLEX exam.

Nurse Sarah’s Notes and Merch

fluid electrolytes nursing nclex, notes, mnemonics, quizzes, nurse sarah, registerednursern

Just released is “ Fluid and Electrolytes Notes, Mnemonics, and Quizzes by Nurse Sarah “. These notes contain 84 pages of Nurse Sarah’s illustrated, fun notes with mnemonics, worksheets, and 130 test questions with rationales.

You can get an eBook version here or a physical copy of the book here.

Please Share:

  • Click to print (Opens in new window)
  • Click to share on Facebook (Opens in new window)
  • Click to share on Twitter (Opens in new window)
  • Click to share on Pinterest (Opens in new window)
  • Click to share on Reddit (Opens in new window)
  • Click to share on LinkedIn (Opens in new window)
  • Click to share on WhatsApp (Opens in new window)
  • Click to share on Pocket (Opens in new window)
  • Click to share on Telegram (Opens in new window)

Disclosure and Privacy Policy

Important links, follow us on social media.

  • Facebook Nursing
  • Instagram Nursing
  • TikTok Nurse
  • Twitter Nursing
  • YouTube Nursing

Copyright Notice

We have a new app!

Take the Access library with you wherever you go—easy access to books, videos, images, podcasts, personalized features, and more.

Download the Access App here: iOS and Android . Learn more here!

  • Remote Access
  • Save figures into PowerPoint
  • Download tables as PDFs

Pathophysiology of Disease: An Introduction to Clinical Medicine, 7e

25:  Case Study Answers

Yeong Kwok, MD

  • Download Chapter PDF

Disclaimer: These citations have been automatically generated based on the information we have and it may not be 100% accurate. Please consult the latest official manual style if you have any questions regarding the format accuracy.

Download citation file:

  • Search Book

Jump to a Section

Case study answers.

  • Full Chapter
  • Supplementary Content

Osteogenesis Imperfecta

The four types of osteogenesis imperfecta are type I (mild), type II (perinatal, lethal), type III (progressive, deforming), and type IV (deforming with normal scleras). All forms of osteogenesis imperfecta are characterized by increased susceptibility to fractures (“brittle bones”), but there is considerable phenotypic heterogeneity, even within individual subtypes. Approximately one fourth of the cases of type I or type IV osteogenesis imperfecta represent new mutations; in the remainder, the history and examination of other family members reveal findings consistent with autosomal dominant inheritance. Type III is also transmitted as an autosomal dominant trait, although type III can occasionally be transmitted in an autosomal recessive manner. Type II, the most severe form, generally occurs as a result of a sporadic dominant mutation.

Type II osteogenesis imperfecta presents at birth (or even in utero) with multiple fractures and bony deformities, resulting in death in infancy and, therefore, not likely to be seen in a child 4 years of age. Type III presents at birth or in early infancy with multiple fractures—often prenatal—and progressive bony deformities. The absence of prenatal fractures and early deformities in this patient’s history is most suggestive of type I or type IV osteogenesis imperfecta. These individuals generally present in early childhood with one or a few fractures of long bones in response to minimal or no trauma, as seen in this case. Type I and type IV osteogenesis imperfecta are differentiated by their clinical severity and scleral hue. Type I tends to be less severe, with 10–20 fractures during childhood plus short stature but few or no deformities. These patients tend to have blue scleras. Patients with type IV osteogenesis imperfecta tend to have more fractures, resulting in significant short stature and mild to moderate deformities. Their scleras are normal or gray.

In patients with type I osteogenesis imperfecta, the fracture incidence decreases after puberty and the main features in adult life are mild short stature, conductive hearing loss, and occasionally dentinogenesis imperfecta (defective dentin formation in tooth development).

Sign in or create a free Access profile below to access even more exclusive content.

With an Access profile, you can save and manage favorites from your personal dashboard, complete case quizzes, review Q&A, and take these feature on the go with our Access app.

Pop-up div Successfully Displayed

This div only appears when the trigger link is hovered over. Otherwise it is hidden from view.

Please Wait

NEJM Knowledge+ Logo

Announcing 50 New Case-Based Questions in NEJM Knowledge+ Internal Medicine Board Review

New Case-Based Questions for Internal Medicine Board Review

At NEJM Knowledge+, we’re committed to ensuring that our products cover the breadth of knowledge that clinicians need for both clinical practice and Internal Medicine Board Exam preparation. NEJM Knowledge+ Internal Medicine Board Review already contains more than 1600 case-based questions on the most relevant and important topics in medicine today. We’re adding another 50 case-based questions now (and 50 more in December 2016) to further expand that knowledge base.

Most of the 50 new questions we’ve added relate to the following  topics:

  • Infectious Disease
  • Dermatology

Each year, we plan to add at least 100 new questions to NEJM Knowledge+ Internal Medicine Board Review — this is in addition to continually updating our content when guidelines change and in response to user feedback. Our goal is to become increasingly comprehensive in the learning we provide while remaining as clinically relevant and up-to-date as possible.

Covering the ABIM Blueprint with New Learning Objectives

In June 2015, ABIM rolled out a revised blueprint for the Maintenance of Certification (MOC) exam that not only listed the topics and subtopics but also showed the likelihood of which aspects of the subtopic will be on the exam, such as diagnosis, testing, and treatment.

Our editorial team analyzed the new ABIM blueprint and are prioritizing development of new IM questions that map to topics in the blueprint that are highly likely to be on the MOC exam.

We have derived learning objectives from the topic/task combinations in the blueprint; for example, ABIM lists six subtopics under “ischemic heart disease”:

Internal Medicine Blueprint for Cardiovascular Disease

These subtopics mostly fall into the “highly likely to be on the exam” bucket (green), so we recruited physician experts to write case-based questions that test those learning objectives that we did not already have at least one question on in our question bank. Here are some examples of the learning objectives we just added to the IM question bank:

  • Choose an optimal initial testing strategy for a patient with a prior acute anterior myocardial infarction who presents with a transient ischemic attack that has a suspected cardioembolic source.
  • Choose an appropriate treatment for improving the likelihood of survival in a patient who has depressed left ventricular systolic function after an acute myocardial infarction.
  • Choose appropriate evaluation for suspected heart failure with reduced ejection fraction.
  • Choose the most appropriate pharmacologic management for a patient who has a recent diagnosis of heart failure with reduced ejection fraction and is already taking an angiotensin-converting enzyme inhibitor.
  • Recognize heart failure with reduced left ventricular ejection fraction, secondary to ischemic cardiomyopathy.

Using this process for content development ensures that our question bank covers what you need to know for the board exam.

Case-Based Questions, Free from Outside Influence

All the questions we develop for NEJM Knowledge+ Internal Medicine Board Review meet the same high-quality standards you’ve come to expect from NEJM Group. The content was written by more than 300 clinicians from academic programs across the country and was subjected to a rigorous editorial process that included review by highly respected professional educators, leading specialists in their fields, generalists, PAs, and NEJM Group editors . You can be sure that what you’re learning in NEJM Knowledge+ is accurate, evidence-based, and relevant to your daily practice.

NEJM Knowledge+ offers a comprehensive question bank that reflects the breadth of primary care cases that physicians encounter in their practices today.

Personalized Learning, Tailored to You

NEJM Knowledge+ uses adaptive learning technology that tailors your learning to your needs. This adaptive learning technology continuously assesses the subjects you know and identifies the areas where you need reinforcement. It then delivers questions based on what you know already, what you need to study more, what you are struggling to master, what you think you know better than you do, and what you might be forgetting.

With the addition of these 50 new questions, NEJM Knowledge+ Internal Medicine Board Review now includes:

  • more than 1680 case-based questions
  • more than 4500 total questions tied to 2500 learning objectives

With the ability to earn:

  • CME credits
  • ABIM MOC points

All in all, we are strengthening one of the most comprehensive solutions available for continuous learning and board exam preparation.

More on NEJM Knowledge+ Content:

Roadmap to Great Content Work Less and Learn More: Here’s How in NEJM Knowledge+ Content Updates

Share This Post!

' src=

Having only a partial point of view of Cases Based Questins,I can see that you tried to do the best work in this section.I am totally sure, that the Clinical Cases presented will be carefully prepared.But, the problem that I see, in ER,Clinics-Hospitals-and in Home , it is that the patient, many times has not the Diagnostics written in his/her Chest.Many times, we have to do an intense work,trying to know what happen with the patients and trying to find(if we are in ER or Hospital) his/her medical record. Being agree with the way you prepared the questions,but I think that in Real Life,we have to face with patients, whose Diagnostics, we do not know,but we have to start with some measuresEx=relieving pain-giving IV solutions (if they are needed)-taking Exams, like Blood-Urina and others-XRay-CT Scan-Ultrasound and calling to others Physicians(Specialists=Cardiologists-Neurologists-Nephrologists etc-etc= so I suggest to add(if if is possible) some Complete Clinical Cases, where Students or Residents, must choose since the beginning the possible diagnostics-type of Blood-Urine -Bacteriologic Exams-Ultrasound-CT-Scan etcetcThis type of Questions(that NEJM sometimes present)are one of the best technique to know what the

' src=

Notify me by email with any comment

' src=

does anyone know of any journal/quis cme’s that are accepted for moc points in internal medicine? Also it seems like it varies from state to state. For example, JAMA articles/cme quizzes for some reason aren’t accepted in NY. I left my email if anyone should have any information. Thanks much

Comments are closed.

 logo

  • Advanced Life Support
  • Endocrinology
  • Gastroenterology
  • Infectious disease
  • Intensive care
  • Palliative Care
  • Respiratory
  • Rheumatology
  • Haematology
  • Endocrine surgery
  • General surgery
  • Neurosurgery
  • Ophthalmology
  • Plastic surgery
  • Vascular surgery
  • Abdo examination
  • Cardio examination
  • Neurolo examination
  • Resp examination
  • Rheum examination
  • Vasc exacmination
  • Other examinations
  • Clinical Cases
  • Communication skills
  • Prescribing

 logo

ABG Examples (ABG exam questions for medical students and PACES)

Print Friendly, PDF & Email

ABG Examples (ABG exam questions for medical students OSCEs and MRCP PACES)

Below are some brief clinical scenarios with ABG results. Try to interpret each ABG and formulate a differential diagnosis before looking at the answer.

Question 1.

You are called to see a 54 year old lady on the ward. She is three days post-cholecystectomy and has been complaining of shortness of breath. Her ABG is as follows:

  • pH: 7.49 (7.35-7.45)
  • pO2: 7.5 (10–14)
  • pCO2: 3.9 (4.5–6.0)
  • HCO3:  22 (22-26)
  • BE: -1 (-2 to +2)
  • Other values within normal range
  • This is type 1 respiratory failure. The PO2 is low with a low CO2.
  • The accompanying alkalosis is a response, due to the patient blowing off CO2 due to her likely high respiratory rate.

What is the differential diagnosis?

  • Pulmonary embolus (PE)
  • Pulmonary oedema
  • Pneumothorax
  • Severe atelectasis

What would you do?

  • Acutely unwell: ABCDE and call for help
  • All of these conditions can may you tachypnoeic and tachycardic. Wheeze will predominate in asthma. Pyrexia points more towards pneumonia (but PE can give a mild pyrexia). Pulmonary embolus will be the only condition that will likely be normal on auscultation.
  • Sudden onset: more likely PE
  • Purulent cough: more likely pneumonia
  • Raised JVP, ankle swelling, fine basal creps: more likely oedema
  • Cultures if pyrexial
  • PE : Heparinisation or thrombolysis if unstable. Remember this patient is post-op so it is a complex decision.
  • Pneumonia : Antibiotics for hospital acquired pneumonia
  • Asthma : Salbutamol, ipatropium and steroid in the first instance
  • Pulmonary oedema : Sit patient up, furosemide, consider catheter

Question 2.

A 75 year old gentleman living in the community is being assessed for home oxygen. His ABG is as follows:

  • pH: 7.36 (7.35-7.45)
  • pO2: 8.0 (10–14)
  • pCO2: 7.6 (4.5–6.0)
  • HCO3: 31 (22-26)
  • BE: +5 (-2 to +2)
  • This does not represent acute pathology.
  • Rather it reflects a compensation for a chronic respiratory acidosis secondary to chronic pulmonary disease.
  • Note this is an acidosis, not an acidaemia (pH normal, but only due to compensatory mechanisms: the high bicarbonate).
  • Lifestyle advice and smoking cessation of necessary.
  • PaO 2  less than 7.3 kPa when stable.
  • Secondary polycythaemia
  • Peripheral oedema
  • Nocturnal hypoxaemia
  • Pulmonary hypertension

Question 3.

A 64 year old gentleman with a history of COPD presents with worsening shortness of breath and increased sputum production .

  • pH: 7.21 (7.35-7.45)
  • pO2: 7.2 (10–14)
  • pCO2: 8.5 (4.5–6.0)
  • HCO3: 29 (22-26)
  • BE: +4 (-2 to +2)
  • Note that the HCO3 is raised in this patient despite the abnormal pH.
  • With the above history this is likely to represent an acute on chronic respiratory acidosis.
  • This would indicate that the patient normally retains CO2 and has a chronically raised HCO3.
  • The drop in pH represents the normal mechanisms of compensation being over whelmed.
  • This is one of the cases where having an old ABG from a previous admission can be useful.

How much oxygen would you give this man?

  • Oxygen administration in this group is a complicated issue. 100% oxygen makes subsets of COPD patients retain CO2, decreasing respiratory drive and worsening hypoxia and hypercapnia.
  • More information can be found on this page: Prescribing oxygen in COPD patients
  • The British Thoracic Society have produced guidelines which give a helpful overview and can be found here.

Question 4.

A 21 year-old woman presents feeling acutely lightheaded and short of breath. She has her final university exams next week.

  • pH: 7.48 (7.35-7.45)
  • pO2: 13.9 (10–14)
  • pCO2: 3.5 (4.5–6.0)
  • HCO3: 22 (22-26)
  • BE: +2 (-2 to +2)
  • This is a respiratory alkalaemia
  • Pulmonary disease
  • Hypermetabolic states (e.g. infection or fever)
  • Anxiety hyperventilation

What's the most likely diagnosis?

  • Based on the history, anxiety hyperventilation is the most likely cause here. However, it is very important to have considered the other options, in particular and to have ruled out a primary respiratory pathology or infection.
  • In the anxious patient who is short of breath and persistently tachycardic have you thought of PE?

Question 5.

A 32 year-old man presents to the emergency department having been found collapsed by his girlfriend.

  • pH: 7.25 (7.35-7.45)
  • pO2: 11.1 (10–14)
  • pCO2: 3.2 (4.5–6.0)
  • HCO3: 11 (22-26)
  • BE: -15 (-2 to +2)
  • Potassium: 4.5
  • Sodium: 135
  • Chloride: 100
  • Anion gap = ([Na + ] + [K + ]) − ([Cl − ] + [HCO 3 − ])
  • Reference range usually 7–16 mEq/L (but varies between hospitals, some using 3-11)
  • Anion gap = [Na + ] − ([Cl – ] + [HCO 3 − ])

What is the anion gap in this case?

  • N.B. Some analysers won’t include potassium in their calculations therefore for them >15 constitutes a raised anion gap.
  • Either way, this is a raised anion gap metabolic acidosis.

What is the differential diagnosis for a metabolic acidosis with raised anion gap? The traditional mnemonic for the causes of a metabolic acidosis with raised anion gap is ‘MUDPILES’:

  • D iabetic ketoacidosis (and alcoholic/starvation ketoacidosis)
  • P ropylene glycol
  • E thylene glycol
  • S alicylates

However, another way is to think about the mechanism of acidosis:

  • DKA, lactic acidosis (produced by poorly perfused tissues)
  • Methanol, ethanol, ethylene glycol
  • Renal failure

[/toggle title="What is the differential diagnosis for a metabolic acidosis with normal or decreased anion gap?" active="false"]

  • From the GI tract (diarrhoea or high-output stoma)
  • From the kidneys ( renal tubular acidosis )

Question 6.

A 67 year-old man with a history of peptic ulcer disease presents with persistent vomiting.

  • pH: 7.56 (7.35-7.45)
  • pO2: 10.7 (10–14)
  • pCO2: 5.0 (4.5–6.0)
  • HCO3: 31 (22-26)
  • BE: +5 (-2 to +2)
  • This is metabolic alkalaemia

[/toggle title="What' s the differential diagnosis of this ABG picture?" active="false"]

Differential diagnosis of a metabolic alkalosis or alkalaemia:

  • E.g. gastric outlet obstruction (the classic example is pyloric stenosis in a baby)
  • Hyperaldosteronaemia
  • Diuretic use
  • Milk alkali syndrome
  • Massive transfusion

Question 7.

A seventeen year-old girl presents to the emergency department after an argument with her boyfriend. He says that she took lots of tablets. She denies this. You persuade her to let you do an ABG:

  • pH: 7.46 (7.35-7.45)
  • pO2: 12.5 (10–14)
  • BE: +1 (-2 to +2)

A few hours later she says she feels increasingly unwell and is complaining of ringing in her ears. A repeat gas shows:

  • pH: 7.15 (7.35-7.45)
  • pO2: 11.0 (10–14)
  • HCO3: 9 (22-26)
  • BE: -18 (-2 to +2)
  • This is the classic picture of aspirin overdose.
  • There is an initial respiratory alkalosis due to central respiratory centre stimulation causing  increased respiratory drive.
  • In the later stages a metabolic acidosis develops along side the respiratory alkalosis as a result of direct effect of the metabolite salicylic acid and more complex disruption of normal cellular metabolism.

How would you manage this patient?

How do you manage an aspirin overdose?

Presentation of aspirin overdose

  • Hyperventilation
  • Nausea & vomiting
  • Epigastric pain
  • ARDS (rare)
  • Hypoglycaemia (children in particular)

Investigations in aspirin overdose

  • Plasma salicylate concentration (initial and repeats)
  • Paracetamol levels (always check in any case of poisoning by anything)
  • Renal failure (rare) sometimes other electrolyte imbalances
  • If dropping sats or any suspicion of ARDS (non-cardiogenic pulmonary oedema)

Management of aspirin overdose

  • ABCDE and supportive care
  • Gastric lavage within 1h of ingestion (although no evidence for mortality reduction)
  • Activated charcoal
  • Correct electrolyte abnormalities
  • Give 225ml of 8.4% bicarbonate solution over 1hr
  • Bicarbonate will increase any pre-existing hypokalaemia – so don’t let it happen
  • N.B. Acidosis increases salicylate transfer across the blood brain barrier
  • Monitor U+Es regularly
  • Haemodialysis

Prognosis in aspirin overdose

  • Generally good with treatment.

Question 8.

A normally fit and well 11 year-old boy presents with diarrhoea and vomiting. He is complaining of non-specific abdominal pain. A venous blood gas shows :

  • pH: 7.12 (7.35-7.45)
  • pO2: 11.5 (10–14)
  • BE: -17 (-2 to +2)
  • Lactate: 4.0
  • Potassium: 5.5
  • Glucose: 22 mmol/L (395 mg/dL)
  • This is diabetic ketoacidosis (DKA) .

What are you going to do?

  • Priorities for management include fluid resuscitation, insulin administration and careful management of potassium levels. Click here for a page detailing this, and click here for DKA questions 

Question 9.

A 22 year-old lady with a known history of asthma presents to the emergency department with difficulty in breathing. Her initial ABG on 15 litres of oxygen shows:

  • pH: 7.54 (7.35-7.45)
  • pO2: 10.0 (10–14)
  • HCO3: 24 (22-26)
  • BE: +0 (-2 to +2)

After initial treatment the nurse in resus calls you to review the patient. The nurse says that although the patient’s respiratory rate has come down slightly she is looking more unwell. Her repeat gas shows:

  • pO2: 9.8 (10–14)
  • BE: -2 (-2 to +2)
  • This patient has asthma, ongoing difficulty in breathing and a rising CO2 (the fact that it is in the normal range is irrelevant) .
  • This is an extremely worrying sign as it shows that the patient is tiring.
  • This patient should be managed in a high dependency area and closely monitored for further deterioration.

Question 10.

A 62 year-old woman with a history of diabetes and a long smoking history presents to the emergency department with worsening shortness of breath. On auscultation of the chest there are widespread crackles and you notice moderate ankle oedema. ABG shows:

  • pH: 7.20 (7.35-7.45)
  • pO2: 8.9 (10–14)
  • pCO2: 6.3 (4.5–6.0)
  • HCO3: 17 (22-26)
  • BE: -8 (-2 to +2)
  • Note that despite the low pH the pCO2 is also high.
  • This is a picture of a mixed respiratory and metabolic acidosis.
  • Given the history of diabetes and ankle swelling, renal failure is a unifying diagnosis with pulmonary oedema contributing to a respiratory acidosis whilst the failure to clear acids causes a metabolic acidosis.

Click here for further questions on ABGs

…and click here to learn the best way to interpret abgs.

Perfect revision for MRCP PACES, OSCES and medical student finals

Diabetes Case Studies: Real Problems, Practical Solutions

Diabetes Case Studies : Real Problems, Practical Solutions

  • Book Chapters
  • Figures & tables
  • Supplementary Data
  • Peer Review
  • Cite Icon Cite
  • Get Permissions

Diabetes Case Studies : Real Problems, Practical Solutions Edited by: Boris Draznin, MD, PhD, Cecilia C. Low Wang, MD, FACP, Daniel J. Rubin, MD, MSc, FACE https://doi.org/10.2337/9781580405713 ISBN (print): 978-1-58040-571-3 Publisher: American Diabetes Association

Download citation file:

  • Ris (Zotero)
  • Reference Manager

Table of Contents

  • Notes Open the PDF Link PDF for Notes in another window
  • Preface Open the PDF Link PDF for Preface in another window
  • Introduction Open the PDF Link PDF for Introduction in another window
  • Case 1: Maturity-Onset Diabetes of the Young (MODY) as a Diagnostic Possibility By Suzi Kochar, MD ; Suzi Kochar, MD 1 Endocrinology Division, Department of Medicine, Rosalind Franklin University of Medicine and Science/Chicago Medical School, North Chicago, IL. 2 Endocrinology Section, Department of Medicine, Captain James A. Lovell Health Care Center, North Chicago, IL. Search for other works by this author on: This Site PubMed Google Scholar Janice L. Gilden, MS, MD, FCP, FACE Janice L. Gilden, MS, MD, FCP, FACE 1 Endocrinology Division, Department of Medicine, Rosalind Franklin University of Medicine and Science/Chicago Medical School, North Chicago, IL. 2 Endocrinology Section, Department of Medicine, Captain James A. Lovell Health Care Center, North Chicago, IL. Search for other works by this author on: This Site PubMed Google Scholar Doi: https://doi.org/10.2337/9781580405713.01 Open the PDF Link PDF for Case 1: Maturity-Onset Diabetes of the Young (MODY) as a Diagnostic Possibility in another window
  • Case 2: Diagnosis of Coexistent Maturity-Onset Diabetes of the Young in a Patient with Type 1 Diabetes By Shazli Azmi, MBChB ; Shazli Azmi, MBChB 1 Institute of Human Development, Center for Endocrinology and Diabetes, University of Manchester. 2 Department of Medicine and Manchester Diabetes Center, University of Manchester and Central Manchester NHS Foundation Trust. Search for other works by this author on: This Site PubMed Google Scholar Rayaz A. Malik, MBChB, PhD Rayaz A. Malik, MBChB, PhD 1 Institute of Human Development, Center for Endocrinology and Diabetes, University of Manchester. 2 Department of Medicine and Manchester Diabetes Center, University of Manchester and Central Manchester NHS Foundation Trust. Search for other works by this author on: This Site PubMed Google Scholar Doi: https://doi.org/10.2337/9781580405713.02 Open the PDF Link PDF for Case 2: Diagnosis of Coexistent Maturity-Onset Diabetes of the Young in a Patient with Type 1 Diabetes in another window
  • Case 3: An Unusual Clinical Presentation of Diabetes Eventually Diagnosed as a Monogenic Form By Vince N. Montes, MD ; Vince N. Montes, MD 1 University of Washington, Division of Metabolism, Endocrinology, and Nutrition, Seattle, WA. Search for other works by this author on: This Site PubMed Google Scholar Alan Chait, MD ; Alan Chait, MD 1 University of Washington, Division of Metabolism, Endocrinology, and Nutrition, Seattle, WA. Search for other works by this author on: This Site PubMed Google Scholar Craig E. Taplin, MD Craig E. Taplin, MD 2 Department of Pediatrics, University of Washington, Seattle Children’s Hospital, Division of Endocrinology and Diabetes, Seattle, WA. Search for other works by this author on: This Site PubMed Google Scholar Doi: https://doi.org/10.2337/9781580405713.03 Open the PDF Link PDF for Case 3: An Unusual Clinical Presentation of Diabetes Eventually Diagnosed as a Monogenic Form in another window
  • Case 4: A Case of Monogenic Diabetes By Robert H. Slover, MD Robert H. Slover, MD 1 Professor of Pediatrics, University of Colorado School of Medicine, Denver, CO; Director of Pediatrics, Barbara Davis Center for Childhood Diabetes; Wagner Family Chair in Childhood Diabetes. Search for other works by this author on: This Site PubMed Google Scholar Doi: https://doi.org/10.2337/9781580405713.04 Open the PDF Link PDF for Case 4: A Case of Monogenic Diabetes in another window
  • Case 5: Recurrent Ketoacidosis: Lessons from Multiple Clinical Presentations By Eli Ipp, MD ; Eli Ipp, MD 1 Division of Endocrinology, Los Angeles Biomedical Research Institute at Harbor–UCLA Medical Center, Los Angeles, CA. Search for other works by this author on: This Site PubMed Google Scholar Kristina Djekic, MS Kristina Djekic, MS 1 Division of Endocrinology, Los Angeles Biomedical Research Institute at Harbor–UCLA Medical Center, Los Angeles, CA. Search for other works by this author on: This Site PubMed Google Scholar Doi: https://doi.org/10.2337/9781580405713.05 Open the PDF Link PDF for Case 5: Recurrent Ketoacidosis: Lessons from Multiple Clinical Presentations in another window
  • Case 6: Ketoacidosis in a Patient with Type 1 Diabetes on a Low-Calorie Meal Replacement Diet By Katy Brown, DO ; Katy Brown, DO 1 Fellow in Endocrinology, University of Colorado, School of Medicine, Denver, CO. Search for other works by this author on: This Site PubMed Google Scholar Daniel Bessesen, MD Daniel Bessesen, MD 2 University of Colorado, School of Medicine, Chief of Endocrinology, Denver Health Medical Center, Denver, CO. Search for other works by this author on: This Site PubMed Google Scholar Doi: https://doi.org/10.2337/9781580405713.06 Open the PDF Link PDF for Case 6: Ketoacidosis in a Patient with Type 1 Diabetes on a Low-Calorie Meal Replacement Diet in another window
  • Case 7: Reevaluation after Ketoacidosis Presentation By Paulina Cruz-Bravo, MD ; Paulina Cruz-Bravo, MD 1 Fellowship in Endocrinology, Diabetes and Metabolism, Division of Endocrinology, Metabolism and Lipid Research, Washington University School of Medicine, St. Louis, MO. Search for other works by this author on: This Site PubMed Google Scholar Janet B. McGill, MD Janet B. McGill, MD 1 Fellowship in Endocrinology, Diabetes and Metabolism, Division of Endocrinology, Metabolism and Lipid Research, Washington University School of Medicine, St. Louis, MO. Search for other works by this author on: This Site PubMed Google Scholar Doi: https://doi.org/10.2337/9781580405713.07 Open the PDF Link PDF for Case 7: Reevaluation after Ketoacidosis Presentation in another window
  • Case 8: Metabolic Syndrome-Related Comorbidities Typical of Older Adulthood Complicate Diabetic Ketoacidosis in a Youth with Type 2 Diabetes By Kristen Nadeau, MD, MS Kristen Nadeau, MD, MS 1 University of Colorado Denver/Children’s Hospital Colorado, Aurora, CO. Search for other works by this author on: This Site PubMed Google Scholar Doi: https://doi.org/10.2337/9781580405713.08 Open the PDF Link PDF for Case 8: Metabolic Syndrome-Related Comorbidities Typical of Older Adulthood Complicate Diabetic Ketoacidosis in a Youth with Type 2 Diabetes in another window
  • Case 9: Not Your Usual Diabetic Ketoacidosis By Ruchi Gaba, MD ; Ruchi Gaba, MD 1 Division of Endocrinology Diabetes and Metabolism, Baylor College of Medicine, Houston, TX. Search for other works by this author on: This Site PubMed Google Scholar Susan L. Samson, MD, PhD ; Susan L. Samson, MD, PhD 2 Department of Medicine, Baylor College of Medicine, Houston, TX. Search for other works by this author on: This Site PubMed Google Scholar Alan J. Garber, MD, PhD, FACE Alan J. Garber, MD, PhD, FACE 3 Departments of Medicine, Molecular and Cellular Biology, Biochemistry and Molecular Biology, Baylor College of Medicine, Houston, TX. Search for other works by this author on: This Site PubMed Google Scholar Doi: https://doi.org/10.2337/9781580405713.09 Open the PDF Link PDF for Case 9: Not Your Usual Diabetic Ketoacidosis in another window
  • Case 10: Ketosis-Prone Diabetes By David Saxon, MD ; David Saxon, MD 1 Division of Endocrinology, Diabetes and Metabolism, University of Colorado, Aurora CO. Search for other works by this author on: This Site PubMed Google Scholar Neda Rasouli, MD Neda Rasouli, MD 1 Division of Endocrinology, Diabetes and Metabolism, University of Colorado, Aurora CO. 2 Denver Veterans Affairs Medical Center, Denver CO. Search for other works by this author on: This Site PubMed Google Scholar Doi: https://doi.org/10.2337/9781580405713.10 Open the PDF Link PDF for Case 10: Ketosis-Prone Diabetes in another window
  • Case 11: From a Total Daily Dose of Insulin of 415 Units to No Insulin: A Case of Ketosis-Prone Diabetes By Caroline T. Nguyen, MD ; Caroline T. Nguyen, MD 1 Keck School of Medicine, Division of Endocrinology, Diabetes, and Metabolism, University of Southern California, Los Angeles, CA. Search for other works by this author on: This Site PubMed Google Scholar Jorge H. Mestman, MD Jorge H. Mestman, MD 1 Keck School of Medicine, Division of Endocrinology, Diabetes, and Metabolism, University of Southern California, Los Angeles, CA. Search for other works by this author on: This Site PubMed Google Scholar Doi: https://doi.org/10.2337/9781580405713.11 Open the PDF Link PDF for Case 11: From a Total Daily Dose of Insulin of 415 Units to No Insulin: A Case of Ketosis-Prone Diabetes in another window
  • Case 12: The Worst Case Scenario: Severe HHS in a Relatively Young Man By Elizabeth Herman, MD ; Elizabeth Herman, MD 1 Boston University School of Medicine, Section of Endocrinology, Diabetes, Nutrition and Weight Management, Boston MA. Search for other works by this author on: This Site PubMed Google Scholar Marie E. McDonnell, MD Marie E. McDonnell, MD 1 Boston University School of Medicine, Section of Endocrinology, Diabetes, Nutrition and Weight Management, Boston MA. Search for other works by this author on: This Site PubMed Google Scholar Doi: https://doi.org/10.2337/9781580405713.12 Open the PDF Link PDF for Case 12: The Worst Case Scenario: Severe HHS in a Relatively Young Man in another window
  • Case 13: Unusual Cause of New-Onset Diabetes By Kelsey M. Shikuma, MD ; Kelsey M. Shikuma, MD 1 Keck School of Medicine, Division of Endocrinology, Diabetes and Metabolism, University of Southern California, Los Angeles, CA. Search for other works by this author on: This Site PubMed Google Scholar Jorge H. Mestman, MD Jorge H. Mestman, MD 1 Keck School of Medicine, Division of Endocrinology, Diabetes and Metabolism, University of Southern California, Los Angeles, CA. Search for other works by this author on: This Site PubMed Google Scholar Doi: https://doi.org/10.2337/9781580405713.13 Open the PDF Link PDF for Case 13: Unusual Cause of New-Onset Diabetes in another window
  • Case 14: What Type of Diabetes? By Anthony L. McCall, PhD, FACP Anthony L. McCall, PhD, FACP 1 University of Virginia School of Medicine and Health System, Charlottesville, VA. Search for other works by this author on: This Site PubMed Google Scholar Doi: https://doi.org/10.2337/9781580405713.14 Open the PDF Link PDF for Case 14: What Type of Diabetes? in another window
  • Case 15: Type 1 Diabetes versus LADA in a Patient Misdiagnosed with Type 2 Diabetes By Cecilia C. Low Wang, MD Cecilia C. Low Wang, MD 1 Department of Medicine Division of Endocrinology, Metabolism and Diabetes; University of Colorado Anschutz Medical Campus School of Medicine, Aurora, CO. Search for other works by this author on: This Site PubMed Google Scholar Doi: https://doi.org/10.2337/9781580405713.15 Open the PDF Link PDF for Case 15: Type 1 Diabetes versus LADA in a Patient Misdiagnosed with Type 2 Diabetes in another window
  • Case 16: Type 1 Diabetes Can Present at Any Age By David S.H. Bell, MB David S.H. Bell, MB 1 Clinical Professor, University of Alabama. Birmingham, AL. Search for other works by this author on: This Site PubMed Google Scholar Doi: https://doi.org/10.2337/9781580405713.16 Open the PDF Link PDF for Case 16: Type 1 Diabetes Can Present at Any Age in another window
  • Case 17: Is This Type 2 Diabetes, Type 1 Diabetes, or Late Autoimmune Diabetes in Adults? By Devjit Tripathy, MD, PhD ; Devjit Tripathy, MD, PhD 1 Audie L. Murphy VA Hospital, South Texas Veterans Health Care System, San Antonio, TX. Search for other works by this author on: This Site PubMed Google Scholar Sheila Pinkson, MPAS, PA-C ; Sheila Pinkson, MPAS, PA-C 1 Audie L. Murphy VA Hospital, South Texas Veterans Health Care System, San Antonio, TX. Search for other works by this author on: This Site PubMed Google Scholar Maureen Koops, MD ; Maureen Koops, MD 1 Audie L. Murphy VA Hospital, South Texas Veterans Health Care System, San Antonio, TX. Search for other works by this author on: This Site PubMed Google Scholar Ralph A. DeFronzo, MD Ralph A. DeFronzo, MD 1 Audie L. Murphy VA Hospital, South Texas Veterans Health Care System, San Antonio, TX. Search for other works by this author on: This Site PubMed Google Scholar Doi: https://doi.org/10.2337/9781580405713.17 Open the PDF Link PDF for Case 17: Is This Type 2 Diabetes, Type 1 Diabetes, or Late Autoimmune Diabetes in Adults? in another window
  • Case 18: A Common Misdiagnosis By Irene E. Schauer, MD, PhD Irene E. Schauer, MD, PhD 1 Department of Medicine, University of Colorado Anschutz Medical Campus School of Medicine, Aurora, CO. 2 Research/Endocrine Sections, Denver Veterans Affairs Medical Center, Denver, CO. Search for other works by this author on: This Site PubMed Google Scholar Doi: https://doi.org/10.2337/9781580405713.18 Open the PDF Link PDF for Case 18: A Common Misdiagnosis in another window
  • Case 19: Almost All Nonobese Young People with an Acute Onset of Diabetes Have Type 1 Diabetes By David S.H. Bell, MB David S.H. Bell, MB 1 Clinical Professor, University of Alabama, Birmingham, AL. Search for other works by this author on: This Site PubMed Google Scholar Doi: https://doi.org/10.2337/9781580405713.19 Open the PDF Link PDF for Case 19: Almost All Nonobese Young People with an Acute Onset of Diabetes Have Type 1 Diabetes in another window
  • Case 20: Symptomatic Postprandial Hyperglycemia By Amita Maturu, MD ; Amita Maturu, MD 1 Division of Endocrinology, Diabetes and Metabolism, University of Colorado, Aurora, CO. Search for other works by this author on: This Site PubMed Google Scholar Michael McDermott, MD Michael McDermott, MD 1 Division of Endocrinology, Diabetes and Metabolism, University of Colorado, Aurora, CO. Search for other works by this author on: This Site PubMed Google Scholar Doi: https://doi.org/10.2337/9781580405713.20 Open the PDF Link PDF for Case 20: Symptomatic Postprandial Hyperglycemia in another window
  • Case 21: A Patient with Newly Diagnosed, Asymptomatic Hyperglycemia By Aidan McElduff, MD Aidan McElduff, MD 1 Discipline of Medicine, Sydney University, Sydney, NSW, Australia. Search for other works by this author on: This Site PubMed Google Scholar Doi: https://doi.org/10.2337/9781580405713.21 Open the PDF Link PDF for Case 21: A Patient with Newly Diagnosed, Asymptomatic Hyperglycemia in another window
  • Case 22: Glucokinase Maturity-Onset Diabetes of the Young and Pregnancy By Jill Apel, MD ; Jill Apel, MD 1 Division of Endocrinology, Rush University Medical Center, Chicago, IL. Search for other works by this author on: This Site PubMed Google Scholar Chung-Kay Koh, MD Chung-Kay Koh, MD 1 Division of Endocrinology, Rush University Medical Center, Chicago, IL. Search for other works by this author on: This Site PubMed Google Scholar Doi: https://doi.org/10.2337/9781580405713.22 Open the PDF Link PDF for Case 22: Glucokinase Maturity-Onset Diabetes of the Young and Pregnancy in another window
  • Case 23: Latent Autoimmune Diabetes of the Adult (LADA) in an Elderly Patient By Monica Shah, MD ; Monica Shah, MD 1 Division of Endocrinology, Rush University, Chicago, IL. Search for other works by this author on: This Site PubMed Google Scholar Mahtab Sohrevardi, MD ; Mahtab Sohrevardi, MD 1 Division of Endocrinology, Rush University, Chicago, IL. Search for other works by this author on: This Site PubMed Google Scholar David Baldwin, MD David Baldwin, MD 1 Division of Endocrinology, Rush University, Chicago, IL. Search for other works by this author on: This Site PubMed Google Scholar Doi: https://doi.org/10.2337/9781580405713.23 Open the PDF Link PDF for Case 23: Latent Autoimmune Diabetes of the Adult (LADA) in an Elderly Patient in another window
  • Case 24: A Diagnostic Dilemma in a Patient with Elevated Glycosylated Hemoglobin By Shalini Paturi, MD ; Shalini Paturi, MD 1 Endocrinology Division, Department of Medicine, Rosalind Franklin University of Medicine and Science/Chicago Medical School, North Chicago, IL. 2 Endocrinology Section, Department of Medicine, Captain James A. Lovell Federal Health Care Center, North Chicago, IL. Search for other works by this author on: This Site PubMed Google Scholar Janice L. Gilden, MS, MD, FCP, FACE Janice L. Gilden, MS, MD, FCP, FACE 1 Endocrinology Division, Department of Medicine, Rosalind Franklin University of Medicine and Science/Chicago Medical School, North Chicago, IL. 2 Endocrinology Section, Department of Medicine, Captain James A. Lovell Federal Health Care Center, North Chicago, IL. Search for other works by this author on: This Site PubMed Google Scholar Doi: https://doi.org/10.2337/9781580405713.24 Open the PDF Link PDF for Case 24: A Diagnostic Dilemma in a Patient with Elevated Glycosylated Hemoglobin in another window
  • Case 25: An Unexplained Decline in HbA1c in Spite of Persistent Hyperglycemia By David S.H. Bell, MB David S.H. Bell, MB 1 Clinical Professor, University of Alabama, Birmingham, AL. Search for other works by this author on: This Site PubMed Google Scholar Doi: https://doi.org/10.2337/9781580405713.25 Open the PDF Link PDF for Case 25: An Unexplained Decline in HbA1c in Spite of Persistent Hyperglycemia in another window
  • Case 26: What to Do with Discrepant HbA1c and SMBG Results? The Utility of Fructosamine and Glycated Albumin By Tatiana Gandrabura, MD ; Tatiana Gandrabura, MD 1 Temple University School of Medicine, Philadelphia, PA. Search for other works by this author on: This Site PubMed Google Scholar Daniel J. Rubin, MD, MSc Daniel J. Rubin, MD, MSc 1 Temple University School of Medicine, Philadelphia, PA. Search for other works by this author on: This Site PubMed Google Scholar Doi: https://doi.org/10.2337/9781580405713.26 Open the PDF Link PDF for Case 26: What to Do with Discrepant HbA1c and SMBG Results? The Utility of Fructosamine and Glycated Albumin in another window
  • Case 27: A “Tricky” Low HbA 1c By Chiara Mazzucchelli, MD ; Chiara Mazzucchelli, MD 1 Department of Internal Medicine, University of Genova, Genova, Italy. Search for other works by this author on: This Site PubMed Google Scholar Caterina Bordone, MD ; Caterina Bordone, MD 1 Department of Internal Medicine, University of Genova, Genova, Italy. Search for other works by this author on: This Site PubMed Google Scholar Davide Maggi, MD, PhD ; Davide Maggi, MD, PhD 1 Department of Internal Medicine, University of Genova, Genova, Italy. Search for other works by this author on: This Site PubMed Google Scholar Renzo Cordera, MD Renzo Cordera, MD 1 Department of Internal Medicine, University of Genova, Genova, Italy. Search for other works by this author on: This Site PubMed Google Scholar Doi: https://doi.org/10.2337/9781580405713.27 Open the PDF Link PDF for Case 27: A “Tricky” Low HbA<sub>1c</sub> in another window
  • Case 28: Use of Insulin U-500 in a Patient with Severe Insulin Resistance By Hussain Mahmud, MBBS ; Hussain Mahmud, MBBS 1 Clinical Assistant Professor of Medicine, Division of Endocrinology, University of Pittsburgh Medical Center, Pittsburgh, PA. Search for other works by this author on: This Site PubMed Google Scholar Mary T. Korytkowski, MD Mary T. Korytkowski, MD 2 Professor of Medicine, Division of Endocrinology, University of Pittsburgh Medical Center, Pittsburgh, PA. Search for other works by this author on: This Site PubMed Google Scholar Doi: https://doi.org/10.2337/9781580405713.28 Open the PDF Link PDF for Case 28: Use of Insulin U-500 in a Patient with Severe Insulin Resistance in another window
  • Case 29: Effective Use of U-500 Insulin via Insulin Pump in a Type 2 Diabetes Patient with Severe Insulin Resistance By Vijay babu Balakrishnan, MD ; Vijay babu Balakrishnan, MD 1 Section of Endocrinology, Diabetes and Metabolism, Temple University School of Medicine, Philadelphia, PA. Search for other works by this author on: This Site PubMed Google Scholar Elias S. Siraj, MD Elias S. Siraj, MD 1 Section of Endocrinology, Diabetes and Metabolism, Temple University School of Medicine, Philadelphia, PA. Search for other works by this author on: This Site PubMed Google Scholar Doi: https://doi.org/10.2337/9781580405713.29 Open the PDF Link PDF for Case 29: Effective Use of U-500 Insulin via Insulin Pump in a Type 2 Diabetes Patient with Severe Insulin Resistance in another window
  • Case 30: U-500 Insulin Pump Case By Anthony L. McCall, MD, PhD, FACP Anthony L. McCall, MD, PhD, FACP 1 James M. Moss Professor of Medicine, University of Virginia, School of Medicine and Health System, Charlottesville, VA. Search for other works by this author on: This Site PubMed Google Scholar Doi: https://doi.org/10.2337/9781580405713.30 Open the PDF Link PDF for Case 30: U-500 Insulin Pump Case in another window
  • Case 31: Difficulties in Managing Patients with Insulin Resistance: Alternatives to U-500 Insulin By Narmada Movva, MD ; Narmada Movva, MD 1 Endocrinology Division, Department of Medicine, Rosalind Franklin University of Medicine and Science/Chicago Medical School, North Chicago, IL. 2 Endocrinology Section, Department of Medicine, Captain James A. Lovell Federal Health Care Center, North Chicago, IL. Search for other works by this author on: This Site PubMed Google Scholar Boby G. Theckedath, MD, FACE ; Boby G. Theckedath, MD, FACE 1 Endocrinology Division, Department of Medicine, Rosalind Franklin University of Medicine and Science/Chicago Medical School, North Chicago, IL. 2 Endocrinology Section, Department of Medicine, Captain James A. Lovell Federal Health Care Center, North Chicago, IL. Search for other works by this author on: This Site PubMed Google Scholar Janice L. Gilden, MS, MD, FCP, FACE Janice L. Gilden, MS, MD, FCP, FACE 1 Endocrinology Division, Department of Medicine, Rosalind Franklin University of Medicine and Science/Chicago Medical School, North Chicago, IL. 2 Endocrinology Section, Department of Medicine, Captain James A. Lovell Federal Health Care Center, North Chicago, IL. Search for other works by this author on: This Site PubMed Google Scholar Doi: https://doi.org/10.2337/9781580405713.31 Open the PDF Link PDF for Case 31: Difficulties in Managing Patients with Insulin Resistance: Alternatives to U-500 Insulin in another window
  • Case 32: Management Issues in the Syndrome of Autoantibodies to the Insulin Receptor (Type B Insulin Resistance) By Elaine Cochran, MSN, CRNP ; Elaine Cochran, MSN, CRNP 1 National Institutes of Health, National Institute of Diabetes, Digestive, and Kidney Diseases, Diabetes, Endocrine, and Obesity Branch, Bethesda, MD. Search for other works by this author on: This Site PubMed Google Scholar Rebecca Brown, MD, MhSc ; Rebecca Brown, MD, MhSc 1 National Institutes of Health, National Institute of Diabetes, Digestive, and Kidney Diseases, Diabetes, Endocrine, and Obesity Branch, Bethesda, MD. Search for other works by this author on: This Site PubMed Google Scholar Phillip Gorden, MD Phillip Gorden, MD 1 National Institutes of Health, National Institute of Diabetes, Digestive, and Kidney Diseases, Diabetes, Endocrine, and Obesity Branch, Bethesda, MD. Search for other works by this author on: This Site PubMed Google Scholar Doi: https://doi.org/10.2337/9781580405713.32 Open the PDF Link PDF for Case 32: Management Issues in the Syndrome of Autoantibodies to the Insulin Receptor (Type B Insulin Resistance) in another window
  • Case 33: Type B Insulin Resistance By Nisha Bincent Jacob, APN, FNP-C, CDE, MBA ; Nisha Bincent Jacob, APN, FNP-C, CDE, MBA 1 University of Texas Southwestern Medical Center, Department of Internal Medicine, Division of Endocrinology and Metabolism, Dallas, TX. Search for other works by this author on: This Site PubMed Google Scholar Hilary Trevino, APN, FNP-C ; Hilary Trevino, APN, FNP-C 1 University of Texas Southwestern Medical Center, Department of Internal Medicine, Division of Endocrinology and Metabolism, Dallas, TX. Search for other works by this author on: This Site PubMed Google Scholar Chanhaeng Rhee, MD, MBA Chanhaeng Rhee, MD, MBA 1 University of Texas Southwestern Medical Center, Department of Internal Medicine, Division of Endocrinology and Metabolism, Dallas, TX. Search for other works by this author on: This Site PubMed Google Scholar Doi: https://doi.org/10.2337/9781580405713.33 Open the PDF Link PDF for Case 33: Type B Insulin Resistance in another window
  • Case 34: Adhering or Not? That Is the Question: A Case of Glucolipotoxicity and Concentrated Insulin By Sanaa Deshmukh, MD ; Sanaa Deshmukh, MD 1 Division of Endocrinology, Diabetes and Metabolism, Department of Medicine, University of Missouri, Columbia, MO. Search for other works by this author on: This Site PubMed Google Scholar Rino Buzzola, MD ; Rino Buzzola, MD 1 Division of Endocrinology, Diabetes and Metabolism, Department of Medicine, University of Missouri, Columbia, MO. Search for other works by this author on: This Site PubMed Google Scholar Mariana Touza, MD ; Mariana Touza, MD 1 Division of Endocrinology, Diabetes and Metabolism, Department of Medicine, University of Missouri, Columbia, MO. Search for other works by this author on: This Site PubMed Google Scholar Michael Gardner, MD ; Michael Gardner, MD 1 Division of Endocrinology, Diabetes and Metabolism, Department of Medicine, University of Missouri, Columbia, MO. Search for other works by this author on: This Site PubMed Google Scholar James R. Sowers, MD James R. Sowers, MD 1 Division of Endocrinology, Diabetes and Metabolism, Department of Medicine, University of Missouri, Columbia, MO. Search for other works by this author on: This Site PubMed Google Scholar Doi: https://doi.org/10.2337/9781580405713.34 Open the PDF Link PDF for Case 34: Adhering or Not? That Is the Question: A Case of Glucolipotoxicity and Concentrated Insulin in another window
  • Case 35: Cosecreting Adrenal Tumor Causing Severe Insulin Resistance By Kathya Rivera, MD ; Kathya Rivera, MD 1 Division of Endocrinology, Diabetes and Metabolism, University of Florida, Gainesville, FL. Search for other works by this author on: This Site PubMed Google Scholar Kenneth Cusi, MD ; Kenneth Cusi, MD 1 Division of Endocrinology, Diabetes and Metabolism, University of Florida, Gainesville, FL. 2 Division of Endocrinology, Diabetes and Metabolism, Malcom Randall VAMC, Gainesville, FL. Search for other works by this author on: This Site PubMed Google Scholar Catherine Edwards, MD Catherine Edwards, MD 1 Division of Endocrinology, Diabetes and Metabolism, University of Florida, Gainesville, FL. Search for other works by this author on: This Site PubMed Google Scholar Doi: https://doi.org/10.2337/9781580405713.35 Open the PDF Link PDF for Case 35: Cosecreting Adrenal Tumor Causing Severe Insulin Resistance in another window
  • Case 36: Management of Severe Insulin Resistance in a Pregnant Patient with Type 2 Diabetes: The Use of U-500 Regular Insulin via Continuous Subcutaneous Infusion By Tiffany Hor, MD ; Tiffany Hor, MD 1 Division of Endocrinology, Rush University, Chicago, IL. Search for other works by this author on: This Site PubMed Google Scholar David Baldwin, MD David Baldwin, MD 1 Division of Endocrinology, Rush University, Chicago, IL. Search for other works by this author on: This Site PubMed Google Scholar Doi: https://doi.org/10.2337/9781580405713.36 Open the PDF Link PDF for Case 36: Management of Severe Insulin Resistance in a Pregnant Patient with Type 2 Diabetes: The Use of U-500 Regular Insulin via Continuous Subcutaneous Infusion in another window
  • Case 37: Diabetes in Hereditary Hemochromatosis By Donald A. McClain, MD, PhD Donald A. McClain, MD, PhD 1 Department of Internal Medicine, University of Utah School of Medicine, Salt Lake City, UT. Search for other works by this author on: This Site PubMed Google Scholar Doi: https://doi.org/10.2337/9781580405713.37 Open the PDF Link PDF for Case 37: Diabetes in Hereditary Hemochromatosis in another window
  • Case 38: Challenging Insights from Albuminuria Early in the Course of Disease By Cem Demirci, MD ; Cem Demirci, MD 1 Department of Endocrinology, Connecticut Children’s Medical Center, Hartford, CT. Search for other works by this author on: This Site PubMed Google Scholar Vered Lewy-Weiss, MD ; Vered Lewy-Weiss, MD 2 Children’s Hospital of Pittsburgh of UPMC, Pittsburgh, PA. Search for other works by this author on: This Site PubMed Google Scholar Mark A. Sperling, MBBS, FRACP Mark A. Sperling, MBBS, FRACP 2 Children’s Hospital of Pittsburgh of UPMC, Pittsburgh, PA. Search for other works by this author on: This Site PubMed Google Scholar Doi: https://doi.org/10.2337/9781580405713.38 Open the PDF Link PDF for Case 38: Challenging Insights from Albuminuria Early in the Course of Disease in another window
  • Case 39: Copresentation of Addison’s Disease and Type 1 Diabetes in a 9-Year-Old Boy By Shideh Majidi, MD ; Shideh Majidi, MD 1 Pediatric Endocrinology Fellow, University of Colorado Denver, Aurora, CO. Search for other works by this author on: This Site PubMed Google Scholar Jennifer Raymond, MD, MCR Jennifer Raymond, MD, MCR 2 Assistant Professor, Pediatric Endocrinology, Barbara Davis Center for Diabetes, University of Colorado Denver, Aurora, CO. Search for other works by this author on: This Site PubMed Google Scholar Doi: https://doi.org/10.2337/9781580405713.39 Open the PDF Link PDF for Case 39: Copresentation of Addison’s Disease and Type 1 Diabetes in a 9-Year-Old Boy in another window
  • Case 40: Diagnosis of Addison’s Disease and Type 1 Diabetes in Twin Boys By Andrea Dann Urban, MSN, PNP-BC, CDE ; Andrea Dann Urban, MSN, PNP-BC, CDE 1 Yale Children’s Diabetes Program, Associate Clinical Faculty, Yale University School of Nursing, New Haven, CT. Search for other works by this author on: This Site PubMed Google Scholar William V. Tamborlane, MD William V. Tamborlane, MD 2 Professor of Pediatrics, Department of Pediatrics, Chief, Pediatric Endocrinology, Deputy Director, Yale Center for Clinical Investigation, Director, Children’s Diabetes Program, Yale School of Medicine, New Haven, CT. Search for other works by this author on: This Site PubMed Google Scholar Doi: https://doi.org/10.2337/9781580405713.40 Open the PDF Link PDF for Case 40: Diagnosis of Addison’s Disease and Type 1 Diabetes in Twin Boys in another window
  • Case 41: New-Onset Type 1 Diabetes, Addison’s Disease, and Hypothyroidism: A Case of Autoimmune Polyendocrine Syndrome Type 2 By Lauren Golden, MD ; Lauren Golden, MD 1 Assistant Professor Clinical Medicine, Naomi Berrie Diabetes Center, Columbia University Medical Center, New York, NY. Search for other works by this author on: This Site PubMed Google Scholar Robin Goland, MD Robin Goland, MD 2 J. Merrill Eastman Professor of Clinical Diabetes, Columbia University Medical Center, New York, NY. Search for other works by this author on: This Site PubMed Google Scholar Doi: https://doi.org/10.2337/9781580405713.41 Open the PDF Link PDF for Case 41: New-Onset Type 1 Diabetes, Addison’s Disease, and Hypothyroidism: A Case of Autoimmune Polyendocrine Syndrome Type 2 in another window
  • Case 42: The Slow Progression of Type 1 Diabetes as Part of Autoimmune Polyendocrine Syndrome Type 2 By Natalia Pertzeva, MD ; Natalia Pertzeva, MD 1 Dnepropetrovsk Medical Academy, Dnepropetrovsk, Ukraine. Search for other works by this author on: This Site PubMed Google Scholar Boris Mankovsky, MD, PhD Boris Mankovsky, MD, PhD 2 National Medical Academy for Postgraduate Education, Kiev, Ukraine. Search for other works by this author on: This Site PubMed Google Scholar Doi: https://doi.org/10.2337/9781580405713.42 Open the PDF Link PDF for Case 42: The Slow Progression of Type 1 Diabetes as Part of Autoimmune Polyendocrine Syndrome Type 2 in another window
  • Case 43: Atypical Type 2 Diabetes with Profound Dyslipidemia By Jeremy H. Pettus, MD ; Jeremy H. Pettus, MD 1 Endocrinology Fellow, Division of Endocrinology and Metabolism, University of California, San Diego, CA. Search for other works by this author on: This Site PubMed Google Scholar Robert R. Henry, MD Robert R. Henry, MD 2 Professor of Medicine, Division of Endocrinology and Metabolism, University of California, San Diego, CA; Chief, Section of Diabetes, Endocrinology and Metabolism; Director, Center for Metabolic Research, VA San Diego Healthcare System, San Diego, CA. Search for other works by this author on: This Site PubMed Google Scholar Doi: https://doi.org/10.2337/9781580405713.43 Open the PDF Link PDF for Case 43: Atypical Type 2 Diabetes with Profound Dyslipidemia in another window
  • Case 44: Patient with Diabetes Who Has Hemiballismus By Miriam Padilla, MD, CDE ; Miriam Padilla, MD, CDE 1 Fellow, Department of Endocrinology and Diabetes at University of Southern California. Search for other works by this author on: This Site PubMed Google Scholar Jorge Mestman, MD Jorge Mestman, MD 2 Professor of Clinical Medicine in the Department of Endocrinology and Diabetes at University of Southern California, Professor of Clinical Medicine in the Department of Obstetrics and Gynecology at University of Southern California. Search for other works by this author on: This Site PubMed Google Scholar Doi: https://doi.org/10.2337/9781580405713.44 Open the PDF Link PDF for Case 44: Patient with Diabetes Who Has Hemiballismus in another window
  • Case 45: A Case of Diabetic Myonecrosis By Umal Azmat, MD ; Umal Azmat, MD 1 Division of Endocrinology, Diabetes and Metabolism, Ohio State University Wexner Medical Center, Columbus, OH. Search for other works by this author on: This Site PubMed Google Scholar Jason E. Payne, MD ; Jason E. Payne, MD 2 Division of Radiology, Ohio State University Wexner Medical Center, Columbus, OH. Search for other works by this author on: This Site PubMed Google Scholar Kathleen Dungan, MD ; Kathleen Dungan, MD 1 Division of Endocrinology, Diabetes and Metabolism, Ohio State University Wexner Medical Center, Columbus, OH. Search for other works by this author on: This Site PubMed Google Scholar Steven W. Ing, MD Steven W. Ing, MD 1 Division of Endocrinology, Diabetes and Metabolism, Ohio State University Wexner Medical Center, Columbus, OH. Search for other works by this author on: This Site PubMed Google Scholar Doi: https://doi.org/10.2337/9781580405713.45 Open the PDF Link PDF for Case 45: A Case of Diabetic Myonecrosis in another window
  • Case 46: A Case of Stiff Person Syndrome in a Patient with Type 1 Diabetes By Matthew P. Gilbert, DO, MPH ; Matthew P. Gilbert, DO, MPH 1 Department of Medicine, Division of Endocrinology and Diabetes, University of Vermont College of Medicine, Burlington, VT. Search for other works by this author on: This Site PubMed Google Scholar Muriel H. Nathan, MD, PhD Muriel H. Nathan, MD, PhD 1 Department of Medicine, Division of Endocrinology and Diabetes, University of Vermont College of Medicine, Burlington, VT. Search for other works by this author on: This Site PubMed Google Scholar Doi: https://doi.org/10.2337/9781580405713.46 Open the PDF Link PDF for Case 46: A Case of Stiff Person Syndrome in a Patient with Type 1 Diabetes in another window
  • Case 47: Stiff Person Syndrome in a Patient with Multiple Autoimmune Diseases By Jing Hughes, MD, PhD ; Jing Hughes, MD, PhD 1 Professor of Medicine, Director, Fellowship in Endocrinology, Diabetes and Metabolism, Division of Endocrinology, Metabolism and Lipid Research, Washington University School of Medicine, St. Louis, MO. Search for other works by this author on: This Site PubMed Google Scholar Janet B. McGill, MD Janet B. McGill, MD 1 Professor of Medicine, Director, Fellowship in Endocrinology, Diabetes and Metabolism, Division of Endocrinology, Metabolism and Lipid Research, Washington University School of Medicine, St. Louis, MO. Search for other works by this author on: This Site PubMed Google Scholar Doi: https://doi.org/10.2337/9781580405713.47 Open the PDF Link PDF for Case 47: Stiff Person Syndrome in a Patient with Multiple Autoimmune Diseases in another window
  • Case 48: Glycogenic Hepatopathy in an Adolescent with Type 1 Diabetes By Nehama Zuckerman-Levin, MD ; Nehama Zuckerman-Levin, MD 1 Department of Pediatrics, Pediatric Diabetes and Obesity Clinic, Rambam Health Care Campus, Bruce Rappaport Faculty of Medicine, Technion, Haifa, Israel. Search for other works by this author on: This Site PubMed Google Scholar Oz Mordechai, MD ; Oz Mordechai, MD 1 Department of Pediatrics, Pediatric Diabetes and Obesity Clinic, Rambam Health Care Campus, Bruce Rappaport Faculty of Medicine, Technion, Haifa, Israel. Search for other works by this author on: This Site PubMed Google Scholar Naim Shehadeh, MD Naim Shehadeh, MD 1 Department of Pediatrics, Pediatric Diabetes and Obesity Clinic, Rambam Health Care Campus, Bruce Rappaport Faculty of Medicine, Technion, Haifa, Israel. Search for other works by this author on: This Site PubMed Google Scholar Doi: https://doi.org/10.2337/9781580405713.48 Open the PDF Link PDF for Case 48: Glycogenic Hepatopathy in an Adolescent with Type 1 Diabetes in another window
  • Case 49: Glycemic Control in a Child with Type 1 Diabetes and Autoimmune Hepatitis By Sandro Muntoni, MD, PhD ; Sandro Muntoni, MD, PhD 1 Department of Biomedical Sciences, University School of Cagliari and Centre for Metabolic Diseases and Atherosclerosis, The ME.DI.CO Association, Cagliari, Italy. Search for other works by this author on: This Site PubMed Google Scholar Mauro Congia, MD Mauro Congia, MD 2 Pediatric Gastroenterologic Unity, Microcitemic Hospital, ASL 8, Cagliari, Italy. Search for other works by this author on: This Site PubMed Google Scholar Doi: https://doi.org/10.2337/9781580405713.49 Open the PDF Link PDF for Case 49: Glycemic Control in a Child with Type 1 Diabetes and Autoimmune Hepatitis in another window
  • Case 50: Dizziness, Lightheadedness, and Syncope in a Patient with Type 2 Diabetes By Shalini Paturi, MD ; Shalini Paturi, MD 1 Endocrinology Division, Department of Medicine, Rosalind Franklin University of Medicine and Science/Chicago Medical School, North Chicago, IL. 2 Endocrinology Section, Department of Medicine, Captain James A. Lovell Federal Health Care Center, North Chicago, IL. Search for other works by this author on: This Site PubMed Google Scholar Janice L. Gilden, MS, MD, FCP, FACE Janice L. Gilden, MS, MD, FCP, FACE 1 Endocrinology Division, Department of Medicine, Rosalind Franklin University of Medicine and Science/Chicago Medical School, North Chicago, IL. 2 Endocrinology Section, Department of Medicine, Captain James A. Lovell Federal Health Care Center, North Chicago, IL. Search for other works by this author on: This Site PubMed Google Scholar Doi: https://doi.org/10.2337/9781580405713.50 Open the PDF Link PDF for Case 50: Dizziness, Lightheadedness, and Syncope in a Patient with Type 2 Diabetes in another window
  • Case 51: Growth Hormone Excess-Induced Diabetic Ketoacidosis By Andrew P. Demidowich, MD ; Andrew P. Demidowich, MD 1 Section on Pediatric Diabetes and Metabolism, Diabetes, Endocrinology, and Obesity Branch, NIDDK, National Institutes of Health, Bethesda, MD. Search for other works by this author on: This Site PubMed Google Scholar Maya Lodish, MD ; Maya Lodish, MD 2 Heritable Disorders Branch, NICHD, National Institutes of Health, Bethesda, MD. Search for other works by this author on: This Site PubMed Google Scholar Kristina I. Rother, MD, MHSc Kristina I. Rother, MD, MHSc 1 Section on Pediatric Diabetes and Metabolism, Diabetes, Endocrinology, and Obesity Branch, NIDDK, National Institutes of Health, Bethesda, MD. Search for other works by this author on: This Site PubMed Google Scholar Doi: https://doi.org/10.2337/9781580405713.51 Open the PDF Link PDF for Case 51: Growth Hormone Excess-Induced Diabetic Ketoacidosis in another window
  • Case 52: Refractory Angina in a Patient with Type 2 Diabetes By Mikhail Kosiborod, MD Mikhail Kosiborod, MD 1 Professor of Medicine, Saint Luke’s Mid America Heart Institute, University of Missouri-Kansas City, Kansas City, MO. Search for other works by this author on: This Site PubMed Google Scholar Doi: https://doi.org/10.2337/9781580405713.52 Open the PDF Link PDF for Case 52: Refractory Angina in a Patient with Type 2 Diabetes in another window
  • Case 53: Glycemic Control in a Patient with Type 2 Diabetes Undergoing Cardiac Surgery By M. Kathleen Figaro, MD, MS M. Kathleen Figaro, MD, MS 1 Medical Director, Diabetes Care Center, Endocrinology, Genesis Health Care, Bettendorf, IA. Search for other works by this author on: This Site PubMed Google Scholar Doi: https://doi.org/10.2337/9781580405713.53 Open the PDF Link PDF for Case 53: Glycemic Control in a Patient with Type 2 Diabetes Undergoing Cardiac Surgery in another window
  • Case 54: Inpatient Insulin Management for Complex Enteral Feedings By Anna Beth Barton, MD ; Anna Beth Barton, MD 1 Endocrinology Fellow, Department of Medicine Division of Endocrinology, Metabolism, and Nutrition, Duke University Medical Center, Durham, NC. Search for other works by this author on: This Site PubMed Google Scholar Kathryn J. Evans, DNP, FNP-BC ; Kathryn J. Evans, DNP, FNP-BC 2 Division of Endocrinology, Metabolism and Nutrition, Duke University Medical Center, Durham, NC. Search for other works by this author on: This Site PubMed Google Scholar Lillian F. Lien, MD Lillian F. Lien, MD 3 Medical Director, Duke Inpatient Diabetes Management Associate Professor, Department of Medicine Division of Endocrinology, Metabolism, and Nutrition, Duke University Medical Center, Durham, NC. Search for other works by this author on: This Site PubMed Google Scholar Doi: https://doi.org/10.2337/9781580405713.54 Open the PDF Link PDF for Case 54: Inpatient Insulin Management for Complex Enteral Feedings in another window
  • Case 55: Glycemic Control in a Patient with Type 1 Diabetes and Severe Burns By R. Matthew Hawkins, PA-C ; R. Matthew Hawkins, PA-C 1 Division of Endocrinology, Diabetes and Metabolism, Department of Medicine, University of Colorado Anschutz Medical Center, Aurora CO. Search for other works by this author on: This Site PubMed Google Scholar Boris Draznin, MD, PhD Boris Draznin, MD, PhD 1 Division of Endocrinology, Diabetes and Metabolism, Department of Medicine, University of Colorado Anschutz Medical Center, Aurora CO. Search for other works by this author on: This Site PubMed Google Scholar Doi: https://doi.org/10.2337/9781580405713.55 Open the PDF Link PDF for Case 55: Glycemic Control in a Patient with Type 1 Diabetes and Severe Burns in another window
  • Case 56: Combined Effect of Intravenous Insulin Infusion and Subcutaneous Rapid-Acting Insulin for Glycemic Control in Severe Insulin Resistance By Magdalena Szkudlinska, MD ; Magdalena Szkudlinska, MD 1 University of Washington School of Medicine, Division of Metabolism, Endocrinology, & Nutrition, Seattle, WA. Search for other works by this author on: This Site PubMed Google Scholar Irl B. Hirsch, MD Irl B. Hirsch, MD 1 University of Washington School of Medicine, Division of Metabolism, Endocrinology, & Nutrition, Seattle, WA. Search for other works by this author on: This Site PubMed Google Scholar Doi: https://doi.org/10.2337/9781580405713.56 Open the PDF Link PDF for Case 56: Combined Effect of Intravenous Insulin Infusion and Subcutaneous Rapid-Acting Insulin for Glycemic Control in Severe Insulin Resistance in another window
  • Case 57: Therapeutic Hypothermia and Severe Insulin Resistance in Patients with Diabetes and Cardiac Arrest By Stacey Seggelke, RD, MS, CDE ; Stacey Seggelke, RD, MS, CDE 1 Division of Endocrinology, Diabetes and Metabolism, Department of Medicine, University of Colorado Anschutz Medical Campus, Aurora, CO. Search for other works by this author on: This Site PubMed Google Scholar Boris Draznin, MD, PhD Boris Draznin, MD, PhD 1 Division of Endocrinology, Diabetes and Metabolism, Department of Medicine, University of Colorado Anschutz Medical Campus, Aurora, CO. Search for other works by this author on: This Site PubMed Google Scholar Doi: https://doi.org/10.2337/9781580405713.57 Open the PDF Link PDF for Case 57: Therapeutic Hypothermia and Severe Insulin Resistance in Patients with Diabetes and Cardiac Arrest in another window
  • Case 58: Extreme Insulin Resistance Following Heart Transplant By Suruchi Gupta, MBBS, MPH ; Suruchi Gupta, MBBS, MPH 1 Division of Endocrinology, Metabolism and Molecular Medicine, Northwestern University Feinberg School of Medicine, Chicago, IL. Search for other works by this author on: This Site PubMed Google Scholar Diana Johnson Oakes, APRN-BC ; Diana Johnson Oakes, APRN-BC 1 Division of Endocrinology, Metabolism and Molecular Medicine, Northwestern University Feinberg School of Medicine, Chicago, IL. Search for other works by this author on: This Site PubMed Google Scholar Ashley Therasse, MD ; Ashley Therasse, MD 1 Division of Endocrinology, Metabolism and Molecular Medicine, Northwestern University Feinberg School of Medicine, Chicago, IL. Search for other works by this author on: This Site PubMed Google Scholar Amisha Wallia, MD, MS ; Amisha Wallia, MD, MS 1 Division of Endocrinology, Metabolism and Molecular Medicine, Northwestern University Feinberg School of Medicine, Chicago, IL. Search for other works by this author on: This Site PubMed Google Scholar Mark E. Molitch, MD Mark E. Molitch, MD 1 Division of Endocrinology, Metabolism and Molecular Medicine, Northwestern University Feinberg School of Medicine, Chicago, IL. Search for other works by this author on: This Site PubMed Google Scholar Doi: https://doi.org/10.2337/9781580405713.58 Open the PDF Link PDF for Case 58: Extreme Insulin Resistance Following Heart Transplant in another window
  • Case 59: Glycemic Control after Left Ventricular Assist Device Placement in a Patient with Type 2 Diabetes By Gitana Staskus, MD Gitana Staskus, MD 1 Assistant Professor of Medicine, Division of Endocrinology, Metabolism and Diabetes, University of Utah, School of Medicine. Search for other works by this author on: This Site PubMed Google Scholar Doi: https://doi.org/10.2337/9781580405713.59 Open the PDF Link PDF for Case 59: Glycemic Control after Left Ventricular Assist Device Placement in a Patient with Type 2 Diabetes in another window
  • Case 60: Management of Diabetic Ketoacidosis in a Patient on Hemodialysis By Roopashree Prabhushankar, MD ; Roopashree Prabhushankar, MD 1 Division of Endocrinology, Diabetes and Metabolism, Department of Medicine, University of Missouri, Harry S. Truman Memorial Veterans’ Hospital, Columbia, MO. Search for other works by this author on: This Site PubMed Google Scholar Sofia Syed, MD ; Sofia Syed, MD 1 Division of Endocrinology, Diabetes and Metabolism, Department of Medicine, University of Missouri, Harry S. Truman Memorial Veterans’ Hospital, Columbia, MO. Search for other works by this author on: This Site PubMed Google Scholar James R. Sowers, MD, FACE, FACP, FAHA James R. Sowers, MD, FACE, FACP, FAHA 1 Division of Endocrinology, Diabetes and Metabolism, Department of Medicine, University of Missouri, Harry S. Truman Memorial Veterans’ Hospital, Columbia, MO. 2 Department of Medical Pharmacology and Physiology, University of Missouri, Harry S. Truman Memorial Veterans’ Hospital, Columbia, MO. 3 Diabetes and Cardiovascular Center, University of Missouri, Harry S. Truman Memorial Veterans’ Hospital, Columbia, MO. Search for other works by this author on: This Site PubMed Google Scholar Doi: https://doi.org/10.2337/9781580405713.60 Open the PDF Link PDF for Case 60: Management of Diabetic Ketoacidosis in a Patient on Hemodialysis in another window
  • Case 61: New Diabetes Emergency: Acute Rhabdomyolysis Complicating Hyperglycemic Hyperosmolar Coma By Cherie Vaz, MD ; Cherie Vaz, MD 1 Section of Endocrinology, Department of Medicine, Temple University School of Medicine, Philadelphia, PA. Search for other works by this author on: This Site PubMed Google Scholar Ajay Chaudhuri, MD, MRCP Ajay Chaudhuri, MD, MRCP 2 Department of Endocrinology, State University of New York, University at Buffalo and Kaleida Health, Buffalo, NY. Search for other works by this author on: This Site PubMed Google Scholar Doi: https://doi.org/10.2337/9781580405713.61 Open the PDF Link PDF for Case 61: New Diabetes Emergency: Acute Rhabdomyolysis Complicating Hyperglycemic Hyperosmolar Coma in another window
  • Case 62: Transitioning from Intravenous to Subcutaneous Insulin in a Complicated Patient By Kathryn J. Evans, DNP, FNP-BC ; Kathryn J. Evans, DNP, FNP-BC 1 Division of Endocrinology, Metabolism and Nutrition, Duke University Medical Center, Durham, NC. Search for other works by this author on: This Site PubMed Google Scholar Lillian F. Lien, MD Lillian F. Lien, MD 2 Duke Inpatient Diabetes Management, Department of Medicine, Division of Endocrinology, Metabolism, and Nutrition, Duke University Medical Center, Durham, NC. Search for other works by this author on: This Site PubMed Google Scholar Doi: https://doi.org/10.2337/9781580405713.62 Open the PDF Link PDF for Case 62: Transitioning from Intravenous to Subcutaneous Insulin in a Complicated Patient in another window
  • Case 63: Failure to Coordinate Diabetes Care between Hospital and Ambulatory Settings: A Threat to Safe and Quality Patient Care By Shawn Peavie, DO ; Shawn Peavie, DO 1 University of Cincinnati College of Medicine; Division of Endocrinology, Diabetes, and Metabolism, Cincinnati, OH. Search for other works by this author on: This Site PubMed Google Scholar Mercedes Falciglia, MD Mercedes Falciglia, MD 1 University of Cincinnati College of Medicine; Division of Endocrinology, Diabetes, and Metabolism, Cincinnati, OH. 2 Cincinnati Veterans Affairs Medical Center, Cincinnati, OH. Search for other works by this author on: This Site PubMed Google Scholar Doi: https://doi.org/10.2337/9781580405713.63 Open the PDF Link PDF for Case 63: Failure to Coordinate Diabetes Care between Hospital and Ambulatory Settings: A Threat to Safe and Quality Patient Care in another window
  • Case 64: Preventing Readmission: Translating the Hospital Diabetes Regimen into a Home Regimen that Is Safe, Effective, and Easy to Follow By Jane Jeffrie Seley, DNP, MPH, MSN, BC-ADM, CDE, CDTC Jane Jeffrie Seley, DNP, MPH, MSN, BC-ADM, CDE, CDTC 1 New York Presbyterian/Weill Cornell Medical Center, New York, NY. Search for other works by this author on: This Site PubMed Google Scholar Doi: https://doi.org/10.2337/9781580405713.64 Open the PDF Link PDF for Case 64: Preventing Readmission: Translating the Hospital Diabetes Regimen into a Home Regimen that Is Safe, Effective, and Easy to Follow in another window
  • Case 65: Novel Combination Therapy for Type 2 Diabetes By Donna White, RPh, CDE, BCACP ; Donna White, RPh, CDE, BCACP 1 University of Virginia Health System, Department of Pharmacy, Charlottesville, VA. Search for other works by this author on: This Site PubMed Google Scholar Svetlana Goldman, PharmD Svetlana Goldman, PharmD 1 University of Virginia Health System, Department of Pharmacy, Charlottesville, VA. Search for other works by this author on: This Site PubMed Google Scholar Doi: https://doi.org/10.2337/9781580405713.65 Open the PDF Link PDF for Case 65: Novel Combination Therapy for Type 2 Diabetes in another window
  • Case 66: Do Many People with Type 2 Diabetes Really Need Insulin? By Stanley S. Schwartz, MD Stanley S. Schwartz, MD 1 Main Line Health System, Wynnewood, PA. 2 University of Pennsylvania, Philadelphia, PA. Search for other works by this author on: This Site PubMed Google Scholar Doi: https://doi.org/10.2337/9781580405713.66 Open the PDF Link PDF for Case 66: Do Many People with Type 2 Diabetes <em>Really</em> Need Insulin? in another window
  • Case 67: Glycemic Control in a Patient with Type 1 Diabetes and Peritoneal Dialysis By Nadir Khir, MD ; Nadir Khir, MD 1 Division of Endocrinology, Diabetes and Metabolism, Department of Medicine, University of Missouri, Columbia, MO. Search for other works by this author on: This Site PubMed Google Scholar Stephen Brietzke, MD ; Stephen Brietzke, MD 1 Division of Endocrinology, Diabetes and Metabolism, Department of Medicine, University of Missouri, Columbia, MO. Search for other works by this author on: This Site PubMed Google Scholar James R. Sowers, MD James R. Sowers, MD 1 Division of Endocrinology, Diabetes and Metabolism, Department of Medicine, University of Missouri, Columbia, MO. Search for other works by this author on: This Site PubMed Google Scholar Doi: https://doi.org/10.2337/9781580405713.67 Open the PDF Link PDF for Case 67: Glycemic Control in a Patient with Type 1 Diabetes and Peritoneal Dialysis in another window
  • Case 68: Insulin Allergy in an Insulin-Requiring Patient By Nestoras Mathioudakis, MD Nestoras Mathioudakis, MD 1 Assistant Professor of Medicine, Division of Endocrinology, Diabetes, & Metabolism, Johns Hopkins University School of Medicine, Baltimore, MD. Search for other works by this author on: This Site PubMed Google Scholar Doi: https://doi.org/10.2337/9781580405713.68 Open the PDF Link PDF for Case 68: Insulin Allergy in an Insulin-Requiring Patient in another window
  • Case 69: Use of 3-Day Continuous Glucose Monitoring to Investigate Persistent Fasting Hyperglycemia in Type 2 Diabetes By Michelle Griffith, MD ; Michelle Griffith, MD 1 Division of Endocrinology and Metabolism, Department of Medicine, University of Pittsburgh School of Medicine, Pittsburgh, PA. Search for other works by this author on: This Site PubMed Google Scholar Mary Korytkowski, MD Mary Korytkowski, MD 1 Division of Endocrinology and Metabolism, Department of Medicine, University of Pittsburgh School of Medicine, Pittsburgh, PA. Search for other works by this author on: This Site PubMed Google Scholar Doi: https://doi.org/10.2337/9781580405713.69 Open the PDF Link PDF for Case 69: Use of 3-Day Continuous Glucose Monitoring to Investigate Persistent Fasting Hyperglycemia in Type 2 Diabetes in another window
  • Case 70: Insulin Injections: What You “See” May Not Be What You Get By Robert J. Rushakoff, MD ; Robert J. Rushakoff, MD 1 Division of Endocrinology and Metabolism, University of California, San Francisco, CA. Search for other works by this author on: This Site PubMed Google Scholar Mary M. Sullivan, DNP, RN, ANP-BC, CDE, FAAN ; Mary M. Sullivan, DNP, RN, ANP-BC, CDE, FAAN 2 Department of Nursing, University of California, San Francisco, CA. Search for other works by this author on: This Site PubMed Google Scholar Arti Shah, MD ; Arti Shah, MD 3 Division of Endocrinology and Metabolism, University of California, San Francisco, CA. Search for other works by this author on: This Site PubMed Google Scholar Heidemarie Windham MacMaster, PharmD, CDE Heidemarie Windham MacMaster, PharmD, CDE 4 Department of Pharmaceutical Services, University of California, San Francisco, CA. Search for other works by this author on: This Site PubMed Google Scholar Doi: https://doi.org/10.2337/9781580405713.70 Open the PDF Link PDF for Case 70: Insulin Injections: What You “See” May Not Be What You Get in another window
  • Case 71: Prolonged Insulin-Free Management of Type 1 Diabetes By Danielle Castillo, MD ; Danielle Castillo, MD 1 Division of Endocrinology & Metabolism, Eastern Virginia Medical School, Norfolk, VA. Search for other works by this author on: This Site PubMed Google Scholar Joseph Aloi, MD, FACE Joseph Aloi, MD, FACE 1 Division of Endocrinology & Metabolism, Eastern Virginia Medical School, Norfolk, VA. Search for other works by this author on: This Site PubMed Google Scholar Doi: https://doi.org/10.2337/9781580405713.71 Open the PDF Link PDF for Case 71: Prolonged Insulin-Free Management of Type 1 Diabetes in another window
  • Case 72: Delayed Response to NPH Insulin By Mayer B. Davidson, MD Mayer B. Davidson, MD 1 Department of Internal Medicine, Charles R. Drew University, Los Angeles, CA. Search for other works by this author on: This Site PubMed Google Scholar Doi: https://doi.org/10.2337/9781580405713.72 Open the PDF Link PDF for Case 72: Delayed Response to NPH Insulin in another window
  • Case 73: Reversal of Type 2 Diabetes by Weight Loss Despite Presence of Macro- and Microvascular Complications By Carl Peters, MB, ChB ; Carl Peters, MB, ChB 1 Newcastle Magnetic Resonance Centre, Campus for Ageing and Vitality, Newcastle upon Tyne, England. Search for other works by this author on: This Site PubMed Google Scholar Sarah Steven, MB, ChB ; Sarah Steven, MB, ChB 1 Newcastle Magnetic Resonance Centre, Campus for Ageing and Vitality, Newcastle upon Tyne, England. Search for other works by this author on: This Site PubMed Google Scholar Roy Taylor, MD Roy Taylor, MD 1 Newcastle Magnetic Resonance Centre, Campus for Ageing and Vitality, Newcastle upon Tyne, England. Search for other works by this author on: This Site PubMed Google Scholar Doi: https://doi.org/10.2337/9781580405713.73 Open the PDF Link PDF for Case 73: Reversal of Type 2 Diabetes by Weight Loss Despite Presence of Macro- and Microvascular Complications in another window
  • Case 74: Glycemic Control in Older Adults with Diabetes and Use of New SGLT2 Inhibitors By Carolyn Horney, MD ; Carolyn Horney, MD 1 University of Colorado Health Sciences Center, Division of Geriatric Medicine, Aurora, CO. Search for other works by this author on: This Site PubMed Google Scholar Jeffrey Wallace, MD, MPH Jeffrey Wallace, MD, MPH 1 University of Colorado Health Sciences Center, Division of Geriatric Medicine, Aurora, CO. Search for other works by this author on: This Site PubMed Google Scholar Doi: https://doi.org/10.2337/9781580405713.74 Open the PDF Link PDF for Case 74: Glycemic Control in Older Adults with Diabetes and Use of New SGLT2 Inhibitors in another window
  • Case 75: Blood Glucose Control of Patients with Hypertriglyceridemia By Henning Beck-Nielsen, DMSc Henning Beck-Nielsen, DMSc 1 Department of Endocrinology, Odense University Hospital, Odense, Denmark. Search for other works by this author on: This Site PubMed Google Scholar Doi: https://doi.org/10.2337/9781580405713.75 Open the PDF Link PDF for Case 75: Blood Glucose Control of Patients with Hypertriglyceridemia in another window
  • Case 76: No Effect of Gluten-Free Diet in Prevention of Autoimmune Type 1 Diabetes and Other Autoimmune Disorders in a Child with Celiac Disease By Sandro Muntoni, MD, PhD ; Sandro Muntoni, MD, PhD 1 Department of Biomedical Sciences, University School of Cagliari and Centre for Metabolic Diseases and Atherosclerosis, The ME.DI.CO. Association, Cagliari, Italy. Search for other works by this author on: This Site PubMed Google Scholar Mauro Congia, MD Mauro Congia, MD 2 Pediatric Gastroenterologic Unity, Microcitemic Hospital, Cagliari, Italy. Search for other works by this author on: This Site PubMed Google Scholar Doi: https://doi.org/10.2337/9781580405713.76 Open the PDF Link PDF for Case 76: No Effect of Gluten-Free Diet in Prevention of Autoimmune Type 1 Diabetes and Other Autoimmune Disorders in a Child with Celiac Disease in another window
  • Case 77: What Does It Take to Keep Glucose Normal? By Lawrence S. Phillips, MD Lawrence S. Phillips, MD 1 Atlanta VA Medical Center, Decatur, GA. 2 Division of Endocrinology and Metabolism, Department of Medicine, Emory University, School of Medicine, Atlanta, GA. Search for other works by this author on: This Site PubMed Google Scholar Doi: https://doi.org/10.2337/9781580405713.77 Open the PDF Link PDF for Case 77: What Does It Take to Keep Glucose Normal? in another window
  • Case 78: Psychosocial Stressors and Management in an Adolescent with Type 2 Diabetes By Radha Nandagopal, MD ; Radha Nandagopal, MD 1 Pediatric Endocrinology, Providence Medical Group, Spokane, WA. Search for other works by this author on: This Site PubMed Google Scholar Kristina I. Rother, MD, MHSc Kristina I. Rother, MD, MHSc 2 Section on Pediatric Diabetes and Metabolism, Diabetes, Endocrinology, and Obesity Branch, NIDDK, National Institutes of Health, Bethesda, MD. Search for other works by this author on: This Site PubMed Google Scholar Doi: https://doi.org/10.2337/9781580405713.78 Open the PDF Link PDF for Case 78: Psychosocial Stressors and Management in an Adolescent with Type 2 Diabetes in another window
  • Case 79: Suicide, Homicide, or Diabetes-Related Incident? By John N. Carter, BSc (Med), MBBS, FRACP, MD John N. Carter, BSc (Med), MBBS, FRACP, MD 1 Clinical Professor of Endocrinology, Sydney Medical School, University of Sydney, Australia. Search for other works by this author on: This Site PubMed Google Scholar Doi: https://doi.org/10.2337/9781580405713.79 Open the PDF Link PDF for Case 79: Suicide, Homicide, or Diabetes-Related Incident? in another window
  • Case 80: The Case of an Older Woman with Diabetes on Insulin Pump Therapy, Struggling with Cognitive Decline, Hypoglycemia, and Loss of Autonomy By Jennifer M. Hackel, DNP, GNP-BC, CDE ; Jennifer M. Hackel, DNP, GNP-BC, CDE 1 University of Massachusetts Boston, Boston, MA. Search for other works by this author on: This Site PubMed Google Scholar Liselle Douyon, MD ; Liselle Douyon, MD 2 University of Michigan, Endocrinology and Metabolism, Ann Arbor, MI. Search for other works by this author on: This Site PubMed Google Scholar Jeffrey B. Halter, MD Jeffrey B. Halter, MD 3 University of Michigan Geriatrics Center, Ann Arbor, MI. Search for other works by this author on: This Site PubMed Google Scholar Doi: https://doi.org/10.2337/9781580405713.80 Open the PDF Link PDF for Case 80: The Case of an Older Woman with Diabetes on Insulin Pump Therapy, Struggling with Cognitive Decline, Hypoglycemia, and Loss of Autonomy in another window
  • Case 81: Somnambulism (Sleepwalking) Caused by Nocturnal Hypoglycemia By David S.H. Bell, MB David S.H. Bell, MB 1 Clinical Professor of Medicine, University of Alabama, Birmingham, AL. Search for other works by this author on: This Site PubMed Google Scholar Doi: https://doi.org/10.2337/9781580405713.81 Open the PDF Link PDF for Case 81: Somnambulism (Sleepwalking) Caused by Nocturnal Hypoglycemia in another window
  • Case 82: Hypoglycemic Unawareness By Amita Maturu, MD ; Amita Maturu, MD 1 Division of Endocrinology, Diabetes and Metabolism, University of Colorado, Aurora, CO. Search for other works by this author on: This Site PubMed Google Scholar Neda Rasouli, MD Neda Rasouli, MD 1 Division of Endocrinology, Diabetes and Metabolism, University of Colorado, Aurora, CO. 2 Denver Veterans Affairs Medical Center, Denver, CO. Search for other works by this author on: This Site PubMed Google Scholar Doi: https://doi.org/10.2337/9781580405713.82 Open the PDF Link PDF for Case 82: Hypoglycemic Unawareness in another window
  • Case 83: Successful Use of Plasmapheresis in the Treatment of Hypoglycemia Due to Insulin Antibody Syndrome By Pankaj Sharda, MD ; Pankaj Sharda, MD 1 Section of Endocrinology, Diabetes and Metabolism, Temple University School of Medicine, Philadelphia, PA. Search for other works by this author on: This Site PubMed Google Scholar Thottathil Gopan, MD ; Thottathil Gopan, MD 2 Department of Endocrinology, Diabetes and Metabolism, Cleveland Clinic, Cleveland, OH. Search for other works by this author on: This Site PubMed Google Scholar Robert Zimmerman, MD ; Robert Zimmerman, MD 2 Department of Endocrinology, Diabetes and Metabolism, Cleveland Clinic, Cleveland, OH. Search for other works by this author on: This Site PubMed Google Scholar Elias S. Siraj, MD Elias S. Siraj, MD 1 Section of Endocrinology, Diabetes and Metabolism, Temple University School of Medicine, Philadelphia, PA. Search for other works by this author on: This Site PubMed Google Scholar Doi: https://doi.org/10.2337/9781580405713.83 Open the PDF Link PDF for Case 83: Successful Use of Plasmapheresis in the Treatment of Hypoglycemia Due to Insulin Antibody Syndrome in another window
  • Case 84: Postprandial Hypoglycemia, an Uncommon Presentation of Type 2 Diabetes By Muhammad W. Salam, MD ; Muhammad W. Salam, MD 1 Division of Endocrinology, Diabetes and Metabolism, Department of Medicine, University of Missouri, Columbia, MO. Search for other works by this author on: This Site PubMed Google Scholar James R. Sowers, MD James R. Sowers, MD 1 Division of Endocrinology, Diabetes and Metabolism, Department of Medicine, University of Missouri, Columbia, MO. 2 Department of Medical Pharmacology and Physiology, University of Missouri, Columbia, MO. 3 Diabetes and Cardiovascular Center, University of Missouri, Columbia, MO. 4 University of Missouri, Harry S. Truman Memorial Veterans’ Hospital, Columbia, MO. Search for other works by this author on: This Site PubMed Google Scholar Doi: https://doi.org/10.2337/9781580405713.84 Open the PDF Link PDF for Case 84: Postprandial Hypoglycemia, an Uncommon Presentation of Type 2 Diabetes in another window
  • Case 85: Factitious Hypoglycemia in a Type 2 Diabetic Patient By Stacey A. Seggelke, MS, RN, CNS, CDE, BC-ADM Stacey A. Seggelke, MS, RN, CNS, CDE, BC-ADM 1 Adult Diabetes Program, University of Colorado Denver, School of Medicine, Denver, CO. Search for other works by this author on: This Site PubMed Google Scholar Doi: https://doi.org/10.2337/9781580405713.85 Open the PDF Link PDF for Case 85: Factitious Hypoglycemia in a Type 2 Diabetic Patient in another window
  • Case 86: Recurrent Hypoglycemia in a Patient with Type 2 Diabetes By Pavani Srimatkandada, MD ; Pavani Srimatkandada, MD 1 Boston University School of Medicine, Section of Endocrinology, Diabetes, Nutrition and Weight Management, Boston, MA. Search for other works by this author on: This Site PubMed Google Scholar Marie E. McDonnell, MD ; Marie E. McDonnell, MD 1 Boston University School of Medicine, Section of Endocrinology, Diabetes, Nutrition and Weight Management, Boston, MA. Search for other works by this author on: This Site PubMed Google Scholar Sonia Ananthakrishnan, MD Sonia Ananthakrishnan, MD 1 Boston University School of Medicine, Section of Endocrinology, Diabetes, Nutrition and Weight Management, Boston, MA. Search for other works by this author on: This Site PubMed Google Scholar Doi: https://doi.org/10.2337/9781580405713.86 Open the PDF Link PDF for Case 86: Recurrent Hypoglycemia in a Patient with Type 2 Diabetes in another window
  • Case 87: Munchausen Syndrome: Hypoglycemia in an Obese Woman with Type 2 Diabetes By R. Paul Robertson, MD R. Paul Robertson, MD 1 Professor of Medicine, University of Minnesota, Professor of Medicine and Pharmacology, University of Washington, President Emeritus and Principal Investigator, Pacific Northwest Diabetes Research Institute. Search for other works by this author on: This Site PubMed Google Scholar Doi: https://doi.org/10.2337/9781580405713.87 Open the PDF Link PDF for Case 87: Munchausen Syndrome: Hypoglycemia in an Obese Woman with Type 2 Diabetes in another window
  • Case 88: The Use of Medical Technologies for the Reduction of Hypoglycemia in Type 1 Diabetes: Technology for Hypoglycemia Reduction By Viral N. Shah, MD ; Viral N. Shah, MD 1 Barbara Davis Center for Diabetes, University of Colorado Denver, Aurora, CO. Search for other works by this author on: This Site PubMed Google Scholar Aaron W. Michels, MD ; Aaron W. Michels, MD 1 Barbara Davis Center for Diabetes, University of Colorado Denver, Aurora, CO. 2 Departments of Internal Medicine and Pediatrics, School of Medicine, University of Colorado, Denver, Aurora, CO. Search for other works by this author on: This Site PubMed Google Scholar Satish K. Garg, MD Satish K. Garg, MD 1 Barbara Davis Center for Diabetes, University of Colorado Denver, Aurora, CO. 2 Departments of Internal Medicine and Pediatrics, School of Medicine, University of Colorado, Denver, Aurora, CO. 3 Editor-in-Chief, Diabetes Technology and Therapeutics , Aurora, CO. Search for other works by this author on: This Site PubMed Google Scholar Doi: https://doi.org/10.2337/9781580405713.88 Open the PDF Link PDF for Case 88: The Use of Medical Technologies for the Reduction of Hypoglycemia in Type 1 Diabetes: Technology for Hypoglycemia Reduction in another window
  • Case 89: Reversal of Insulin-Requiring Type 2 Diabetes and Development of Hypoglycemia in a Morbidly Obese Patient By David S.H. Bell, MB David S.H. Bell, MB 1 University of Alabama, Birmingham, AL. Search for other works by this author on: This Site PubMed Google Scholar Doi: https://doi.org/10.2337/9781580405713.89 Open the PDF Link PDF for Case 89: Reversal of Insulin-Requiring Type 2 Diabetes and Development of Hypoglycemia in a Morbidly Obese Patient in another window
  • Case 90: Munchausen-by-Proxy: Hypoglycemia in an Islet Autotransplantation Recipient By R. Paul Robertson, MD R. Paul Robertson, MD 1 Professor of Medicine, University of Minnesota; Professor of Medicine and Pharmacology, University of Washington; President Emeritus and Principal Investigator, Pacific Northwest Diabetes Research Institute. Search for other works by this author on: This Site PubMed Google Scholar Doi: https://doi.org/10.2337/9781580405713.90 Open the PDF Link PDF for Case 90: Munchausen-by-Proxy: Hypoglycemia in an Islet Autotransplantation Recipient in another window
  • Case 91: Treatment of a Patient with Diabetes and Severe Hypoglycemia By Henning Beck-Nielsen, DMSc Henning Beck-Nielsen, DMSc 1 Department of Endocrinology, Department of Endocrinology, Odense University Hospital, Odense, Denmark. Search for other works by this author on: This Site PubMed Google Scholar Doi: https://doi.org/10.2337/9781580405713.91 Open the PDF Link PDF for Case 91: Treatment of a Patient with Diabetes and Severe Hypoglycemia in another window
  • Case 92: Hypoglycemia with Use of Glargine Insulin in the Management of Type 2 Diabetes, Occurring with Titration Aimed at Achieving Prebreakfast Glucose Levels <100 mg/dL (5.6 mmol/L) By Saira Adeel, MD ; Saira Adeel, MD 1 Atlanta VA Medical Center, Division of Endocrinology, Emory University School of Medicine, Atlanta, GA. Search for other works by this author on: This Site PubMed Google Scholar Lawrence S. Phillips, MD Lawrence S. Phillips, MD 2 Division of Endocrinology and Metabolism, Emory University, Atlanta, GA. Search for other works by this author on: This Site PubMed Google Scholar Doi: https://doi.org/10.2337/9781580405713.92 Open the PDF Link PDF for Case 92: Hypoglycemia with Use of Glargine Insulin in the Management of Type 2 Diabetes, Occurring with Titration Aimed at Achieving Prebreakfast Glucose Levels &lt;100 mg/dL (5.6 mmol/L) in another window
  • Case 93: Progressive Hypoglycemia Due to Insulinoma in a Patient with Type 2 Diabetes: Treatment with Image-Guided Minimally Invasive Pancreas-Sparing Surgery By Mary-Elizabeth Patti, MD ; Mary-Elizabeth Patti, MD 1 Joslin Diabetes Center, Boston, MA. 5 Harvard Medical School, Boston, MA. Search for other works by this author on: This Site PubMed Google Scholar Mark P. Callery, MD, FACS ; Mark P. Callery, MD, FACS 2 Institute for HepatoBiliary and Pancreatic Surgery, Beth Israel Deaconess Medical Center, Boston, MA. 5 Harvard Medical School, Boston, MA. Search for other works by this author on: This Site PubMed Google Scholar Robert M. Najarian, MD ; Robert M. Najarian, MD 3 Department of Pathology, Beth Israel Deaconess Medical Center, Boston, MA. 5 Harvard Medical School, Boston, MA. Search for other works by this author on: This Site PubMed Google Scholar Mandeep S. Sawhney, MD, MS ; Mandeep S. Sawhney, MD, MS 4 Department of Medicine, Beth Israel Deaconess Medical Center and Dana Farber Harvard Cancer Center, Boston, MA. 5 Harvard Medical School, Boston, MA. Search for other works by this author on: This Site PubMed Google Scholar Lyle Mitzner, MD ; Lyle Mitzner, MD 1 Joslin Diabetes Center, Boston, MA. 5 Harvard Medical School, Boston, MA. Search for other works by this author on: This Site PubMed Google Scholar Allison B. Goldfine, MD ; Allison B. Goldfine, MD 1 Joslin Diabetes Center, Boston, MA. 5 Harvard Medical School, Boston, MA. Search for other works by this author on: This Site PubMed Google Scholar A. James Moser, MD, FACS A. James Moser, MD, FACS 2 Institute for HepatoBiliary and Pancreatic Surgery, Beth Israel Deaconess Medical Center, Boston, MA. 5 Harvard Medical School, Boston, MA. Search for other works by this author on: This Site PubMed Google Scholar Doi: https://doi.org/10.2337/9781580405713.93 Open the PDF Link PDF for Case 93: Progressive Hypoglycemia Due to Insulinoma in a Patient with Type 2 Diabetes: Treatment with Image-Guided Minimally Invasive Pancreas-Sparing Surgery in another window
  • Case 94: Managing Pain and Paralysis in Chronic Inflammatory Demyelinating Polyneuropathy in Diabetes By Aaron I. Vinik, MD, PhD, FCP, MACP, FACE Aaron I. Vinik, MD, PhD, FCP, MACP, FACE 1 Eastern Virginia Medical School, Strelitz Diabetes Center, Norfolk, VA. Search for other works by this author on: This Site PubMed Google Scholar Doi: https://doi.org/10.2337/9781580405713.94 Open the PDF Link PDF for Case 94: Managing Pain and Paralysis in Chronic Inflammatory Demyelinating Polyneuropathy in Diabetes in another window
  • Case 95: Neuropathy in Metformin-Treated Type 2 Diabetes By Aaron I. Vinik, MD, PhD, FCP, MACP, FACE Aaron I. Vinik, MD, PhD, FCP, MACP, FACE 1 Eastern Virginia Medical School, Strelitz Diabetes Center, Norfolk, VA. Search for other works by this author on: This Site PubMed Google Scholar Doi: https://doi.org/10.2337/9781580405713.95 Open the PDF Link PDF for Case 95: Neuropathy in Metformin-Treated Type 2 Diabetes in another window
  • Case 96: A Case of Acute Sensory Neuropathy in Type 1 Diabetes By Andrew J.M. Boulton, MD, DSc, FACP, FRCP Andrew J.M. Boulton, MD, DSc, FACP, FRCP 1 University of Manchester, UK; University of Miami, Miami, FL; Manchester Royal Infirmary, Manchester, UK; President, European Association for the Study of Diabetes, Dusseldorf, Germany. Search for other works by this author on: This Site PubMed Google Scholar Doi: https://doi.org/10.2337/9781580405713.96 Open the PDF Link PDF for Case 96: A Case of Acute Sensory Neuropathy in Type 1 Diabetes in another window
  • Case 97: Nondiabetic Neuropathy in a Patient with Type 2 Diabetes By David S.H. Bell, MB David S.H. Bell, MB 1 University of Alabama, Birmingham, AL. Search for other works by this author on: This Site PubMed Google Scholar Doi: https://doi.org/10.2337/9781580405713.97 Open the PDF Link PDF for Case 97: Nondiabetic Neuropathy in a Patient with Type 2 Diabetes in another window
  • Case 98: Severe Distal Symmetrical and Autonomic Neuropathy in a Patient with a Short Duration of Type 1 Diabetes By David S.H. Bell, MB David S.H. Bell, MB 1 University of Alabama, Birmingham, AL. Search for other works by this author on: This Site PubMed Google Scholar Doi: https://doi.org/10.2337/9781580405713.98 Open the PDF Link PDF for Case 98: Severe Distal Symmetrical and Autonomic Neuropathy in a Patient with a Short Duration of Type 1 Diabetes in another window
  • Case 99: Diabetic Amyotrophy and Neuropathic Cachexia By David S.H. Bell, MB David S.H. Bell, MB 1 University of Alabama, Birmingham, AL. Search for other works by this author on: This Site PubMed Google Scholar Doi: https://doi.org/10.2337/9781580405713.99 Open the PDF Link PDF for Case 99: Diabetic Amyotrophy and Neuropathic Cachexia in another window
  • Case 100: High GAD Antibody Levels and Cerebellar Atrophy in a Patient with Type 1 Diabetes By David S.H. Bell, MB David S.H. Bell, MB 1 University of Alabama, Birmingham, AL. Search for other works by this author on: This Site PubMed Google Scholar Doi: https://doi.org/10.2337/9781580405713.100 Open the PDF Link PDF for Case 100: High GAD Antibody Levels and Cerebellar Atrophy in a Patient with Type 1 Diabetes in another window
  • Case 101: Resolution of Infertility with Diabetes Therapy By David S.H. Bell, MB David S.H. Bell, MB 1 University of Alabama, Birmingham, AL. Search for other works by this author on: This Site PubMed Google Scholar Doi: https://doi.org/10.2337/9781580405713.101 Open the PDF Link PDF for Case 101: Resolution of Infertility with Diabetes Therapy in another window
  • Case 102: The Initial Pregnancy Visit of a Woman with Type 1 Diabetes and Diabetes Complications By Aidan McElduff, MD Aidan McElduff, MD 1 Discipline of Medicine, Sydney University, Sydney, NSW, Australia. Search for other works by this author on: This Site PubMed Google Scholar Doi: https://doi.org/10.2337/9781580405713.102 Open the PDF Link PDF for Case 102: The Initial Pregnancy Visit of a Woman with Type 1 Diabetes and Diabetes Complications in another window
  • Case 103: Gastroparesis and Pregnancy By Carl Peters, MB, ChB ; Carl Peters, MB, ChB 1 Newcastle Magnetic Resonance Centre, Campus for Ageing and Vitality, Newcastle upon Tyne, England. Search for other works by this author on: This Site PubMed Google Scholar Roy Taylor, MD Roy Taylor, MD 1 Newcastle Magnetic Resonance Centre, Campus for Ageing and Vitality, Newcastle upon Tyne, England. Search for other works by this author on: This Site PubMed Google Scholar Doi: https://doi.org/10.2337/9781580405713.103 Open the PDF Link PDF for Case 103: Gastroparesis and Pregnancy in another window
  • Index Open the PDF Link PDF for Index in another window
  • Diabetes Care
  • Clinical Diabetes
  • Diabetes Spectrum
  • Standards of Medical Care in Diabetes
  • Scientific Sessions Abstracts
  • BMJ Open Diabetes Research & Care
  • ShopDiabetes.org
  • ADA Professional Books

Clinical Compendia

  • Clinical Compendia Home
  • Latest News
  • DiabetesPro SmartBrief
  • Special Collections
  • DiabetesPro®
  • Diabetes Food Hub™
  • Insulin Affordability
  • Know Diabetes By Heart™
  • About the ADA
  • Journal Policies
  • For Reviewers
  • Advertising in ADA Journals
  • Reprints and Permission for Reuse
  • Copyright Notice/Public Access Policy
  • ADA Professional Membership
  • ADA Member Directory
  • Diabetes.org
  • X (Twitter)
  • Cookie Policy
  • Accessibility
  • Terms & Conditions
  • Get Adobe Acrobat Reader
  • © Copyright American Diabetes Association

This Feature Is Available To Subscribers Only

Sign In or Create an Account

LITFL-Life-in-the-FastLane-760-180

Top 100 Ultrasound

Litfl 100+ ultrasound quiz.

Clinical cases and self assessment problems from the Ultrasound library  to enhance interpretation skills through Ultrasound problems. Preparation for examinations.

Each case presents a clinical scenario; a series of questions; clinical images and finally some pearls to highlight the key learning points. We trust that you will find a few clinical pearls or reminders that you could apply to your patients that you care for in your emergency department or other health setting

Search by keywords; disease process; condition; eponym or clinical features…

LITFL Top 100 Self Assessment Quizzes

ULTRASOUND LIBRARY

POCUS, eFAST and basic principles

Privacy Overview

We have a new app!

Take the Access library with you wherever you go—easy access to books, videos, images, podcasts, personalized features, and more.

Download the Access App here: iOS and Android . Learn more here!

  • Remote Access
  • Save figures into PowerPoint
  • Download tables as PDFs

Pharmacology in Rehabilitation, 5e

Appendix C:  Answers to Case Study Questions

  • Download Chapter PDF

Disclaimer: These citations have been automatically generated based on the information we have and it may not be 100% accurate. Please consult the latest official manual style if you have any questions regarding the format accuracy.

Download citation file:

  • Search Book

Jump to a Section

Chapter 6 case study, chapter 7 case study.

  • CHAPTER 8 CASE STUDY
  • CHAPTER 9 CASE STUDY
  • CHAPTER 10 CASE STUDY
  • CHAPTER 11 CASE STUDY
  • CHAPTER 12 CASE STUDY
  • CHAPTER 13 CASE STUDY
  • CHAPTER 14 CASE STUDY
  • CHAPTER 15 CASE STUDY
  • CHAPTER 16 CASE STUDY
  • CHAPTER 17 CASE STUDY
  • CHAPTER 21 CASE STUDY
  • CHAPTER 22 CASE STUDY
  • CHAPTER 23 CASE STUDY
  • CHAPTER 24 CASE STUDY
  • CHAPTER 25 CASE STUDY
  • CHAPTER 26 CASE STUDY
  • CHAPTER 27 CASE STUDY
  • CHAPTER 29 CASE STUDY
  • CHAPTER 30 CASE STUDY
  • CHAPTER 31 CASE STUDY
  • CHAPTER 32 CASE STUDY
  • CHAPTER 33 CASE STUDY
  • CHAPTER 34 CASE STUDY
  • CHAPTER 35 CASE STUDY
  • CHAPTER 36 CASE STUDY
  • CHAPTER 37 CASE STUDY
  • CHAPTER 38 CASE STUDY
  • Full Chapter
  • Supplementary Content

Sedative-Hypnotic Drugs

What is the most likely reason for R.S.’s poor performance in the morning rehabilitation sessions?

The sedative-hypnotic (flurazepam) appeared to be producing a hangover-like effect, which limited the patient’s cognitive skills during the early daily activities. Flurazepam is a benzodiazepine drug with a half-life of 2.3 hours and a duration of action of 7 to 8 hours. This drug, however, is metabolized in the liver to active metabolites with half-lives ranging from 30 to 200 hours. It seems likely that R.S. was continuing to experience sedative effects from these active metabolites well into the next morning.

What would be the likely solution?

The therapists can deal with this problem initially by reserving the early morning session for stretching and ROM activities and then gradually moving into upper-body strengthening. Activities that require more patient learning and comprehension can be done later in the morning or in the afternoon. The hangover-like problem should also be brought to the attention of the physician. In this case, the physician switched the hypnotic drug to zolpidem (Ambien), 10 mg administered at bedtime. Zolpidem has a half-life of 2.5 hours and a duration of action of 6 to 8 hours, so R.S. should still get the benefit of a full night’s sleep. Because zolpidem is not metabolized to active metabolites, it is unlikely that it will continue to exert sedative-hypnotic effects into the next morning. Zolpidem also affects neuronal GABA receptors differently than benzodiazepines such as flurazepam. This difference might reduce the risk of problems, such as rebound insomnia, when it is time to discontinue the drug.

Antidepressant Drugs

How can the therapist reduce the risk of orthostatic hypotension during rehabilitation sessions?

To reduce orthostatic hypotension, the therapist decided to place the patient on the tilt table for the first day after imipramine was started and to monitor blood pressure regularly. The therapist had the patient perform weight shifting and upper-extremity facilitation activities while he was on the tilt table. The patient tolerated this well, so the therapist had him resume ambulation activities using the parallel bars on the following day. With the patient standing inside the bars, the therapist carefully watched for any subjective signs of dizziness or syncope in the patient (i.e., facial pallor, inability to follow instructions). Standing bouts were also limited in duration. By the third day, ambulation training continued with the patient outside the parallel bars, but the therapist made a point of having the patient’s wheelchair close at hand in case the patient began to appear faint. These precautions of careful observation and short, controlled bouts of ambulation were continued throughout the remainder of the patient’s hospital stay, and the therapist observed no incident of orthostatic hypotension during physical therapy.

Will clinicians notice an immediate improvement in J.G.’s mood after starting this?

Pop-up div Successfully Displayed

This div only appears when the trigger link is hovered over. Otherwise it is hidden from view.

Please Wait

Ohio State nav bar

The Ohio State University

  • BuckeyeLink
  • Find People
  • Search Ohio State

Review Questions

QUESTION ONE

When Mr. Johnson was first diagnosed with Type 2 Diabetes Mellitus what classic symptoms should he have been told he would exhibit?  Select all that apply.

a. Visual changes, recurrent infections, and pruritis

b. Nausea, hypotension, and mental confusion

c. Polyuria, polyphagia, and polydipsia

d. Sweet fruity breath, Kussmaul breathing, and vomiting

QUESTION TWO

What risk factors increase the chances of developing Type II diabetes?

a. Smoking, race, diet, family history, and height

b. Family history, hygiene, smoking, increased age, and hypertension

c. Hygiene, lifestyle, genetics, smoking, and obesity

d. Family history, increased age, obesity, hypertension, and smoking

QUESTION THREE

What does insulin resistance mean?

a. The pancreas is underactive and can keep up with the production of insulin needed to overcome the high amount of glucose in the blood.

b. Glucose is raised above normal levels.

c. The inability for cells to absorb and use blood sugar for energy.

d. The pancreas is overactive and cannot keep up with the insulin demands due to an abundance of glucose in the blood.

QUESTION FOUR

What evidence-based suggestions can you provide your patients to prevent or manage Type II diabetes?

a. Eating whatever you desire as long as you work out.

b. Eating a healthy diet and exercising.

c. Eating a healthy diet only.

d. Staying inside all day under the blankets.

QUESTION FIVE

What are the long term effects of untreated Type 2 Diabetes Mellitus? [Select all that apply]

a. Blindness

b. Kidney Disease

c. Tinnitus

d. Peripheral Neuropathy

Answer:  A & C

Visual changes, recurrent infections, and pruritis are all complications of Type 2 Diabetes Mellitus. Although polyuria, polyphagia, and polydipsia are known as the classic symptoms for Type 1 Diabetes Mellitus, they are also present in Type 2 Diabetes Mellitus. Nausea, hypotension and mental confusion are signs of hypoglycemia.  Sweet fruity breath, Kussmaul respirations, and vomiting are signs of diabetic ketoacidosis.

Reference: McCance, K. L., Huether, S. E., Brashers, V. L., & Rote, N. S. (2019). Pathophysiology: the biologic basis for disease in adults and children  (8th ed.). St. Louis, MO: Elsevier.

Answer: D, risk factors for type II diabetes include obesity, diet, lack of exercise, race, increased age, and family history.

Maintaining a healthy weight and engaging in physical activity helps to control weight, uses glucose for energy and allows cells to be more sensitive to insulin. Having a family history increases the risk of type II diabetes.  For unclear reasons, African Americans, Hispanics, American Indians, and Asian Americans have an increased risk of developing type II diabetes. An increase in age is also a risk factor due to weight gain and the tendency to be less active.  A diet high in red meats, processed carbohydrates, sugar, and saturated and trans fat increases the risk of type II diabetes. Options A, B, and C are incorrect. Height and hygiene are not contributing factors.

Type 2 diabetes. (2019, January 9). Retrieved from https://www.mayoclinic.org/diseases-conditions/type-2-diabetes/symptoms-causes/syc-20351193 .

Answer: C. Insulin resistance is the inability for cells to absorb and use blood sugar for energy due to cells being desensitized to insulin.

Cells that are desensitized to insulin do not take up insulin thus not taking up glucose to use for energy. Option A is incorrect, the pancreas becomes overactive when there is a high level of glucose in the blood but does not define insulin resistance. Option B is the lab value result of a patient with diabetes. Option D is what happens with type II diabetes but does not define insulin resistance.

Felman, A. (2019, March 26). Insulin resistance: Causes, symptoms, and prevention. Retrieved from https://www.medicalnewstoday.com/articles/305567.php.

Answer: B. Eating a healthy diet and exercising to maintain a healthy weight or lose weight.

Eating a healthy diet, high in fruits and vegetables and low in carbs helps the pancreas not get overworked creating insulin thus keeping blood glucose levels in a normal range. Exercising allows for hypertrophy of the muscles which respond better to insulin after exercise. Therefore, an active person can help prevent or reverse insulin resistance. Option A does not help a patient manage their diabetes if the same high carb, high sugar foods are being consumed. Option C should be complimented with exercise to help the muscles respond better to insulin. Option D will only worsen diabetes due to a lack of exercise.

The Diabetes Diet. (2019, June 19). Retrieved from https://www.helpguide.org/articles/diets/the-diabetes-diet.htm.

Answers: A, B, C & D

This patient already wears glasses, so he is at risk for blindness due to the risk of glaucoma, cataracts, and diabetic retinopathy from persistent hyperglycemia. Kidney disease can occur from untreated diabetes due to persistent hypertension and strain on the kidneys. Diabetes is the leading cause of kidney failure in the U.S. Tinnitus is ringing in the ears caused by inadequate blood flow to vessels in the ear due to hyperglycemia. Peripheral neuropathy occurs from nerve damage as well from high levels of glucose in the bloodstream. This increases the risk of infection and amputation of feet.

Felson, S. (Ed.). (2019, May 6). Diabetes Complications: How Uncontrolled Diabetes Affects Your Body. Retrieved from https://www.webmd.com/diabetes/guide/risks-complications-uncontrolled-diabetes#1.Holcát,

M. (2007, May). Tinnitus and diabetes. Retrieved from https://www.ncbi.nlm.nih.gov/pubmed/17642439.

McCance, K. L., Huether, S. E., Brashers, V. L., & Rote, N. S. (2018).  Pathophysiology: the biologic basis for disease in adults and children . St. Louis, MO: Elsevier.

PharmD

Clinical Case Studies with Answers

This section includes clinical case studies* with answers for the theoretical practice of Doctor of Pharmacy ( Pharm.D ) students. Also, it is equally useful for Pharm.D PB students. These Pharm.D case studies are regularly discussed in the telegram group .   You can join there and participate in active case discussions. Case studies help you revise your Pharmacotherapy syllabus and increase knowledge of disease in an innovative way apart from practical knowledge. 

You can access these clinical case studies with answers with no charge, which are contributed by various seniors from Doctor of Pharmacy stream only. In some cases practical references are also taken from certified & specialized practitioner from the respective field along with standard treatment guidelines.

*Clinical Case Studies included here are collected from various sources and are only for study purpose while maintaining the privacy of patients.

1. Pharm.D Case Studies for second year and Pharm.D PB first year 

A) cardiovascular system case studies with answers: .

  • Hypertension    :   Case Study 1      and Case Study 2  
  • Congestive Heart Failure   :  Case Study 3 and Case Study 4
  • Angina Pectoris  :   Case study 5 and Case Study 6
  • Myocardial Infarction :  Case Study 7 and Case Study 8
  • Hyperlipidaemias :   Case Study 9 and Case Study 10  
  • Electrophysiology of heart and Arrhythmia :   Case Study 11 and Case Study 12

b) Respiratory System Case Studies with answers:

  • Asthma:   Case Study 13 and Case Study 14
  • COPD : Case Study 15 and Case Study 16

c) Endocrine System Case Studies with answers:

  • Diabetes:   Case Study 17    and  Case Study – 18 
  • Hypothyroidism :  Case Study -19 and Case Study -20
  • Other Thyroid Disorders :  Case Study -21    and Case Study -22
  • Oral Contraceptive Use : Case Study- 23 and Case Study- 24
  • Hormone Replacement Therapy:   Case Study- 25    and  Case Study- 26
  • Osteoporosis: Case Study – 27 and Case Study – 28

d) Opthalmology Case Studies with answers: 

  • Glaucoma: Case Study – 29    and Case Study – 30 
  • Conjunctivitis: Case Study – 3 1 and Case Study – 32

e) General prescribing guidelines, Small Case Study with answers:

  • Pediatrics : Case Study – 33

Case Studies for Doctor of Pharmacy students :

Case study-1 (hypertension with cardiovascular comorbidities).

JR is a 58-year-old man with a medical history of elevated low-density lipoprotein levels and well-controlled chronic stable angina (experiencing <1 angina attack per month) secondary to coronary artery disease (CAD). He presents to his primary care physician for a follow-up appointment after his blood pressure (BP) was found to be 165/94 mm Hg at his annual physical exam. At today’s visit, JR’s BP is found to be 166/93 mm Hg, resulting in a diagnosis of hypertension. JR is currently on  Atorvastatin 40 mg daily  and  Metoprolol Tartrate 100 mg twice daily.  And he reports no adverse effects from either medication. He has no other medical history of note, and his resting heart rate is 65 to 70 beats per minute.

Questions :

  • What is the target goal for BP in this patient?
  • What are the main classes of anti-hypertensives that can be used in this case ?
  • Prepare a therapeutic regimen for this patient.

For Case Study-1 Answer, Click here.

Case Study-2 (Hypertension with Type-2 Diabetes Mellitus)

Mr. MK a 55-year-old man, having history of  hypertension and type-2 diabetes mellitus  for past 10 years with non-compliance to medication and poor diet control referred to the clinic for further management of poorly controlled diabetes and hypertension.

Socio-demographics:

Age :55                                   Sex : Male BMI : 35kg/m²                     Weight : 98 kg Occupation : Salesman

Family History :

Mother : Diabetic ( on dialysis). Father : Stroke ( residual left hemiparesis).

Subjective and Objective Evidence :

  • Minimal bilateral leg edema.
  • Bilateral proliferative retinopathy. Vitals –         Blood Pressure : 160/90 mm/Hg                      Pulse rate. : 88 /min

Investigation results :

A1c                                       :      9.2% FBS.                                     :      11.8 mmol/L Serum creatinine               :      1.2 mg/dl eGFR                                    :      88 ml/min/1.73m² 24 hr urinary protein         :      200mg/24 hr. ECG                                      :       Left Ventricular Hypertrophy.

Past medication :

Metformin          1000 mg   BD Gliclazide           160 mg      BD Amlodipine        10 mg        OD

  • What might be reasons for his poorly controlled diabetes and hypertension?
  • What would be the A1c and BP target?
  • How would you manage both HTN and T2DM ?
  • Is there any Drug interaction, that should be taken in consider?
  • What are the major patient counselling points?

For Case Study-2 Answer,  Click here

For more Pharm.D Case studies. Click here.

Case Study-3 ( Congestive Cardiac Failure)

Mrs. JE a 70-year-old woman, was admitted to the medical unit with complaints of increasing dyspnea on exertion.

Subjective data:

  • Had a severe MI at 58 years of age
  • Has experienced increasing dyspnea on exertion during the last 2 years
  • Recently had a respiratory tract infection, frequently cough and edema in legs 2 weeks ago
  • Shortness of breathe while having average walking
  • Has to sleep with head elevated on 3 pillows
  • Does not always remember to take medication

Objective Evidence :

  • In respiratory distress, use of accessory muscles.
  • Heart murmur.
  • Moist cracle in both lungs.
  • Cyanotic lips and extremeties.
  • Skin cool and diaphoretic. Vitals –  Blood Pressure : 130/80 mm/Hg                Pulse rate. : 70 /min                Respiratory rate : 36 / min

Chest X-Ray : Cardiomegaly with right and left ventricular hypertrophy, fluid in lower lung fields.

Current Medication/ Collaborative Care

Digoxin                               0.25 mg            PO            qd Furosemide                        40 mg               IV              bid Potassium                         40 mEq             PO            bid Enalapril                             5 mg                 PO            qd Sodium diet                       2 gm Oxygen                               6 L/min

  • What is the significance of the findings of the chest X- Ray?
  • Are there any collaborative problems that has been not considered?
  • Is Mrs. JE provided with appropriate drug regimen? Justify.

For  Case Study-3 Answer, Click here .

For more Pharm.D Case studies. Click here.

Case Study-4 ( Case of Congestive Heart Failure in Pediatrics patient)

A 6 weeks old female presented to the emergency room with the chief complaints of lethargy, poor feeding, and respiratory distress. Her parents reported that she sweats a lot on her forehead when feeding. Her parents have also noted her to be increasingly lethargic, with tachypnea, and retractions.

  • She is the product of a G3P2, full term, uncomplicated pregnancy. Delivery was unremarkable except for meconium stained fluid. Her pediatric follow-up has been poor.
  • Developed a febrile illness with cough, rhinorrhea, and emesis prior 2 weeks
  • Subsequently developed progressive respiratory distress.

Objective Evidence:

  • Acyanotic, Lethargic, tachypneic, mildly cachectic
  • Mild to moderate subcoastal & intercoastal Retractions

                                    Heart rate                : 160 / min  

                                    Respiratory rate     : 72 / min   

                                    Temp.                       : 98.24 ᵒF

HEENT exam : unremarkable

  • Neck is supple without lymphadenopathy.
  • Skin is clear with no rashes or other significant skin lesions.
  • Lungs have scattered crackles with slightly decreased aeration in the left lower lobe.
  • Precordium is mildly active.
  • Heart is of regular rate and rhythm, with a Grade II/VI holosystolic murmur at the mid lower left sternal border with radiation to the cardiac apex.
  • S1 is normal and the S2 is prominent. An S4 gallop is noted at the cardiac apex. There are no rubs or valve clicks.
  • Her abdomen is soft, non-distended, and non-tender.
  • The liver edge is palpable 3 to 4 cm below the right costal margin. Bowel sounds are hypoactive.
  • Capillary refill is 4 to 5 seconds (delayed).
  • Chest x-ray:  Moderate cardiomegaly with a moderate degree of pulmonary edema.  No pleural effusions.
  • 12 lead ECG:    Sinus tachycardia, normal PR and QTc intervals, and a left axis deviation. Voltage evidence of biventricular hypertrophy is present. No significant Q-waves or ST segment changes are noted.

A large peri-membranous ventricular septal defect with non-restrictive left to right shunting. All cardiac chambers are dilated. Left ventricular contractility is at the lower range of normal. There is no pericardial effusion.

  • Suggest the best therapeutic regimen to control/overcome the current situation.
  • Should she be referred for surgical correction of Ventricular Septal Defect after the drug regimen has been started?

Go to  Case Study-4 Answers  |  Other  Case Studies

Case Study-5 ( Case of Angina Pectoris)

A 62-year-old male smoker with Type-2 Diabetes Mellitus and Hypertension presents with a 4-month history of exertional chest pain. 

Physical examination shows a  blood pressure  of 152/90 mm Hg, but is otherwise unremarkable. 

The  ECG  is normal, and laboratory tests show a  fasting blood glucose  value of 110 mg/dL,  glycosylated hemoglobin  6.0%,  creatinine  1.1 mg/dL,  total cholesterol  160,  LDL  120,  HDL  38, and  triglycerides  147 mg/dL. 

He exercises for 8 minutes, experiences chest pain, and is found to have a 2-mm  ST-segment depression  in the inferolateral leads at the end of exercise. 

The patient is diagnosed with  chronic stable angina .

  • What is the treatment goal and strategy for this case?
  • Suggest the best follow-up for this case.
  • What are the conditions which worsens the symptoms of angina (in general)?

Case Study-6 (Case of Angina Pectoris)

A Mr SW is a 48-year-old man going through a stressful time at work, who for the past 6 months, has been increasingly short of breath while walking to the bus. He has put this down to his ‘unhealthy ‘ lifestyle. Although he has cut down from two packets to one packet of cigarettes a week, reduced his alcohol intake from about 40 units to 25 units a week and is trying to lose weight (currently 1.8 m tall, weighing 100 kg). He sometimes finds himself short of breath with mild chest tightness, especially when he is running late. He has a strong family history of cardiovascular disease with his father having a stroke in his early 50s and his older brother having had a CABG (coronary artery bypass graft) 2 years ago. Both have encouraged Mr. SW to see his general physician due to his worsening symptoms.

Mr. SW was seen by a physician 2 years ago who prescribed  aspirin  75 mg daily,  atenolol  50 mg daily and a  GTN spray  to be used if he experiences chest pain. Since this time, he has stopped taking the aspirin, because he feels that he does not need it and he has stopped the atenolol for more than a year because he was feeling tired and read that it can cause impotence. He has used the GTN once but, after experiencing a headache and facial flushing, he has not used it since and he does not carry it with him.

After his current physician visit, he is prescribed with:

Aspirin                                              75 mg                                    daily

Simvastatin                                     40 mg                                    at night

Amlodipine                                      5 mg                                      daily

GTN spray                                                                                      when required

  • What information and counselling points would you include?
  • How is stable angina managed?
  • What options are there, if Mr. SW experiences further symptoms despite the use of amlodipine?

 Go For  Case Study-6 Answer  | Go back to  Case Studies | Go to  Home

Case Study-7 (Myocardial Infarction)

A 50 year-old male ( Height -154 cm , weight – 70 kg ) who was auto driver visited to the clinic with chief complaints of chest pain. The patient was apparently alright till 3 h back, when he suddenly felt a vague chest pain present at the center of the chest. Pain was located in the substernal location and was radiating to the right side of the shoulder. The quality of the pain was dull aching, which was increasing in severity rapidly over few hours. Chest pain was aggravated by exertion. Pain was not relieved even during rest. There was profuse sweating associated with the chest pain, and also a sense of doom or impending death. There were mild dyspnea and palpitation associated with the chest pain. There was no history of pedal edema, abdominal distension and facial puffiness, loss of appetite/fullness, right hypochondriac pain, or increased neck pulsations. Chest pain  was  diffuse in nature and  not  localized. There is no relation of the chest pain to food intake. There were no associated vomiting and hematemesis. There was no history of trauma and no history of any psychological disorders in the past. Review of other systems was normal.

Past Medical History:

The patient is not a known diabetic, or hypertensive. He has had no similar history in the past.

Past Medication History:

The patient is not on any medications. There was no history of any intake of any cardiotoxic drugs (cancer chemotherapy or prolonged steroids).

Personal History:

The patient takes mixed diet and smokes 1 packet cigarette/day for last 15 years. There was no alleged history of any alcohol or illicit drug abuse.

Pulse rate                             : 120/min      with    regular rhythm

Blood pressure                    : Left hand – 138/94 mm Hg

                                                 Left leg   -. 144/90 mm Hg.

Lab Findings:

Hb                                           : 15 g/dL

WBC                                       : 10,000 u/mcL

Creatinine clearance        : 90 ml/min

ECG Finding :

There is ST elevation >2 mm in v2–v6, and >1 mm ST-elevation in lead 1 and aVL with some minimal reciprocal changes seen in lead 3 suggestive of anterior wall + lateral wall MI due to complete left anterior descending (LAD) occlusion. Likely in the proximal LAD.

Diagnosis : Acute Myocardial infarction

Treatment :

T. Aspirin                               75 mg            OD

T. Ticagrelor                        90 mg           BD (after initial loading dose of 180 mg)

T. Atorvastatin                    80 mg            OD HS

T. Metoprolol                      50 mg            OD

T. Ramipril                            2.5 mg           BD

T. Lasix + Spironolactone (20/50) mg   OD

1. What is the latest definition of STEMI?

2. What are the classical ECG criteria for diagnosing STEMI ?

3. What are the different types of MI ?

4. Justify the treatment given to this patient.

5. Determine the duration of DAPT In this patient ?

For Case Study-7 Answers | Go to Guidelines | Go to other Case Studies

Case Study-8 (Myocardial Infarction)

Mr TR, a 54-year-old man, presented to his general physician with sudden-onset epigastric pain that started the previous night and ‘felt like trapped wind’ . This radiated through his back, up to his neck and into both shoulders/arms. He was extremely flatulent.

The patient was referred for coronary angiogram/percutaneous coronary intervention (PCI), which was carried out on site the same day. It showed the presence of a thrombus in the OM (obtuse marginal coronary artery). The distal LAD (distal left anterior descending coronary artery) and the RCA (right coronary artery) were 80% and 60- 70% stenosed, respectively.

Echocardiogram: normal LV function.

Diagnosis: NSTEMI

Two drug- eluting stents were inserted into the OM. The following drugs were prescribed post-PCI:

  • Aspirin                                              75 mg                        daily
  • Clopidogrel                                     75 mg                        daily
  • Bisoprolol                                        2.5 mg                      daily
  • Ramipril                                           1.25 mg                     at night
  • Atorvastatin                                   40 mg                        at night
  • Lansoprazole                                  30 mg                        daily

Mr TR was discharged after 3 days, after an uneventful outpatient stay, with the drugs listed above and a GTN spray to be used sublingually. Arrangements were made for an exercise tolerance test (ETT) in 6 weeks to determine whether further PCI was indicated, and for Mr TR to enter the local cardiac rehabilitation programme.

  • Is the diagnosis & treatment given to Mr TR justified? Explain.
  • Is there a need of counselling?  

Go for Case Study-8 Answers | Explore more Case Studies | Go to Guidelines

Case Study-9 (Hyperlipidemia)

A 62-year-old male is referred for management of elevated cholesterol. He has history of obesity, hypertension, and hyperlipidemia. He had a non–ST-segment elevation myocardial infarction (NSTEMI) one year ago with drug-eluting stent placement in his right coronary artery. His current medications include aspirin 81 mg daily, lisinopril 20 mg daily, and metoprolol XL 50 mg daily. His physical exam is notable for a body mass index (BMI) of 32 kg/m 2  but is otherwise unremarkable. His blood pressure is 135/85 mm Hg.

A recent lipid panel shows the following:

  • Total Cholesterol: 226 mg/dL
  • Triglycerides: 154 mg/dL
  • High-Density Lipoprotein Cholesterol (HDL-C): 39 mg/dL
  • Low-Density Lipoprotein Cholesterol (LDL-C): 190 mg/dL
  • He has a normal creatinine and normal liver enzymes. His TSH and vitamin D levels are within normal limits.
  • What would be the target goal for LDL-C in this patient?
  • What is the drug of choice in this patient to treat LDL-C?
  • What would be the treatment plan, if the patient’s LDL-C goal is not reached even after initiation of statin therapy?
  • What would be choice of drug in this patient, if he is intolerant to statin therapy?

Go for Case Study-9 Answers | Explore more Case Studies | Refer Guidelines

Case Study-10 ( Hyperlipidemias)

Four months ago, a 46-year-old man was admitted to hospital with acute chest pain. A subendocardial inferior MI was diagnosed and he was treated with thrombolytics and aspirin. After discharge, he complained of angina, and coronary angiography was performed. This showed severe triple-vessel disease not suitable for stenting, and coronary artery bypass grafting was performed. He is attending a cardiac rehabilitation clinic and he has had no further angina since his surgery.

Family History:

He has a strong family history of ischaemic heart disease, with his father and two paternal uncles having died of myocardial infarctions in their 50s; his 50-year-old brother has angina. He is married with two children.

Social History:

He smokes 25 cigarettes per day and drinks at least 40 units of alcohol per week.

Medication History:

He is taking atenolol and aspirin.

Examination :

He is slightly overweight (85 kg; body mass index ) 28). He has tar-stained nails. He has bilateral corneal arcus, xanthelasmata around his eyes and xanthomata on his Achilles tendons. He has a well-healed midline sternotomy scar. His pulse is 64/min regular, blood pressure 150/84 mmHg. He has no palpable pedal pulses. His respiratory, gastrointestinal and neurological systems are normal.

Lab Investigation:

medical case study questions and answers

Urinalysis : no abnormality detected.

  • What is the metabolic abnormality present? 
  • Discuss the Patient counselling for this case?

Go to Case Study-10 Answers | Explore more Case Studies | Refer Guidelines

Case Study-11 ( Arrthymias )

(Arrhythmia) A 2 month-old male who presents to the emergency room with a chief complaint of fever, lethargy, and poor feeding for the past 36 hours. His parents began noticing increasing lethargy and tiring with feeding and increased work of breathing for about 12 hours prior to presentation.

He is the product of a G2P1, full term, uncomplicated pregnancy and spontaneous vaginal delivery. Nursery course was uneventful.

Vital Sign: Temp: 99.32 ᵒF,

        Heart Rate: 240 per min,

        Respiratory Rate: 72 per min,

        BP: 87/64mmHg, oxygen saturation 98% in room air.

He is well developed, well nourished, but pale, lethargic and tachypneic, with mild subcostal retractions. HEENT exam is normal . Neck is supple without adenopathy.

Lungs have good aeration with fine crackles and mild retractions. His heart is tachycardic with a regular rhythm. No murmur, rub, or valve clicks are heard. His abdomen is soft, non-distended, non-tender, and without masses. His liver is 2 to 3 cm below right costal margin. His feet and hands are cool.

His peripheral pulses are 1+ to 2+ (out of 4+) throughout. Capillary refill time is 3 to 4 seconds. He has no rashes or other significant lesions.

Chest x-ray shows mild cardiomegaly and mild pulmonary edema. A 12 lead electrocardiogram shows a narrow complex tachycardia (rate of 240 bpm) with no visible P-waves (rhythm strip below).

The patient is felt to be in supraventricular tachycardia and mild congestive heart failure.

A peripheral IV is started and he is given a rapid IV bolus dose of adenosine. The patient immediately becomes briefly bradycardic followed by resumption of a normal sinus rhythm at a rate of 140 beats per minute. He is admitted for overnight observation and initiation of an anti-arrhythmic medication.

A 12-lead electrocardiogram (ECG) following conversion shows no evidence of a delta-wave, so he is started on digoxin .

  • What medicine used to treat Supraventricular tachycardia is contraindicated specifically in Wolff-Parkinson-White syndrome?
  • What would be the possible differential diagnosis?

Go For Case Study-11 Answers | Explore More Case study | Go to Guidelines

Case Study-12 ( Atrial Fibrillation )

A 67-year-old man presents to the emergency department with palpitations and dyspnea which began approximately 4 hours ago. He has a history of hypertension, diabetes, and gastroesophageal reflux disease, and LVH. On further questioning, he reports drinking 1 cup of coffee daily and 1-2 beers on the weekends.

He denies binge drinking and the use of herbal or alternative medications. He quit smoking 10 years ago. His urine drug screen is negative. His body mass index is 36 and he admits to snoring and daytime sleepiness. He is at high risk for obstructive sleep apnea (OSA). His current medications include lisinopril, metformin, and omeprazole. He has no history of congestive heart failure, stroke, or transient ischemic attack (TIA).

He appears to be in mild respiratory distress.

Blood pressure is 88/60 mmHg, pulse rate is 140 bpm, respiratory rate is 24/min, and temperature is normal. Oxygen saturation is 90% on 40% oxygen by face mask. Cardiac exam reveals tachycardia with an irregularly irregular tachycardic rhythm. There are crackles in the lower lung fields.

Electrocardiogram demonstrates atrial fibrillation (AF) with rapid ventricular rate.

  • What would be the most appropriate initial management in this patient?
  • Calculate the risk of thromboembolism and risk of bleeding in this patient?

Go for Case Study-12 Answers | Explore more Case Study | Go to G uidelines

Case Study-13 ( Case study of Asthma )

A 29-year-old man with mild persistent asthma presented to an outpatient office for a follow-up visit.  He was originally referred 6 months ago by his primary care provider after having an asthma exacerbation which required treatment in an emergency room.

At his initial visit, he reported wheeze and cough 4 days a week and nocturnal symptoms three times a month.  Spirometry revealed forced vital capacity (FVC) 85% predicted, forced expiratory volume in 1 second (FEV 1 ) 75% predicted, FEV 1 /FVC 65%, and an increase in FEV 1  of 220 ml or 14% following an inhaled short-acting bronchodilator.  He was placed on a low-dose  inhaled corticosteroid  twice a day and a short-acting  inhaled beta-agonist  as needed. 

He returned 4 weeks later improved, but with continued daytime symptoms 2 days a week.  He also had symptoms of rhinitis; therefore he was referred to an allergist for evaluation. Skin testing was positive for trees, ragweed, dust mites, and cats, and he was prescribed a nasal steroid spray and nonsedating oral antihistamine.   He presents today and reports no asthma exacerbations since his last visit. 

Furthermore, during the past 4 weeks, he has not been awakened by his asthma, experienced morning breathing symptoms, missed work, had any limitations in activities due to asthma, or required the use of rescue albuterol.  He currently denies shortness of breath or wheezing.  He performs aerobic exercise 4 days a week for 45 minutes per session without symptoms, provided he premedicates with a short-acting inhaled beta-agonist.  His review of symptoms is otherwise unremarkable. 

His current medications include low-dose inhaled corticosteroid, 1 puff twice a day; steroid nasal spray, 2 puffs each nostril daily; a nonsedating antihistamine, 1 tablet daily; and inhaled beta-agonist, 2 puffs as needed.   

His  past medical history is significant for intermittent asthma diagnosed at age 13 and frequent “colds.”  He has never required hospitalization for an asthma exacerbation.  He works as a hospital microbiologist and does not smoke, drink alcohol, or use illicit drugs.  He recently moved to a pet-free apartment complex and instituted dust mite protective barriers for his bedding.  His family history is noncontributory.

Physical Exam

On physical exam, he is an age-appropriate man in no acute distress.  His height and weight are proportionate and resting oxygen saturation as measured by a pulse (SpO 2 ) is 98% on room air.  A Head and neck exam revealed mild erythema of the nasal mucosa.  A heart exam revealed normal heart tones, no murmurs, gallops or rubs, and the lungs were clear to auscultation.  Extremities were free of oedema, cyanosis, or clubbing.

Lab:  In the office, spirometry is completely normal. He states he feels great and inquires about stopping his inhalers, particularly his inhaled steroid.

1. Based on current evidence, which of the following would be the most appropriate recommendation regarding his asthma medication regimen?

A. Maintain current medication regimen; no adjustment is indicated.

B. Discontinue the inhaled corticosteroid; maintain on an Inhaled beta-agonist as needed.

C. Decrease the inhaled corticosteroid to 1 puff daily.

D. Discontinue the inhaled corticosteroid; start a leukotriene modifier at bedtime.

E. Discontinue the inhaled corticosteroid; start low-dose inhaled corticosteroid/long-acting beta-agonist, 1 inhalation at bedtime.

2. Which of the following should be done routinely with each follow-up visit?

A. Methacholine challenge testing

B. Sputum for eosinophils

C. In-office peak flow recordings

D. Review of proper inhaler technique and adherence

E. Measurement of exhaled nitric oxide (NO)

3. What findings would suggest that the patient requires a step-up in asthma medication?

A. Two or more nighttime awakenings per month due to Asthma.

B. Two or more interruptions in daytime activities per month due to asthma .

C. Peak flow readings 85% of personal best

D. Short-acting beta-agonist for rescue once a week

E. Short-acting beta-agonist 4 days/week prior to exercise

4. The patient was provided with an asthma action plan to follow at home. Which component of the asthma action plan is considered the most critical element for improving asthma outcomes?

A. A list of the patient’s controller and rescue medications

B. A list of symptoms indicative of worsening asthma

C. Medication changes based on personal-best peak flow readings

D. Instructions describing when, how, and how long to increase medications when symptomatic

E. Medication changes based on symptoms

5.  How often is spirometry testing recommended if the previous readings are normal and the patient’s asthma is well controlled?

A. Every 1 to 2 years

B. Only if asthma controller medications are changed

C. Only if symptomatic

D. Every 4 to 6 months

E. Every follow-up visit

Image result for asthma

Explore more  Case Studies  | Go to  Case Study-13  Answers  |  Guidelines

Case Study-14 ( Case Study - II of Asthma )

A 14-year-old girl with a history of asthma requiring daily inhaled corticosteroid therapy and allergies to house dust, mites, cats, grasses, and ragweed presents to the emergency department in mid-September, reporting a recent “cold” complicated by worsening shortness of breath and audible inspiratory and expiratory wheezing.

She appears frightened and refuses to lie down but is not cyanotic.

Her pulse is 120 bpm, and respirations are 32/min.

Her mother states that she has used her albuterol inhaler several times a day for the past 3 days and twice during the previous night. She took an additional two puffs on her way to the emergency department, but her mother states that “the inhaler didn’t seem to be helping so I told her not to take any more.

1. What emergency measures are indicated?

2. How should her long-term management be altered?

Case Study-14 Answer | More Case Studies | Guidelines

Case Study-15 ( Chronic Obstructive Pulmonary Disease : COPD Case Study)

Mrs. Glenda is a 60-year-old and has recently retires from her job working for a firm that manufactures fabrics. She is a thin lady who appears older than her stated age. Glenda visits her GP as she beginning to get short of breath while climbing the stairs and is struggling to walk to the end of her road. She explains to the GP that for the past six months she has become increasingly short of breath while carrying out daily activities.

Glenda takes no regular medications and has no history of drug allergies .

She says she has not had any acute changes in her breathing, but she does have a chronic cough that produces around one or two tablespoons of clear sputum daily. Her cough has not changed recently, and the colour and volume of her sputum have also remained unaltered. She says she is not suffering from chest pains or wheezing and has not been coughing up blood.

She has smoked at least ten cigarettes a day since she was 20 but has recently cut down to five a day because of her shortness of breath. In the last few years, she has had at least two chest infections each year requiring treatment with antibiotics.

The GP suspects COPD and conducts spirometry testing, the results of which are:

1. What clinical features and risk factors of COPD does glenda exhibit?  What grade of severity does glenda’s COPD fall into?

2. What initial treatment would you recommend for glenda?

3. Glenda continues to report that her breathlessness is getting worse. Her medical research council dyspnea score is now four and in the last few days she has been producing more sputum than usual. Her sputum has turned a yellow green colour. What does these changes indicate & what treatment would you recommend?

Explore more Case Studies | Guidelines | Case Answers of this COPD Case Study-15

Case Study-16 ( Case of COPD with comorbidity )

A 63-year-old woman, 67 kg, is admitted to hospital with chest pain, shortness of breath and sweating. She is seen in casualty and treated using a salbutamol nebuliser. She looks obese. She has been a life-long smoker who stopped one day ago.

Her previous medical history includes chronic obstructive pulmonary disease (COPD) for 10 years, the last admission to hospital was two weeks ago; ischaemic heart disease since 1995, myocardial infarction 4 years ago; osteoporosis diagnosed 3 years ago; hypertension diagnosed 9 years ago; and pulmonary embolism two months ago.

On examination:

Blood pressure                    105/90 mmHg

Heart rate                             90 bpm

Respiratory rate                 20 breaths per minute.

Arterial blood gases on admission:

pH                                           7.388 on 35% O2

PCO2                                      9.67 kPa

PO2                                        6.5 kPa.

Oxygen saturation: SpO2 89%. Lungs were hyperinflated, no wheeze, few right base crepitations.

Laboratory tests at admission  :

WCC                 16.5 × 109/L                    (4–11 × 109/L)

Na+                   140 mmol/L                     (135–145 mmol/L)

K+                      4.4 mmol/L                      (3.5–5 mmol/L)

Creatinine     75 micromol/L            (59–104 micromol/L)

Urea                 7.8 mmol/L                       (1.7–8.3 mmol/L)

Hb                     11.6 g/dL                            (13–17 g/dL)

Medication on admission:

Prednisolone                   10 mg                    o.d.

Fluticasone inhaler         500 micrograms  b.d.

Aspirin                              75 mg                    o.d.

Bumetanide                    1 mg                        o.d.

Combivent nebs             2                              q.d.s.

Enalapril                          5 mg                        o.d.

Uniphyllin Continus       200 mg                    bd

Senna                              2 tablets                  nocte

Warfarin                          5 mg                        o.d.

Zopiclone                        7.5 mg                    nocte

Diclofenac                       50 mg                     p.r.n.

Oxygen  (O2)                                     2 L nasal specs.

1.  How the sign, symptoms & pathophysiology of COPD, relates to the patient?

2.  Comment on the current drug therapy and describe the role of O2 in this patient.

3.  What are the social issues in treating this patient at home?

Case Study-16 Answer  | Explore more  Case Studies  |  Guidelines

Case Study-17 ( Type-1 Diabetes in Pregnancy )

Mrs Jaya is a 36-year-old married lady who has type 1 diabetes. She undertook a home pregnancy test because she was feeling particularly nauseated in the mornings and her period was late. The  test was positive  confirming that she was pregnant.

However, at 8 weeks, she experienced  vaginal bleeding  and  abdominal pain . She attended the Accident & Emergency department, where a miscarriage was confirmed.

Upon questioning, it was discovered that she had been taking  folic acid 400 μcg daily  for the previous 6 months but had not received any pre-conception diabetes care.

Her most recent HbA1c was 7.3% (56 mmol/mol). Her regular medications are  ramipril 10 mg daily ,  simvastatin 40 mg daily ,  insulin glargine at night  and  insulin aspart three times daily  with meals.

  • Why should women of childbearing age be offered advice about pregnancy?
  • Was she taking appropriate dietary supplements prior to conception?
  • What advice should she be given with respect to her regular medication?

Go to Case Study-17 Answers

Case Study-18 (Type-2 Diabetes)

Case scenario:.

Mk a 68-yr-old woman who has previously diagnosed with type 2 diabetes 15 years ago came to the clinic for her regular follow-up. she reports paraesthesia’s in her feet at night but her sleep was not disturbed by these symptoms. She checked her blood sugars at home two to four times each day and found them to range from 100-250 mg/dl.

She reported compliance with her medications that included lisinopril 20 mg OD, hydrochlorothiazide 25 mg OD, diltiazem 180 mg OD, glargine insulin 0.4 units/kg , atorvastatin 40 mg , and aspirin 81 mg OD.

Past medical history & medication history :

  • Type 2 diabetes, hyperlipidaemia and hypertension diagnosed at the age of 47. For these conditions she was treated with metformin, atorvastatin, and hydrochlorothiazide and maintained Hb A1c below 7.0 %, LDL below 100 mg/dl, and blood pressure below 135/80 mmHg for many years.
  • At the age 59, she was noted to have an increase in her urine albumin to 54 mg/g of creatinine and lisinopril was added to her regimen.
  • Three years later at the age 62, her serum creatinine increased to 1.56 mg/dl, and her GFR was estimated to be 36 ml/min. Her Hb A1c increased to 7.2% and she began to experience paraesthesia’s in her feet. The metformin was discontinued, and she was started on glargine insulin.
  • At age 63, she developed proliferative retinopathy in her right eye and underwent laser photocoagulation. Bilateral macular edema developed at age 65.

Examination – Present visit:

  • BMI – 38.2 kg/m² (height – 151 cm, weight – 87 kg)
  • Blood pressure – 142/90 mmHg
  • Heart rate – 68 bpm
  • Retinal exam – significant for panretinal photocoagulation changes with scattered dot haemorrhages and macular edema in both eyes.
  • Heart, lung and abdominal exams were unremarkable.
  • Extremities – Pulses in her feet were reduced, but her feet were warm and without ulcers. She was unable to detect a Semmes Weinstein 5.07 monofilament on the soles of her feet.

Lab parameters:

  • Hb A1c : 7.9%.
  • Serum creatinine : 2.45 mg/dl.
  • GFR : 20 ml/min.
  • Calcium : 9.8 mg/dl. Phosphorus                                                  : 4.6 mg/dl.
  • PTH : 75 pg/ml
  • Total cholesterol : 188 mg/dl,
  • Triglycerides :82 mg/dl,
  • High-density lipoprotein cholesterol :42 mg/dl,
  • LDL cholesterol  : 131 mg/dl
  • Hb  : 10.8 g/dl.
  • Prepare pharmaceutical care plan for this case study.

Go to Case Study-18 Answers

Case Study-19 (Thyroid Disorder)

A 55-year-old man presents to his general practitioner, complaining of lack of energy. He has become increasingly tired over the past 18 months. He works as a solicitor and describes episodes where he has fallen asleep in his office.

He is unable to stay awake after 9:30 pm, and sleeps through until 7:30 am. He finds it difficult to concentrate at work, and has stopped playing his weekly game of tennis. He had an episode of depression 10 years ago related to the break-up of his first marriage. He has no current personal problems. He has had no other major illnesses.

His brother developed type 1 diabetes mellitus at the age of 13. On direct questioning, he has noticed that he has become more constipated but denies any abdominal pain or rectal bleeding. He has put on 8kg in weight over the past year.

Examination:

On examination he is overweight. His facial skin is dry and scaly. His pulse is 56/min, regular and blood pressure 146/88 mmHg. Examination of his cardiovascular, respiratory and abdominal systems is unremarkable. Neurological examination was not performed.

Haemoglobin                                               10.3 g/dL                              13.3–17.7 g/dL

Mean corpuscular volume (MCV)          92 fL                                       80–99 fL

White cell count                                         4.3 x 10⁹/L                            3.9–10.6 x 10⁹/L

Platelets                                                        154 x 10⁹/L                           150–440 x 10⁹/L

Sodium                                                          140 mmol/L                         135–145 mmol/L

Potassium                                                    4.4 mmol/L                          3.5–5.0 mmol/L

Urea                                                               6.4 mmol/L                          2.5–6.7 mmol

Creatinine                                                    125 μmol/L                        70–120 μmol/L

Glucose                                                         4.7 mmol/L                          4.0–6.0 mmol/L

Calcium                                                         2.48 mmol/L                        2.12–2.65 mmol/L

Phosphate                                                    1.20 mmol/L                        0.8–1.45 mmol/L

Cholesterol                                                  6.4 mmol/L                          3.9–6.0 mmol/L

Triglycerides                                                1.4 mmol/L                          0.55–1.90 mmol/L

Urinalysis: nothing abnormal detected (NAD)

  • What is the likely diagnosis? Main differential diagnosis?
  • How would you further manage this patient?

Go to Case Study-19 Answers

Case Study-20 (Thyroid Disorder)

A 40-year-old woman (51kg) presents with complaints of left side chest pain since 4 months that last for 1-2 hours and radiates to back, increase on exertion; associated with palpitation, perspiration. Headache since 1 month in occipital region that lasts 30-40 min. Generalised weakness, easy fatigue, facial swelling, peri-orbital swelling, bilateral lower limb swelling.

Past medication history :

Blood transfusion 2 times 4 months back.

Temp : Afebrile                               Pulse: 86 beats/min

BP: 130/84mmHg (pattern for hypotension seen for 2 days)

Lab details:

Hb                                           10.1 g%                                 13-17g%

WBC                                      4600 cumm                        4000-11,000 cumm

PCV                                        31.3%                                    36-47%

MCV                                       85.2fL                                   82-92fL

MCH                                       27.6pg                                   27-31pg

MCHC                                    32.3%                                    32-36%

Platelet                                1.55 lac/cumm                    1.5-4.5 lakh/cumm

RBC                                        3.67 millions/ μ L              4.4-5.9 millions/ μ L

RDW                                      17.7%                                    13-15 %

Pus & epithelial cells        1-2/hpf

Reticulocyte count            0.5%                                       0.5-2.5%

Na                                           135 mmol/L                        135-145mmol/L

K                                              4.2mmol/L                           3.5-5.5mmol/L

Cl                                            100mmol/L                          98-110mmol/L

Creatinine                           0.8mg%                                0.6-1.3 mg%

Glucose : RBS                      103mg%                               70-140mg%

RBC Smear : Mild anisocytosis                                                                                  

Stool Test ( Occult Blood) : Positive

TSH                                         >100 miu/ml                       >15miu/ml

Total T4                                0.10 ng /dL                          0.82- 2ng/dL                      

Free T4                                  0.10ng /dL                           0.9- 2.3 ng/dL        

ECG : normal sinus rhythm , poor P-wave progression

LVEF: 60%; Grade 1 diastolic function; mild MR; mild TR; mild PAH; Concentric LVH

            Local part

Findings: Left lobe P/O  — hyperplastic nodule

                  Both lobe P/O – inflammatory etiology thyroiditis

Inj pantoprazole    40mg                                                                         12hrly

Infusion 0.45% NS 500ml + 2 ampoule Optineuron at 60cc/hr

Tab Fdson MP Forte                                                                                0-1-0

Tab febac XT                                                                                             1-0-1

Tab Thyronorm      25mg                                                                        1-0-0

  • What is the likely diagnosis in this case? And are lab finding clinically justified?
  • What are the Pharmacist Intervention points in this case?
  • Patient counselling regarding drug & disease?
  • Write the Generic Names of above advised drugs.

Go to Case Study-20 Answers

Case Study-21 (Thyroid Disorder)

A 34-year-old pregnant woman came to the emergency department with complaints of fetal movements not felt since 1 day ago.

At present the patient is the third pregnant at the age of 23-24 weeks, so far the regular control to the midwife and specialist doctors.

The patient has never experienced bleeding or trauma before. A history of previous labour was normal with a healthy baby. The patient has been suffering from Grave’s disease for the past 10 years and has not been treated for one year.

During pregnancy, the patient feels palpitations and tightness during activity. The patient also complaint about weight loss and fatigue.

The patient’s general condition appeared weak with a blood pressure of 197/87 mmHg and pulse 148 times per minute regular.

The exophthalmos and the thyroid gland feel soft in the neck without pain.

Laboratory tests results:

 An increase in FT4 levels of 75.62 pmol / L and low TSHs levels of 0.005 µIU / mL. Wayne index with a value of 23 or found signs of toxicity and the Burch Wartofsky scale with a value of 45 or impending thyroid storm.

Q1 . What is the likely diagnosis ?

The patient was taken to the endocrine section and given methimazole 30 mg twice daily, propranolol 30 mg twice daily, and Lugol 5 drops per 6 hours .

The termination of pregnancy is carried out by vaginal delivery and administration of oxytocin postpartum.

The patient complained of shortness of breath and anxiety after two hours postpartum.

Patient was consulted in the Cardiology division with pulmonary oedema and advised giving diuretics and vasodilators .

Patient was given nitroglycerin at a dose of 5 meq per hour and furosemide 30 mg per hour intravenously to improve the condition of pulmonary oedema.

Close monitoring is carried out on the patient for several days until the patient’s condition is stable.

Q2. Was the Treatment given to the patient justified according to clinical condition?

Q3. Widely used anti-thyroid drugs in pregnant women with hyperthyroidism?

Patient is planned to go home for outpatient care after monitoring side effects and postpartum complications. Hemodynamic condition is stable with blood pressure 110/70 mmHg and pulse 92 times per minute. Patient was given anti-thyroid therapy and beta-blockers on discharge. During treatment at home, patient is advised to monitor thyroid function to the clinic every once a month.

Q4. What should be the Patient Counselling points ?

Go to  Case Study-21 Answers

Case Study- 22 (Thyroid Disorder)

A 55-year-old woman presents with complaints of pus discharge per vagina since 8-9 months. Decreased urine output. Palpitation 3-4 months. Shortness of breath 4-6 months

Past medical history :

Hysterectomy 10-12 years ago

Social History :

“ Chulha ” user

Temp : Afebrile                               Pulse: 140 beats/min

BP: 126/80mmHg

Provisional Diagnosis :

Atrial Fibrillation w/ hemorrhoids w/ hyperthyroidism w/ perianal fissure.

Hb                                           13.2 g%                                 13-17g%

WBC                                       8100 cumm                         4000-11,000 cumm

Platelet                                  2.11 lac/cumm                    1.5-4.5 lakh/cumm

Pus cell                                  5-6

Epithelial cells                     1-2/hpf

Na                                           142 mmol/L                        135-145mmol/L

K                                              3.8mmol/L(3 rd day: 2.9)          3.5-5.5mmol/L

Cl                                             106mmol/L                         98-110mmol/L

Creatinine                             0.6mg%                                0.6-1.3 mg%

Urea                                       24

Mg                                          2.1

Ca                                          8.6

Protein                                 6.1 g %                                    6.0-8.0 g %

Albumin                                3.4g%                                    2.7-5.0 g %

Globulin                                2.7 g%                                   2.5-4.0 g %

SGOT                                     59

SGPT                                      27

Bilirubin                                1.6

TSH                                         0.01                                        0.39-5.0

Free T3                                  7.01                                        2.1-3.8

Free T4                                  4.28ng /dL                            0.9- 2.3 ng/dL         

CRP                                         <10                                         0-6.0

ECG : Flat T- wave (V5) atrial fibrillation.

ECO : Mild MR; Mild TR; LVEF: 60%

Inj  Dilzem                            12.5mg                                 stat w/ 10ml NS

Tab Dilzem                           30mg                                     1-1-1

Inj Lasix                                20mg                                     1-0-0

Inj PAN                                  40mg                                     24 hrly

Tab Metro                           400mg                                  1-1-1             

Syp Duphalac                     2tsp                                       0-0-1             

Tab Ciplox T2                      500/600                               1-0-1             

Tab Neomercazole            10mg                                                 1-1-1             

Tab Propanolol                  10mg                                    1-0-1

Syp Potklor                          15ml                                     1-1-1 

Metrogyl ointment                                                           1-1-1-1 

Inj NS + KCl 1 amp                                                             12hrly                       

  • Are there any drug-drug interactions?
  • Is the dosing of medication for hyperthyroidism, according to standard treatment guidelines?
  • What should be Patient counselling in this case, regarding drug & disease?

Go to Case Study-22 Answers

Case Study-23 (OB/GYN: Oral Contraceptive Use)

An 18-year-old female presents with an absence of periods for 6 months. This has occurred twice before in the past but on both occasions menstruation returned so she was not too concerned. Her periods started at the age of 12 years and were initially regular.

She has no medical history of note and denies any medication. She is currently in her first year at university. She sometimes follows inconsistent diet plan. However, She runs most days and reports a ‘healthy ‘ diet avoiding carbohydrate foods and meat. She is the oldest of three siblings and her parents separated when she was 12 years. She has minimal contact with her father and lives mainly with her mother who she says she gets on well with. She has had a boyfriend in the past but has veered away from any sexual relationships.

The woman is tall and thin with a body mass index (BMI) of 15.5 kg/m².

There is evidence of fine downy hair growth on her arms.

Heart rate is 86/min and blood pressure 100/65 mmHg.

Abdominal examination reveals no scars or masses, and genital examination is not performed.

INVESTIGATIONS:

Follicle-stimulating hormone                  1.0 IU/L                     Day 2-5

 Luteinizing hormone                                0.8 IU/L                     Day 2-5

0.5-14.5 IU/L

Oestradiol                                                    52 pmol/L                 70-600 pmoI/L

Prolactin                                                       630 mu/L                  90-520 mu/L

Testosterone                                               1.6 nmol/L                0.8-3.1 nmol/L

Diagnosis : Hypogonadotrophic hypogonadism

  • What should be the further investigation ?
  • How, this patient would be managed ?
  • What Patient counselling points should be included in this case?

Go to Case Study-23 Answers

Case Study-24 (OB/GYN: Oral Contraceptive Use)

A 19-year-old female was referred by her general practitioner with increased hair. She first noticed the problem when she was about 16 years old and it has progressively worsened such that she now feels very self-conscious. It also affects her forming relationships.

The hair growth is noticed mainly on her arms, thighs and abdomen. Hair has developed on the upper lip more recently. She has tried shaving but this seems to make the problem worse. She feels depilation creams are ineffective. Waxing is helpful but very expensive and she has bleached her upper-lip hair.

Her GP has not prescribed any medication in the past.

There is no significant previous medical history of note. Her periods started at the age of 13 years and she bleeds every 30-35 days. The periods are not painful or heavy and there is no intermenstrual bleeding or discharge. She has never been sexually active.

Examination

On examination she has an increased body mass index (BMI) of 29 kg/m². The blood pressure is 118/70 mmHg. There is excessive hair growth on the lower arms, legs and thighs and in the midline of the abdomen below the umbilicus. There is a small amount of growth on the upper lip too.

The abdomen is soft and no masses are palpable. Pelvic examination is not indicated.

INVESTIGATIONS

Follicle-stimulating hormone (FSH)                  7 IU/L                         Day 2-5

                                                                                                                        1-11 IU/L

Luteinizing hormone (LH)                                    12 IU/L                      Day 2-5

                                                                                                                        0.5-14.5 IU/L

Prolactin                                                                   780 mu/L                  90-520 mu/L

Testosterone                                                           3.2 nmol/L                0.8-3.1 nmol/L

Thyroid-stimulating hormone                            4.9mu/L                    0.5-5.7 mu/L

Free thyroxine                                                         14.7pmol/L              10-40pmol/L

Provisional Diagnosis : Polycystic ovarian syndrome(PCOS)

Clinical features: Hirsutism , acne, increased BMI and slight menstrual irregularity

Go to Case Study-24 Answers

Case Study-25 (Hormone Replacement Therapy)

History of present illness :.

A 51-year-old newly postmenopausal woman suffering from intense hot flashes and night sweats, as well as anxiety, insomnia, and stress, presents to the hospital.

The doctor presents a breakdown of the patient’s self-reported symptoms before going into the details of her test results – where she highlights the biochemical factors that explain both her symptoms and her observed results.

Symptoms she rated as severe , included, depression, anxiety, and sleep disturbances , moderate symptoms included hot flashes, night sweats, foggy thinking, vaginal dryness, and mood swings. In addition, she had multiple symptoms that she rated as mild in severity.

The Patient had already tried multiple supplements in an attempt to address her own symptoms – these included elements like magnesium, calcium, selenium, zinc, and copper, as well as L-theanine and Rhodiola, vitamin D, fish oil, and some probiotics.

Investigation:

Saliva Hormone Test Results:

Name                                                      Lab value                     Normal range

Estrogen hormone                                     0.3                                0.9-3.1 pg/ml

Progesterone hormone                            5                                   12-100 pg/ml

DHEA                                                           1                                   2-23 ng/ml

Testosterone                                              90                                 16-55 pg/ml

Neurotransmitter Test revealed:

LOW levels of:

Nor Epinephrine

  • What should be the further investigation for this patient ?
  • Give the provisional diagnosis for this case.

Go to Case Study-25 Answers

Case Study-26 (Hormone Replacement Therapy)

A 25 year-old woman with menarche at 13 years and menstrual periods until about 1 year ago.

Complains of hot flushes, skin and vaginal dryness, weakness , poor sleep and scanty and infrequent menstrual periods of a year duration.

She visits her gynecologist, who obtains plasma levels of follicle-stimulating hormone and luteinizing hormone, both of which are moderately elevated.

She is diagnosed with premature ovarian failure, and estrogen and progesterone replacement therapy is recommended.

A dual energy absorptiometry scan (DEXA) reveals bone density t-score f <2.5 SD, ie., Frank osteoporosis.

  • How should the ovarian hormones she lacks be replaced?
  • What extra measures should she take for her osteoporosis while receiving treatment?

Go to Case Study-26 Answers

Case Study-27 (Osteoporosis)

History of present illness:.

     A 73-year- old woman presenting for a physical examination who looks and feels healthy and well.

Medication history:

Takes a multivitamin daily plus a calcium tablet

History of low-trauma Colles’ fracture (11 years ago)

Physical Examination:

Weight:  55 kg (121 lbs.)

Height: 157 cm (5’2”)

Body Mass Index (BMI): 22.3 kg/m2

*Changes in height and weight can be signs of vertebral fractures

INVESTIGATION:

BMD :                 Value                       

Spine                -3.6;

Hip                     -2.0

Provisional Diagnosis : Osteoporosis with moderate risk of fracture

  • What all investigations are needed for the further management?
  • How this patient would be managed ?
  • How will you do the Patient counselling in this case?
  • Mention the indications for BMD testing.

Case Study-28 (Osteoporosis)

A 64-year-old retired firefighter Retired nine years ago; now doing contract carpentry Presents for physical examination, complaining his back has been “worse than usual” the past three weeks.

On no medications

Prior smoker (45 pack/year history)

Quit smoking one year ago

Height: 180 cm (5’11”)

Patient recalls being 185.5 cm (6’1”)

Weight: 80 kg (up 5 kg from one year ago)

Body mass index (BMI): 24.7 kg/m2

*Other indicators of vertebral fracture in physical examination: Rib-pelvis distance and occiput-wall distance

INVESTIGATION:      

  Screening for osteoporosis with dual energy X-ray absorptiometry (DXA) is  T-score -1.9 at femoral neck

Lateral thoraco-lumbar spine X-ray is ordered to rule out vertebral compression deformities

*The radiologist makes note of two vertebrae being wedge shaped and just meeting the criteria for vertebral compression fracture

  • What is the significance of T- Score?

Go to Case Study – 28 Answers

Case Study-29 (Glaucoma)

The patient, a 61 year-old female retired school teacher, has not had an eye exam in 10 years. She reports no problems with driving, watching TV, computer or reading. She uses over-the-counter readers for close work. She admits to mild eye burning feeling after reading for long periods of time or in the afternoons. She denies any flashes, floaters, pain, redness or double vision.

Past Ocular History:

Presbyopia. No prior eye surgeries, hx of eye trauma, amblyopia or strabismus.

Ocular Medications:

Hypertension

Surgical History:

Cesarean delivery x 1

Past Family Ocular History:

Cataract surgery in her mother and father. Negative for macular degeneration, glaucoma or blindness.

Never smoked

Medications :

Hydrochlorothiazide

Allergies :

Denies any recent illness or any new CNS, heart, lungs, GI, skin or joint symptoms.

Ocular Exam:

Visual Acuity (cc):

IOP (tonoapplantation):

OD: 21 mmHg

OS: 23 mmHg

Equal, round and reactive to light, no APD

Extraocular Movements:

Full OU, no nystagmus

Confrontational Visual Fields:

Full to finger counting OU

Normal, both sides

Lids and Lashes:      Normal OU

Conjunctiva/Sclera:         Normal OU

Cornea:                 Clear OU; no krukenberg spindle or embryotoxon

Anterior Chamber: Deep and quiet OU

Iris:              Normal, no neovascularization or atrophy

Lens: 1+ nuclear sclerotic cataracts OU

Anterior Vitreous: Clear OU

Dilated Fundus Examination:

OD:    Clear view, CDR 0.7 with sharp optic disc margins (no obvious rim thinning or disc hemorrhage); flat macula with normal foveal light reflex; normal vessels and peripheral retina.

OS:    Clear view, CDR 0.8 with sharp optic disc margins (no obvious rim thinning or disc hemorrhage); flat macula with normal foveal light reflex; normal vessels and peripheral retina

Gonioscopy: Open angles with minimal pigmentation in the trabecular meshwork, no synechiae OU

Automated visual field test: superior arcuate defect in both eyes

Pachymetry: 560 OD; 551 OS (within normal range)

  • What findings are needed for the diagnosis of POAG?
  • Explain the result of the above mentioned investigations of the ocular exam?
  • Enlist the Patient counselling points in this case?

Go to Case Study – 29 Answers

Case Study-30 (Galucoma)

The patient is a 51 year-old stay-at-home mother who presented to the ED with severe R brow, R cheek and R eye pain/pressure that started 2-3 hrs prior. She also noticed blurry vision from that eye and rainbow-colored halos around lights around the same time. 

Accompanying symptoms include acute nausea. She has vomited twice since feeling eye pain. Denies prior episodes. Denies flashes, floaters or diplopia. There is mild redness in the R eye.

Hx myopia OU No prior eye surgeries, trauma, amblyopia or strabismus

Degenerative disc disease – lower back

Father: chronic angle-closure glaucoma. No FHx of macular degeneration or other blinding diseases.

30 pack/year smoking history Drinks alcohol on occasion No illicit drug use

Medications:

Multivitamins Vicodin prn (uses few days/month for back pain)

Denies recent illnesses, new medications, CNS, lungs, GI, skin, joint problems except for above.

Ocular Exam

Visual Acuity (cc): OD: 20/70 OS: 20/20 IOP (tonoapplantation): OD: 62 mmHg OS: 11 mmHg Pupils: OD: pupil mid dilated, sluggish to respond to light. OS: pupil round and reactive to light No obvious APD Extraocular Movements: Full OU No nystagmus Confrontational Visual Fields: Full to finger counting OU External: Normal, both sides

Lids and Lashes: Normal OU Conjunctiva/Sclera: Mild diffused injected conjunctiva OD, Normal OS Cornea: Hazy cornea OD, Normal OS Anterior Chamber: Shallow anterior chamber 360 OD – hazy view, Deep and quiet OS Iris: Mid dilated iris OD, Normal OS Lens: Trace nuclear sclerosis OU Anterior Vitreous: Clear OU

OD: Dilation not performed, examination through undilated pupil showed hazy view, CDR 0.5 with sharp optic disc margins; flat macula OS: Dilation not performed, examination through undilated pupil showed clear media, CDR 0.4 with sharp optic disc margins; flat macula

Gonioscopy: Performed with Abraham 4 mirrored lens, shallow angle and no view of the angle structures 360 degrees OD, shallow angle with view of the trabecular meshwork 360 degrees OS without synechiae, mild pigmentation 360 degrees OS

1. What is the Diagnosis in this case? 2. What is the mechanism of angle closure in an episode of acute angle closure glaucoma resulting from a pupillary block? 3. Mention in brief the surgical options for the disease? 

Go to Case Study – 30 Answers

Case Study - 31 (Conjunctivitis)

The patient is an 31 year-old male who reports a 4-day history of irritation and itching, first in the L eye followed by the R eye one day later. Both eyes have also had a mild yellow discharge and mattering of the eyelids making it difficult to open the eyes in the morning. There is minimal eye redness but no foreign body sensation, flashes, floaters, decreased vision or diplopia. Not using any drops. No environmental exposures to the eyes.

No history of eye trauma, surgery, amblyopia or strabismus. No history of contact lens use.

Born at term without complications

No history of glaucoma, macular degeneration or other blinding diseases

No smokers at home

Exposure to the common cold (neighbor friend). No history of environmental allergies, recent cold, CNS, heart, lung, GI, skin or joint problems.

OD: 17 mmHg

OS: 14 mmHg

Equal, round and reactive OU, no APD

Normal-appearing orbital structures; no redness or swelling either eye

Lids and Lashes:           Crusted dry flaky material on eyelashes OU, no follicles in inferior or superior fornix OU. No foreign body in fornices OU

Conjunctiva/Sclera               Mild conjunctival injection OU, no chemosis

Cornea                           Clear OU, no infiltrates

Anterior Chamber               Deep and quiet OU

Iris                       Normal OU

Lens                   Normal OU

Anterior Vitreous            Clear OU

Dilated Fundus Examination :

OD     Clear view, CDR 0.2 with sharp optic disc margins, flat macula with normal foveal light reflex, normal vessels and peripheral retina

OS      Clear view, CDR 0.2 with sharp optic disc margins, flat macula with normal foveal light reflex, normal vessels and peripheral retina

No preauricular or submandibular lymph node enlargement

  • What are the patient counselling points in these case?
  • What is not a typical exam finding of conjunctivitis?

Go to Case Study-31 Answers

Case Study - 32 (Conjunctivitis)

History: A 35 year old female presented with complaints of sticky eyelids, watery and green ocular discharge, redness, soreness and slightly blurred vision in both eyes.

The symptoms started 2 weeks ago; the right eye was affected first.

Initially prescribed with Chloramphenicol (0.5%), no improvement was reported.

Chloramphenicol was replaced with Tobramycin, improvement was noticed only on 1 st day.

·         Ocular swab taken for PCR and was advised to stop tobramycin.

Again, on the visit, she reported that the left eye now felt worse and noticed an increase in green discharge, however a decrease in redness since the drop discontinuation.

Past medical history : A metal foreign body removal from the right eye – 10years ago.

She had flu, 3 weeks ago but is in good health now (was not atopic and she was not taking any medications.)

Vision: OD :    6/6                                                      OS :     6/5

Slit lamp examination:

OD: Mild conjunctival injection. Mild follicular change of the inferior palpebral conjunctiva. No corneal surface involvement. No anterior chamber inflammation.

OS: 360 degree conjunctival injection grade 3. Multiple white nodules at the limbus. Mild follicular change of the inferior palpebral conjunctiva. No anterior chamber inflammation.

Other Investigation:

PCR test : negative for Adenovirus, Varicella-Zoster virus, Herpes Simplex virus and Chlamydia trachomatis.

Provisional Diagnosis: Bacterial Conjunctivitis

For left eye:

Instill one drop of ofloxacin                     q2h

* reduce the dosage to “qid” the next day if the condition improves.

For right eye:

Ofloxacin                                                      qid     

Revisit after 1 week.

  •   What are the possible differential diagnosis in this case?
  •   What is the role of Ofloxacin in this Patient’s case?
  •   Why Tobramycin wasn’t readvised after the PCR result, is there any evidence suggestive for the change?

 Go to Case Study -32 Answers

Case Study - 33 ( Pediatrics)

Leanne is a 6-year-old girl whose teachers have suggested that her parents take to her GP. They have noticed that she seems to have problems listening and to be daydreaming a lot in class. The GP asks whether her parents have also seen her daydream. Her mother has, but has not thought much about it. However, more recently, it seems to have been happening more frequently. On direct questioning by the GP, Leanne’s mother thinks that these daydreams or ‘trances’ as she calls them sometimes occur when Leanne is in the middle of doing or saying something, and they interrupt her activity. Leanne’s birth and early medical history, including her development, have been normal. There was a history of epilepsy on her father’s side of the family. Her younger brother and older sister are well.

1.What should Leanne’s GP consider as a possible diagnosis?

2. What further investigations should the paediatrician or paediatric neurologist Request, and what considerations should be taken into account?

3. What should the discussion around medication include, and what medications may be Prescribed?

Go to Case Study- 33 Answers

medical case study questions and answers

Answers for Case Studies for Pharm.D / PharmD students:

Case study-1 answers.

  • According to AHA guidelines the target BP goal for hypertensive patients with CAD is <130/80 mmHg.
  • Beta blockers, ACE inhibitors, ARB’s, thiazide diuretics, and calcium channel blockers (CCB). ( Beta – blocker , first line agent to treat HTN in patients with Angina).

Explanation : stepwise                                 Go to CASE STUDY 1

Step 1:  In HTN patients with angina as comorbidity Beta blockers are the first line agents. Here metoprolol ( beta- blocker is already prescribed). Check whether target BP goal is achieved or not. Here target BP goal is not achieved in this patient (noted as 166/93 mmHg on his today’s visit to clinic). Follow step-2 if target Bp goal is not achieved. Step-2 :  Prescribe the maximum dose of metoprolol (100 mg twice daily). In this case already maximum dose is prescribed. Follow step 3 if target BP goal is not achieved. Step-3 :  Add another anti-hypertensive agent from ACEI’s / ARB’s / Thiazide diuretics/ CCB’s to the Beta -blocker. Note :  Here the patient is tolerant to METOPROLOL , however if the patient is intolerant to METOPROLOL ,Consider ACEi/ARB as first line agent and add CCB/thiazide as adjuvant.

       Go to CASE STUDY 1  

Case Study-2 Answers

Poor diet control, non-compliance to medication and obesity were considered to be the main reasons for poorly controlled hypertension and diabetes in Mr. MK

  • HbA1C target value should be less than 6.5%
  • Target value for BP should be 130/80 mm Hg.
  • Achieving these targets found to be beneficial in decreasing the Cardiovascular morbidities and prevention or delay of CKD progression.

3.Ans)   FOR HTN:                                                         Go to  Case Study 2.

For diabetic patients whose blood pressure levels are ≥160/100 start with two anti-hypertensive agents. Diabetic and hypertensive Patients with albuminuria consider ACEI /ARB as first line agent and add adjunctive drug from classes calcium channel blockers or Diuretic. Eg : Telmisartan (ARB) + Furosemide (diuretic)

FOR DIABETES: Metformin is the preferred initial pharmacologic agent for the treatment of type 2 diabetes. Once initiated, metformin should be continued as long as it is tolerated and not contraindicated; If the A1c levels are not achieved within the target levels, consider other class of agents to metformin. LIRAGLUTIDE (GLP1 AGONISTS) has the benefit of decreasing both the HBA1c values as well as body weight. So replacing GLICLAZIDE with LIRAGLUTIDE increases the effectiveness of treatment. Rx : Metformin + LIRAGLUTIDE .

4. Ans) DRUG INTERACTION:

AMLODIPINE+METFORMIN  – Amlodipine decreases effect of Metformin by pharmacodynamic antagonism. Management  – Monitor closely after withdrawal of Amlodipine whether the blood sugar levels are decreased or any hypoglycemic condition is observed.

5.Ans) Patient counselling:

  • Counsel the patient regarding importance of medication adherence.
  • Suggest to maintain regular exercise or at least do walking for 1 hour as it will be more beneficial for the weight loss.
  • Follow  DASH  diet to control HTN and reduce cardiovascular comorbidities.
  • Reduce intake of saturated fat and trans-fat; increase of dietary n-3 fatty acids, viscous fiber, and plant stanols/sterols.
  • Counsel the patient for regular checkup of A1c levels at least four times yearly.

Considerations:                                         Go to  Case Study 2.

  • LIRAGLUTIDE -1.2MG SC as GLP 1 agonists mainly decrease the HbA1c value (~0.8% -1.6%)and it also decreases body weight to (1-3kgs). Monitor closely while using liraglutide as it is contraindicated in medullary thyroid carcinoma and Monitor the calcitonin levels .
  • Metformin is contraindicated in patients with an eGFR ,30 mL/min/1.73 m2;
  • eGFR should be monitored while taking metformin;
  • The benefits and risks of continuing treatment should be reassessed when eGFR falls ,45 mL/min/1.73 m2;
  • Metformin should not be initiated for patients with an eGFR ,45 mL/min/1.73 m2; and
  • Metformin should be temporarily discontinued at the time of or before iodinated contrast imaging procedures in patients with eGFR 30 60 mL/min/1.73m2

Within these constraints, metformin should be considered the first-line treatment for all patients with type 2 diabetes, including those with CKD.

  • If  cost of medication  was the reason for  non-compliance  replace Liraglutide with insulin therapy basal insulin with lowest acquisition cost. If replaced with insulin there must be strict diet control and exercise that helps the patient to loose weight.
  • If Furosemide was given as an adjunctive drug advice the patient to observe closely for hypoglycemia, if needed switch to other anti-hypertensive agent.

Go to  Case Study 2.  | For more  CASE STUDIES .

Case Study-5 Answers

1.   what is the treatment goal and strategy for this case.

Ans :  Goals of therapy of CSA are :

  • Amelioration of anginal symptoms and improved angina-free exertion capability
  • Prevention or reduction of subsequent acute MI, UA, or ISD and there by increasing quality of life.

1.  Vasodilating – Beta-blockers as initial therapy:

Vaso-dilating beta blockers like Carvedilol and nebivolol can be used as first line agents, as they don’t have negative metabolic effects as compared to non-vasodilating beta-blockers like metoprolol.

Usually non-vasodilating  beta-blockers like metoprolol, propranolol etc., have high risk for new onset diabetes or masking of hypoglycemia.

{ Usually beta-blockers are preferred as initial therapy in the absence of any contraindications or chances for severe side effects.

In case of contraindications to beta blockers → use long acting calcium channel blockers  as initial therapy. }

2.  Lipid lowering agents  :

Lipid lowering agents like atorvastatin can be used as prophylactic therapy to prevent or minimizing risk of CAD.

3. Anti-platelet agents :

Adding  Aspirin  to the treatment shows a good evidence of Preventing MI and Death and Reducing Symptoms.

2. Suggest the best follow-up for this case.

Ans:     Monitoring of Symptoms and Antianginal Therapy:

During the first year of therapy, evaluations every 4 to 6 months are recommended. After the first year of therapy, annual evaluations are recommended if the patient is stable and reliable enough to call or make an appointment when anginal symptoms become worse or other symptoms occur.

  • Has the patient decreased the level of physical activity since the last visit?
  • Have the patient’s anginal symptoms increased in frequency and become more severe since the last visit? If the symptoms have worsened or the patient has decreased physical activity to avoid precipitating angina, then he or she should be evaluated and treated according to either the unstable angina or chronic stable angina guidelines, as appropriate.
  • How well is the patient tolerating therapy?
  • How successful has the patient been in reducing modifiable risk factors and improving knowledge about ischemic heart disease?
  • Has the patient developed any new comorbid illnesses or has the severity or treatment of known comorbid illnesses worsened the patient’s angina.

3.What are the conditions which worsens the symptoms of angina (in general)?

  • Tobacco use,
  • high blood pressure,
  • high cholesterol,

Go to  Case 5  | Explore more  Case Studies  | Go to  guidelines

Case Study-6 Answers

1 .  what information and counselling points would you include.

Ans:  Mr SW  has already been taking steps  to address his unhealthy lifestyle and reduce his cardiovascular risk.

  • Furthermore, he can be counselled for risk factors for cardiovascular disease that are commonly distinguished as modifiable and non-modifiable. Modifiable risk factors are those that can be controlled, treated or modified. These include  smoking, diet  (especially cholesterol and lipids),  weight and obesity, physical exercise, blood sugar levels and hypertension .

Non-modifiable risk factors are those that cannot be changed, such as age, gender, ethnicity and family history.

  • However, Mr SW should be counselled about how primary and secondary prevention helps in delaying the progression of disease by early detection of any further onset of disease .
  • He should be given proper information about the side effects (like muscle spasm, headache, flushing, breathlessness on physical work) & use, storage of medications; and how the benefits of treatment are much more as compared to side effects. So that, Mr SW will understand the rational use of medication and develop better medication adherence.
  • Also, if Mr SW founds any side effects that he is concerned about ( like impotence in previous ), rather quitting the drug by himself, consult his physician for the same. Drugs can be changed/further treatment option will be suggested based on the situation, the physician finds.

2. How is stable Angina managed ?                          Go to CASE STUDY-6

Ans: Managing stable Angina:

Treatments can be divided into those that reduce the risk of future cardiovascular events, thereby reducing mortality ( secondary prevention ) and those that prevent symptoms ( which can be further subdivided into short- and   long-term relief ).

For Secondary Prevention:

Aspirin  75 mg daily   (reduction in non-fatal MI & vascular events)

Simvastatin  40 mg daily  (starting dose)

  • Alternative / lower dose can be used if contraindication / interactions found.
  • If total cholesterol <4 mmol/L or LDL <2 mmol/L not achieved at initial dose, titration of simvastatin or alternative should be used.

For Short-Acting relief:

GTN      sublingually  (rapid onset of action: within 1-5 min)                                            Go to CASE STUDY-6

  • Minimizes the discomfort
  • Choice of dosage form should be discussed with patient
  • Pt should be counselled for side effects, use & storage
  • Can cause difficulties in Pt with significant arthritis or reduced hand dexterity

For Long-Acting symptom control:

  • 1 st  line treatment should be with either beta-blocker or CCBs.
  • If symptoms are not controlled, the next step is to swap or add the other (usually avoiding the combination of  beta – blocker & verapamil ).
  • Long-acting nitrate
  • Response to treatment should be reviewed 2-4 weeks after starting or changing drug treatment.               Go to CASE STUDY-6

3. What options are there, if Mr. SW experiences further symptoms despite the use of amlodipine?

Ans :   If further symptoms are experienced, i.e., symptoms not adequately

controlled with medication, the relative merits and risks of coronary artery bypass grafts (CABG) versus percutaneous coronary intervention to alleviate symptoms should be discussed. A multidisciplinary discussion should take place when the coronary artery disease is more complex.

REFERENCES:

1. European Society of Cardiology (2006). Guidelines on the management of stable Angina Pectoris.

2. national institute for health and clinical excellence (2011). management of stable angina. clinical guideline 126. london.

Go to CASE STUDY-6

Explore more  Case Study  |  Guidelines  |  Home

Case Study-7 Answers

Ans. the fourth universal definition of mi (udmi) was released in 2018. according to the udmi, mi is defined as myocardial injury in the clinical setting of myocardial ischemia. there are two components:.

  • Myocardial injury which is defined as raise of troponins above the 99th percentile
  • Clinical Setting of myocardial ischemia-symptoms and signs of myocardial ischemia, ECG changes with new ischemic changes or pathological Q waves, imaging evidence like loss of viability, regional wall motion abnormality (RWMA), thrombus in angiography, evidence of thrombus in autopsy, sudden cardiac death.

2. What are the classical ECG criteria for diagnosing STEMI ? Go to Case Study-7  

Ans.  There should be two contiguous leads with ST elevation (measured at J point).

>2.5 mm for men <40 years

>2 mm for men >40 years           in leads V2, V3

>1.5 mm for women.

And/or       >1 mm in other leads in absence of LVH, LBBB.

For posterior MI, in leads v7–v9, 0.5 mm is itself enough to diagnose posterior wall MI in inferior MI.

3. What are the different types of MI ? Go to Case Study-7  

Ans.  UDMI classifies MI in five types:

a) Type 1 MI due to thrombosis of an atherosclerotic plaque

b) Type 2 MI due to myocardial oxygen supply demand imbalance in the context of another acute illness.

c) Type 3 MI presenting as sudden death

d) Type 4 post-percutaneous coronary intervention (PCI)

e) Type 5 postcoronary artery bypass grafting.

4. Justify the treatment given to this patient.  Go to Case Study-7  

  • Aspirin  – ISIS 2 was the landmark trial in 1988 after which aspirin was considered as mainstay in ACS patients
  • DAPT  – benefit of DAPT in ACS setting and post-PCI setting were shown in CURE and PCI CURE trials, where they used Clopidogrel as the p2y12 inhibitor of choice. There was a significant reduction in composite primary endpoint of cardiovascular (CVS) mortality, non-fatal MI and stroke (9.3vs. 11.4%) with number needed to treat (NNT=48)

Ticagrelor was compared to clopidogrel in PLATO trial. Addition of ticagrelor to aspirin was able to further reduce composite primary endpoint of cardiovascular mortality, non-fatal MI, and stroke from 11.7% to 9.8% (P < 0.001) with an insignificant increase in major bleeding (11.6% vs. 11.2%)

  • In this case BMI of the patient was found to be 29.2 which is overweight. This puts patient at high risk for increased LDL, Cardiovascular comorbidities etc.

Statins  have a huge body of evidence both for early initiation and intensive treatment strategy mainly from meta-analysis data by cholesterol treatment trialist. Most of the trials utilized atorvastatin in the STEMI setting. With high intensity statin defined as the atorvastatin 40-80 mg with an ability to decrease LDL cholesterol by >50

  • Beta blockers  found to have robust benefit in people with MI and post MI reduced LVEF.
  • ACE inhibitor  trials are the SAVE, AIRE and TRACE trials which demonstrated mortality benefit of starting ACE inhibitors within first 24 h. AIRE trial used the drug ramipril.

#   Discontinue  Lasix + Spironolactone  :

Early initiation of Lasix and Spironolactone was not was not found to beneficial in Acute Myocardial infarction. ( As MI is one of the main cause for reduced ejection fraction or systolic heart failure consider using Mineralocorticoid receptor antagonists when reduced ejection fraction is less than 40% ).

(These can be used if the patient is having post-MI heart failure or LV dysfunction or Ejection fraction < 40% or diabetic with symptoms of heart failure.)

5. Determine the duration of DAPT In this patient ? Go to Case Study-7  

Ans.  In this case patient has had an acute coronary event (STEMI). He is a 56-year-old male, with only smoking as risk factor. He does not have any features of high bleeding risk. He is a candidate with a Low Bleeding risk with a moderate ischemic risk. PRECISE DAPT SCORE in this patient is <25. Hence, strategy for DAPT duration will be till 12 months with Aspirin 75 mg dose and ticagrelor 90 mg BD dose.  Reassess the patient at the end of 12 months , with DAPT score. If the patient is found to have high ischemic risk features, prefer continuing the patient on ticagrelor 60 mg BD till 3 years’ duration if cost is not an issue for greater benefit .

( Refer : high bleeding risk criteria and Precise DAPT scoring)

Go to Case Study-7  

Explore more  Clinical Case Studies  |  Guidelines  |  Home

Case Study-8 Answers

1. is the diagnosis & treatment given to mr tr justified explain.  case 8  .

Ans:  The diagnosis and treatment given to patient is  appropriate .

             Justification:

Diagnosis  :  The most likely diagnosis was confirmed by angiogram, which identified thrombosis and stenosis in the coronary arteries obtuse marginal, LAD& RCA, requiring deployment of drug-eluting stents. Also, Echocardiogram shows normal LV function.

Treatment  :

  • Drug eluting stents(DES) – for stenosed coronary artery to restore blood flow.
  • Dual antiplatelet therapy (Aspirin+Clopidogrel) – required as DES is inserted, until re-endothelisation occurs. However, treatment courses of 2 nd  antiplatelet agent(clopidogrel) should be kept to a minimum(1-3 months or 1 year)
  • Beta blocker & ACEI started with minimum required dose which is required.
  • High-intensity statins are recommended after an acute coronary syndrome.
  • GTN helps in venodilatation and arterial dilatation & reduction of cardiac ischaemia.

***Major and intermediate drugs interactions are there. However, drugs dosage is already  maintained minimum as recommended , if  used together .

2. Is there a need of counselling?  Case 8  

Ans:  Approx. 50% or more cardiovascular patients do not take their medications as intended. Poor adherence can lead to increase in recurrence, hospitalization, mortality. So, counselling is required to increase positive adherence.

Pt should be informed of drugs effects and side effects and the benefits of medication adherence.

Also, he should be counselled for lifestyle modification.

The patient should be encouraged to report any adverse effects from the drugs.

**He may have to take Aspirin lifelong to prevent secondary cardiovascular events.

Case 8   |  Clinical Case Studies  |  Guidelines  |  Home

Case Study-9 Answers

  • As the patient is at High risk with known CVD, the target goal for LDL-C would be < 70 mg/dl.
  • High-intensity statins (Atorvastatin 40-80 mg , Rosuvastatin 20 – 40 mg ) would be choice of treatment in this patient.
  • If on maximal statin & LDL-C ≥70 mg/dL (≥1.8 mmol/L), add ezetimibe.
  • Using non-statin therapy like niacin would be helpful in the patients who are intolerant to statins. (prior to withdrawing statin therapy check whether the patient is having true intolerance to statin or not).

References :

  • https://www.acc.org/~/media/Non-Clinical/Files-PDFs-Excel-MS-Word-etc/Guidelines/2018/Guidelines-Made-Simple-Tool-2018-Cholesterol.pdf
  • https://www.lipid.org/sites/default/files/6-_lipid_u-_guidelines-_jones.pdf

Case 9 | Clinical Case Studies | Guidelines | Home

Case Study-10 Answers

1. what is the metabolic abnormality present  case 10.

Ans: The obvious abnormal investigation is a very high serum cholesterol with high

LDL and low HDL levels. He has many clinical features to go with the high cholesterol and premature vascular disease. The patient has familial hypercholesterolaemia . He has presented with premature coronary artery disease. His absent pedal pulses suggest peripheral vascular disease.

** The metabolic defect is a result of a reduced number of high-affinity cell-surface LDL receptors. This leads to increased LDL levels. Increased uptake of LDL by macrophage scavenger receptors leads to increased oxidized LDL, which is particularly atherogenic. Triglyceride and VLDL levels are normal or mildly elevated. HDL levels are low. The other major causes of hypercholesterolaemia are familial combined hyperlipidaemia and polygenic hypercholesterolaemia. Familial combined hyperlipidaemia differs from familial hypercholesterolaemia by patients having raised triglycerides. Patients with polygenic hypercholesterolaemia have a similar lipid profile to familial hypercholesterolaemia but they do not develop xanthomata.

2. Discuss the Patient counselling for this case?  Case 10

Ans: The patient is at extremely high risk for further vascular events and especially

occlusion of his coronary artery bypass grafts. His risk depends on the combination of his risk factors, and all of these need attentions.

He should be counselled about following points:

  • To stop smoking,
  • Reduce his alcohol intake
  • Take more exercise and
  • Eat a strict low-cholesterol diet.
  • He should be suggested that diet alone will not control this level of cholesterol. So, a proper adherence with the pharmacological treatment with a statin or combined treatment for this level of hyperlipidaemia, is needed.
  • Importance of periodic lab check-ups.
  • His children should have their lipid profile measured so that they can be treated to prevent premature coronary artery disease.

Case 10 | Clinical Case Studies | Guidelines | Home  

Case Study-11 Answers

1. what would be the possible differential diagnosis  case 11  .

Ans:  The differential diagnosis of a pediatric patient who presents with a  narrow complex tachycardia  includes atrioventricular reentrant tachycardia  (AVRT ), AV nodal reentrant tachycardia ( AVNRT ),  sinus tachycardia ,  atrial flutter ,  atrial fibrillation ,  junctional ectopic tachycardia ,  atrial ectopic tachycardia , and  multifocal atrial tachycardia/chaotic atrial tachycardia .

Provisional Diagnosis: SVT ( supraventricular tachycardia)

2. What is the treatment goal and strategy for this case?   Case 11  

Ans: Treatment goals include:

  • Reducing the symptoms and stabilizing the patient,
  • Terminating the SVT and
  • Stablishing a mechanism to prevent any cardiac emergency condition.

Treatment Strategy:

If the patient is clinically stable , vagal maneuvers may be initially attempted to convert the tachycardia. Such vagal maneuvers may include bearing down (as though having a bowel movement (i.e., Valsalva maneuver)), blowing in a straw or inducing the diving reflex using an ice bag to the face for infants. Other vagal maneuvers such as eyeball pressure and unilateral carotid massage are harmful and should not be performed.

If the patient appears clinically unstable , urgent electrical cardioversion is indicated using 0.5-1 J/kg. If an intravenous line is already in place (antecubital preferred over a hand vein), an  IV bolus of 0.1-0.2 mg/kg adenosine  may be given prior to cardioversion. Adenosine causes a transient AV block and sinus bradycardia, thus interrupting the reentrant circuit involving the AV node and accessory pathway. It must be remembered that this medication is metabolized by the red blood cells and has a very short half-life (approximately 5 seconds), therefore it should be administered via bolus injection followed by an immediate bolus of saline (rapid push and flush). A 12-lead ECG should be obtained before and after conversion, if possible, and a rhythm strip should be continuously run during attempted conversion.  External pacing equipment should be available since some patients go into sinus arrest following administration of adenosine .

If adenosine initially fails to convert the SVT , but the patient is hemodynamically stable, they may be started on a medication such as  propranolol, digoxin or verapamil  (digoxin should be avoided in WPW, verapamil should be avoided in infants) followed by a repeat dose of adenosine. 

3. What medicine used to treat Supraventricular tachycardia(SVT) is contraindicated specifically in Wolff-Parkinson-White syndrome(WPW)?  Case 11  

Ans:    Digoxin .

** In the setting of WPW, digoxin can facilitate impulse conduction via accessory pathway and increase risk for ventricular arrhythmias (i.e., ventricular fibrillation).

Case 11  |  Clinical case Studies  |  Guidelines  |  Home 

Case Study-12 Answers

1. acute management of af with hemodynamic instability     case 12.

Given his presentation of AF with hypotension and pulmonary edema, considering AF with hemodynamic instability, sinus rhythm must be rapidly restored by synchronised electrical cardioversion. After achieving normal sinus rhythm, consider anti-coagulation as the patient is at risk for thromboembolism. Anticoagulation must be initiated and should be continued for at least 4 weeks.

(The decision to continue anticoagulation beyond the initial 4 weeks is based on the long-term risk for thromboembolism associated with nonvalvular AF. This risk is estimated by determining his CHA 2 DS 2 -VASc score, which is calculated by assigning points for congestive heart failure (1 point), hypertension (1 point), age (1 point for 65-74 years and 2 points for >75), diabetes (1 point), stroke, TIA, or thromboembolism (2 points), vascular disease defined as history of myocardial infarction, peripheral vascular disease, or aortic atherosclerosis (1 point), and female gender (1 point) )

2. CHA 2 DS 2 -VASc score (estimating risk of stroke & thromboembolism):    3 points       Case 12

Stroke risk was 3.2% per year in >90000 patients ( the Swedish atrial fibrillation cohort study) and 4.6%risk of stroke/TIA/systemic embolism.

  HAS-BLED score ( for estimating major bleeding risk) : 2 points

Risk was 4.1% in one validation study (Lip 2011) and 1.88 bleeds per 100 patient-years in another validation study      (Pisters 2010).Anticoagulation can be considered, however patient does have moderate risk for major bleeding (~2/100 patient-years).

Case 12 | Clinical Case Studies | Guidelines | Home

Case Study-13 Answers

1. based on current evidence, which of the following would be the most appropriate    recommendation regarding his asthma medication regimen   case study-13.

A. Maintain current medication regimen; no adjustment is indicated

B. Discontinue the inhaled corticosteroid; maintain on an inhaled beta-agonist as needed

C. Decrease the inhaled corticosteroid to 1 puff daily

D. Discontinue the inhaled corticosteroid; start a leukotriene modifier at bedtime

E. Discontinue the inhaled corticosteroid; start low-dose inhaled corticosteroid/long-acting beta-agonist, 1 inhalation at bedtime

ANS:  correct answer = option E

EXPLANATION : The goal of asthma therapy is to minimize risk and maintain asthma control with the least amount of medication (1).  In patients with mild persistent asthma, recent studies have demonstrated several options for “step-down therapy.”  The American Lung Association Asthma Clinical Research Centers network study found that patients who stepped down from twice daily low-dose fluticasone to once daily combination therapy with fluticasone/salmeterol had equivalent asthma control scores, FEV 1 , and frequency of exacerbations compared with continued therapy with twice daily fluticasone (2).  Once-daily montelukast demonstrated a slightly higher treatment failure compared with either of the regimens containing inhaled steroids.  Despite the slight increase in treatment failure with montelukast, each of the treatment groups had equivalent symptom-free days and rates of clinically significant asthma exacerbations.  Thus, while either regimen would be appropriate, stepping down to once-daily combination therapy with fluticasone/salmeterol appears to be more beneficial.

Recent studies also suggest that those with mild persistent asthma taking inhaled corticosteroids in combination with either a long-acting beta-agonist or a short-acting beta-agonist when symptomatic, had no increase in adverse outcomes compared with those taking scheduled daily inhaled doses.  Boushey et al. (3) compared patients with mild persistent asthma using twice-daily budesonide versus twice-daily zafirlukast verses placebo. All three groups used budesonide as-needed following a symptom-based action plan. The study found that in comparison with patients on a daily controller (budesonide or zafikulast), participants using only as-needed budesonide had no significant difference in morning peak expiratory flow, postbronchodilator FEV 1 , quality of life, or frequency of asthma exacerbations. Results of this study raise the possibility of treating mild persistent asthmatics with as-needed inhaled corticosteroids. More recently, Papi et al. (4) found as-needed use of an inhaler containing both beclomethasone and albuterol for symptom relief was associated with fewer exacerbations and higher morning peak flow readings than using an inhaler with albuterol alone.  The morning peak flow readings in the as-needed combination beclomethasone/albuterol group was equivalent to those taking scheduled daily doses of beclomethasone alone, or scheduled daily doses of beclomethasone/albuterol combined.  The combination of an inhaled steroid and a short-acting beta-agonist in a single inhaler is not currently available in the United States.

In the mild persistent asthmatic there is now strong evidence to support multiple treatment approaches which provide good asthma control.  Matching the drug regimen with the patient’s preferences, lifestyle, comorbidities, and financial limitations will help ensure drug adherence and maintain asthma control.

2. Which of the following should be done routinely with each follow-up visit?   Case Study-13

ANS: correct answer = option D

EXPLANATION: Assessing inhaler technique at each office visit allows the provider an opportunity to assess compliance, reinforce proper use, and identify motor or physical limitations affecting technique (8).

When studied, only approximately 25% of patients are able to properly demonstrate use of a meter dose inhaler when asked.  The remaining 75% improved with specific instruction and practice which reinforces the need to incorporate proper inhaler use during the office visit (9,10).  The use of a spacer significantly improves accuracy and dose delivery, particularly in patients with poor coordination skills (9,10).

Assessing patient adherence is best approached with a non-judgmental attitude.  Adherence to inhaled corticosteroids is estimated at < 50% (11).  Causes of nonadherence are multifactorial but may be improved by providing asthma education, encouraging self management through use of an asthma action plan, and facilitating open communication (11).  Financial barriers often transcend all other efforts to improve adherence and must be taken into account when prescribing asthma therapy (11).

Methacholine challenge testing is useful to demonstrate airway hyperresponsiveness in those with normal spirometry and a suspicion of asthma, but is not recommended as a serial procedure.  Biomarkers for inflammation such as eosinophils or nitric oxide are being investigated in clinical trials but currently have no indication in routine asthma care (1).  Peak flow monitoring is useful for long-term home assessment of asthma control and medication response, but is not indicated for regular office assessment or diagnostic purposes (1).

3. What findings would suggest that the patient requires a step-up in asthma medication?   Case Study-13

A. Two or more night time awakenings per month due to Asthma.

B. Two or more interruptions in daytime activities per month due to asthma

ANS:  correct answer = option A

EXPLANATION: Inadequate asthma control and a need for step-up therapy is based on two or more daytime symptoms per week, two or more nighttime symptoms per month, interference with activities of daily living, use of short-acting beta-agonist > 2 days/week (excluding use for prevention of exercise-induced bronchospasm),or peak flow or FEV 1  <80% predicted/personal best (1). 

Asthma symptoms should be assessed at each office visit to determine asthma control.  Validated self-assessment tools such as the Asthma Control Test (ACT), Asthma Therapy Assessment Questionnaire (ATAQ), or Asthma Control Questionnaire (ACQ) can facilitate consistent measurement and documentation of asthma symptoms during office visits (1, 8).  All asthmatics are at risk for a severe asthma attack regardless of their asthma classification; therefore, providers are encouraged to teach patients to recognize symptoms of inadequate asthma control and provide them with specific instructions for adjusting their medications or seeking medical care (1)

4. The patient was provided with an asthma action plan to follow at home. Which component of the asthma action plan is considered the most critical element for improving asthma outcomes?   Case Study-13

ANS:  correct answer = option D

EXPLANATION : All of the elements are important components of an asthma action plan. However, Gibson and Powell (5) found a 40% reduction in hospital admissions and a 20% reduction in emergency room visits when the plan contained personalized instructions regarding the medications to add, criteria for adding the medication, duration of use, and when to seek medical help when patients are symptomatic. An asthma action plan serves as a patient guide for early recognition and treatment of an exacerbation. Treatment guidelines may be based on symptoms, peak flow readings, or both. When peak flow readings are used, personal-best readings were consistently associated with improved health outcomes compared with percentage-predicted readings (5).

5. How often is spirometry testing recommended if the previous readings are normal and the patient’s asthma is well controlled?   Case Study-13

EXPLANATION: Spirometry is a simple test that can be performed in-office and can be used to assist the provider in determining the degree of airway obstruction (1, 6).  There are no widely accepted data correlating frequency of spirometry with clinical outcomes in asthmatics, thus one must rely on expert opinion and individual patient needs.  Spirometry is recommended during the initial evaluation after treatment is initiated and the patient’s symptoms have stabilized during periods of progressive or prolonged loss of asthma control and at least every 1-2 years (1).

When spirometry is used to diagnose or confirm asthma, testing must include pre- and post-bronchodilator readings (1).  A change in FEV 1  of >200 ml  and  ≥ 12% from the baseline measure following the administration of a short-acting bronchodilator is indicative of significant airway reversibility which has been shown to correlate with airway inflammation (7).  

The Expert Panel (1) classifies asthma severity by FEV 1 , FEV 1 /FVC, short-acting beta-agonist use, or frequency of asthma symptoms.   Parameters are measured at baseline with asthma severity determined by the worse parameter, e.g., daily symptoms with normal FEV 1  is classified as moderate persistent asthma.  Correct identification of asthma severity guides the provider in choosing the appropriate type and amount of therapy

REFERENCES :

  • Expert Panel Report 3 (EPR 3). Guidelines for the Diagnosis and Management of Asthma. Bethesda, Md: National Institutes of Health; 2007. NIH Publication No. 08-4051.
  • The American Lung Association Asthma Clinical Research Centers. Randomized comparison of strategies for reducing treatment in mild persistent asthma. N Engl J Med 2007;356:2027-2039.
  • Boushey HA, Sorkness CA, King TS, et al. Daily versus as-needed corticosteroids for mild persistent asthma. N Engl J Med 2005;352:1519-1528.
  • Papi A, Giorgio GW, Maestrelli P, et al. Rescue use of beclomethasone and albuterol in a single inhaler for mild asthma. N Engl J Med 2007;356:2040-2052.
  • Gibson PG, Powell H. Written action plans for asthma: an evidence-based review of the key components. Thorax 2007;59:94-99.
  • Miller MR, Hankinson J, Brusasco V, et al. Series ATS/ERS Task Force: Standardization of lung function testing. Eur Respir J 2005;26:319-338.
  • Pellegrino R, Viegi G, Brusasco V, et al. Interpretative strategies for lung function tests. N Engl J Med 2005;26:948-968.
  • Global Initiative for Asthma. Pocket guide for asthma management and prevention. Bethesda, Md: National Institutes of Health; 2006.
  • Giraud V, Roche N. Misuse of corticosteroid metered-dose inhaler is associated with decreased asthma stability. Eur Respir J 2002;19(2):246-251.
  • Johnson DH, Robart P. Inhaler technique of outpatients in the home. Respir Care 2000;45(10):1182-1187.
  • Elliott RA. Poor adherence to anti-inflammatory medication in asthma reasons, challenges, and strategies for improved disease management. Dis Manage Health Outcomes 2006;14(4):223-233.Top of Form

Case Study-14 Answers

1. what emergency measures are indicated   case study- 14.

Ans: In this case patient demonstrates the destabilizing effects of a respiratory infection on asthma, and her mother’s comments demonstrate the common (and dangerous) phobia about overuse of bronchodilator or steroid inhalers. The patient has signs of imminent respiratory failure, including her refusal to lie down, her fear and her tachycardia, which cannot be attributed to her minimal treatment with albuterol.

#Critically important immediate steps are

  • to administer high flow oxygen and to start albuterol by nebulisation.
  • Adding ipratropium to the nebulized solution is recommended.
  • A corticosteroid (0.5 – 1.0 mg/kg of methylprednisolone) should be administered intravenously.

#Alert the intensive care unit, because a patient with severe bronchospasm who tires can slip into respiratory failure quickly, and intubation can be difficult.

2. How should her long-term management be altered?    Case Study- 14

Ans: Presuming this patient recovers, she needs adjustments to her therapy before discharge. The strongest predictor of severe attacks of asthma is their occurrence in the past. Thus, this patient therapy needs to be stepped up to a higher level, like a high dose inhaled corticosteroid in combination with a long acting beta agonist. Both the patient and her  parents need instructions on the importance of regular adherence to therapy, with reassurance that it can be stepped down to a lower dose of inhaled corticosteroid( although still in combination with long acting beta agonist) once her condition stabilizes.

They also need instruction on an action plan for managing severe symptoms. This can be as simple as advising that if the patient has a severe, frightening attack, she can take up to four puffs of albuterol every 15 minutes, but if the first treatment doesn’t bring significant relief, she should take next four puffs while on her way to an emergency department or urgent care clinic.

She should also be given a prescription for prednisone, with instructions to take 40-60 mg orally for severe attacks, but not to wait for it to take effect if she remains severely short of breath even after albuterol inhalations.

# Asthma is a chronic disease, and a good care requires close follow up and creation of a provider-patient partnership for optimal management. If she has had several previous exacerbations, she should be a candidate considered for monoclonal anti-IgE antibody therapy with omalizumab, which effectively reduces the rate of asthma exacerbations _ even those associated with viral respiratory infection.

Reference : Text book Bertram G. katzung Basic and clinical pharmacology 14 th edition page number 365.

Go to Case Study- 14

For more Case Studies | Guidelines

All subject, PharmD books in pdf free download

Case Study-15 Answers

1. what clinical features and risk factors of copd does glenda exhibit  what grade of severity does glenda’s copd fall into    case study -15.

Ans: Glenda presents with a clinical scenario suggestive of COPD based on her age, smoking history, weight, frequent chest infections and gradual worsening of respiratory symptoms – breathlessness on exertion, a reduction in exercise tolerance, chronic cough and regular sputum production. Working in a fabric factory is also one of the identified occupational risk factors for COPD. Glenda is considered to have moderate COPD based on her spirometry results and breathlessness score.

(refer NICE classification of severity of airflow obstruction)

2. What initial treatment would you recommend for glenda?     Case Study -15

Ans: Start a short acting beta agonist such as salbutamol or a short acting muscarinic antagonist, such as ipratropium bromide, to alleviate symptoms as required. Short acting bronchodilators should be used as needed; their onset of action ranges from approximately five minutes (beta2 agonists) to 30 minutes (muscarinic antagonists) and effects lasts for between 3 and 6 hours.

Refer: NICE clinical guideline for COPD

3. Glenda continues to report that her breathlessness is getting worse. Her medical research council dyspnea score is now four and in the last few days she has been producing more sputum than usual. Her sputum has turned a yellow green colour. What does these changes indicate & what treatment would you recommend?     Case Study -15

Ans: Glenda’s symptoms suggest that she is experiencing exacerbations of COPD. She must be prescribed an antibiotic for 5 days at a therapeutic dose with Oral prednisolone 30 mg every morning for 7-14 days.

(Systemic corticosteroids are beneficial in the management of exacerbations of COPD. They shorten recovery time and improving lung function (FEV10 and hypoxemia (PaO2).  Antibiotics should be used to treat exacerbations of COPD associated with a history of more purulent sputum. Initial empirical treatment should be amoxicillin, doxycycline or clarithromycin depending on a local resistance pattern.

Go to Case Study -15

Explore more Case Studies | Guidelines

Case Study-16 Answers

1. how the sign, symptoms & pathophysiology of copd, relates to the patient   case study-16.

Ans:   Clinical signs and symptoms of COPD, the pathophysiology and how this relates to the patient.

2. Comment on the current drug therapy and describe the role of O2 in this patient.     Case Study-16

Role of oxygen:

As this patient has a PaO2 of less than 7.3 kPa and oxygen saturation of arterial blood of less than 90%, she is eligible for Long-Term Oxygen Therapy (LTOT). She will need to use oxygen at least 15 hours a day and needs to be counselled on the importance of smoking cessation. Ambulatory and short-burst oxygen therapy should also be considered, as per NICE guidelines (NICE, 2004).

3. What are the social issues in treating this patient at home?    Case Study-16

Ans: Patients may be anxious and refuse such support. The cultural and social setting of the patient needs to be taken into account. As patients lose their mobility and increase dependence on others for help with day-to-day living, anxiety increases. Patients can become hesitant to seek help because of the perception that their condition was self-inflicted. Poor populations tend to have a higher risk of developing COPD, other factors include poor nutrition, crowding, exposure to pollutants, poor access to healthcare and early respiratory infections. Some evidence suggests women are more susceptible to COPD development than men. A multidisciplinary team should be involved in the support of the patient at home. Additional use can be made of nebulisers, compressors, oxygen, visiting respiratory nurses and increased social service input. The patient remains under care of the hospital but the GP is made aware of the extra support. Health status is better in home-treated patients. COPD is linked with other co-morbid conditions. Patients are more likely to have ischaemic heart disease, pneumonia and diabetes, making treatment more complicated and requiring a holistic approach to care.

This patient demonstrates five co-morbidities. These in turn impact on the medication load she has to cope with, so concordance is important.

Book: Pharmacy case studies by Rebekah     Raymond

Go to Case Study-16

Explore more Case Studie s | Guidelines

PharmD 1st year, 2nd year, 3rd year, 4th year books pdf free download

Case Study-17 Answers

1. Why should women of childbearing age be offered advice about pregnancy?  Case Study-17

Ans:  Mrs Jaya or other women like her should have been offered preconception advice prior to becoming pregnant because glucose control needs to be optimal to reduce the risks of miscarriage, congenital malformation, stillbirth and neonatal death associated with diabetes in early pregnancy. Preconception advice should also include information for the patient on how diabetes affects pregnancy and how pregnancy affects diabetes, what dietary supplements to take and advice on diabetes-related medicines that are unsafe to take during pregnancy.

2.  Was she taking appropriate dietary supplements prior to conception?     Case Study-17

Ans:  Mrs Jaya was taking the appropriate dietary supplement; however, the recommended dose for women with diabetes is 5 mg daily, rather than 400 μcg daily. The 5-mg strength tablets are available on prescription.

3.  What advice should she be given with respect to her regular medication?     Case Study-17

Ans:  Mrs Jaya should have been advised to stop her ramipril and simvastatin since both have been associated with an increased risk of birth defects.

Case Study - 18 Answers

You can get Case -18th answers with explanation by this link.

Go back to Case Study-18 

Case Study - 19 Answers

Answers :        (CORRECTED)

1.     What is the likely diagnosis? Main differential diagnosis?          Case Study-19

Ans:   Diagnosis: Hypothyroidism.

The differential diagnosis is extensive.

However, Main Differential Diagnosis : Depression and Hypothyroidism

** Fatigue is a very common symptom of both physical and mental illness. The differential diagnosis is extensive and includes cancer, depression, anaemia, renal failure and endocrine diseases. He has a past history of depression, but currently has no obvious triggers for a further episode of depression. He is not waking early in the morning or having difficulty getting to sleep, which are common biological symptoms of severe depression.

There are a number of clues in this case to the diagnosis of hypothyroidism . Insidious onset of fatigue, difficulty concentrating, increased somnolence, constipation and weight gain are features of hypothyroidism. As in this case there may be a family or past medical history of other autoimmune diseases such as type 1 diabetes mellitus, vitiligo or Addison’s disease. Hypothyroidism typically presents in the fifth or sixth decade, and is about five times more common in women than men. Obstructive sleep apnoea is associated with hypothyroidism and may contribute to daytime sleepiness and fatigue. On examination the facial appearances and bradycardia are consistent with the diagnosis

2.     How would you further manage this patient?                        Case Study-19

Ans:    For the management of the patient in this case, patient should be advised for Thyroid Function Test . Level of TSH, T ₃ , T ₄ will lead to decide the accurate therapy for him.

**However, a starting dose of levothyroxine 50-70 μ g/day will be sufficient. Clinical benefits begin in 3-5 days.

Response is measured clinically and biochemically by the return of TSH to the normal range.

*Elderly patients or those with coronary heart disease should be started cautiously on T4 because of the risk of precipitating myocardial ischaemia.

The most common cause of hypothyroidism is autoimmune thyroiditis and the patient should have thyroid autoantibodies assayed.

Patients should be advised to avoid or do heavy physical labor with caution (hypotonia may be there sometimes).

·         No specific diets are required for hypothyroidism . WHO recommends a daily dietary iodine intake of 150 μ g for adults.

CORRECTION :                    Case Study-19

1.      Starting Dose of Levothyroxine should be 1.6 – 1.8 mcg/kg/day of lean body mass.

2.       If started with levothyroxine, then, soyabean, walnuts, dietary fiber, calcium fortified juices should be avoided, if possible. When levothyroxine is given during continuous enteral nutrition for more than 7 days, the tube should be interrupted for at least one hour before and one hour after the dose of levothyroxine.  

Ref-2: Recommendations for the use of medications with continuous enteral nutrition , “ American journal of health-system pharmacy: AJHP: official journal of the American Society of Health-System Pharmacists ”  

Case Study - 20 Answers

1.     What is the likely diagnosis in this case?  & Are the Lab finding s clinically justified?    Case Study- 20

Ans: Diagnosis: Hypothyroidism with Anemia.

           !  Ix Hashimoto Thyroiditis .

Yes , the lab findings are clinically justified.

·         High TSH & low levels of T4 suggestive of Hypothyroidism.

·         Stool occult blood positive: Reason to justify blood loss, decrease in levels of RBC, Haemglobin.

·         High RDW with normal MCV indication for anemia further evalualtion needed for iron/B12/FA deficiency/anemia of chronic disease.

·         Decrease in reticulocyte:(borderline to decrease) indication for iron deficiency/anemia of chronic disease.

·         USG: indicates Thyroditis further lab values needs for antibody testing(thyroxine peroxidase).

2.     What are the Pharmacist Intervention points in this case?          Case Study- 20

Ans:   Overdose: According to TSH value,  Tab Thyronorm 75 mcg should have been  started instead of Tab Thyronorm 25mg on the basis of wt. of patient (1.6-1.8mcg/kg/day).

Inappropriate Dose:  There is no electrolytic imbalance seen therefore, isotonic (0.90% NS) should have been administered.

Untreated Indication: No medication for headache has been prescribed (antipyretic SOS can be prescribed)

Drug- Drug Interaction: Moderate Interaction

( Pantoprazole : Iron supplements ), ( Levothyroxine: Pantoprazole ), [ Levothyroxin: Iron Supplements ( Theoritical )]

Drug-Food Interaction: Is seen with soyabean products

3.     Patient counselling regarding drug & disease?          Case Study- 20

Ans:    Disease: It is a disorder in which under activity of Thyroid glands occur(in local language).

Signs & Symptoms : related to slow metabolism (functioning) of the body .

·         Fatigueness, dry skin, constipation, feeling cold, slow heart rate, weight gain

     *Avoid soyabean & its products; cruciferous vegetables.

·         Don’t chew the tablet

·         Don’t split the tablet

·         Tightly place the cap of container after use

·         In the case of missed dose, Don’t take double dose; If it is time for next dose, take the single dose only.

·         Don’t take the dose immediately, when you recall anytime, after you missed the dose

·         If case, vomiting occurs immediately after taking the dose, consume it after sometime. If another episode occurs, consult your physician. Don’t stop taking the medication [this further may lead to untreated condition, resulting in myxedema (emergency condition)]

ü The Improvement of symptoms may not be evident for several weeks after the starting of the therapy. So, she should be counselled for proper medication adherence.

Drugs Counselling Points:

·         Take tab Thyronorm on an empty stomach (30min to 1 hour prior to breakfast) once daily.

·         Take tab Thyronorm 4 hours apart from Pantoprazole and Febac XT, Fdson MP Forte.

·         Take tab Fdson MP Forte once daily after lunch

·         Take one tab Febac XT after breakfast and another after dinner.

·         For Tab Fdson MP Forte & Febac XT, you may observe metallic taste in the mouth. Therefore, don’t stop taking the medication.

·         Take Tab Pantoprazole once a day 1 hour before lunch.

4.     Write the Generic Names of above advised drugs.      Case Study- 20

Ans:                                                                                         Generic

Pantoprazole                                                  Pantoprazole

            Optineuron                                                   Vit. B-Complex

            Fdson MP Forte                                          Folic Acid 5mg + Methylcobalamin

        1500mcg + Pyridoxine 20mg

            Febac XT                                                       Ferrous Ascorbat 100mg + Folic Acid

          1.5mg + Zinc Sulphate 22.5 mg

            Thyronorm                                                  Levothyroxine Sodium

Case Study - 21 Answers

Q1 . What is the likely diagnosis ?          Case Study-21

Ans: Diagnosis : Grave disease in pregnancy with impending thyroid storm and

        IUFD at 23-24 weeks gestation.

Q2. Was the Treatment given to the patient justified according to clinical condition?            Case Study-21

Ans: The dosing and the regimen advised to the patient is according to the prescribed guidelines and in the limit according to the lab reports.

***Methimazole 30mg BD,                     recommended 60-80mg/day (impending

thyroid storm cond. )

*** Lugol solution can be given 5 drops 2 times a day, but should not be more than 1 week.

*** The combination of propranolol 40 mg every 6 hours with iodide usually results in clinical improvement within 2 to 7 days.

Q3. Widely used anti-thyroid drugs in pregnant women with hyperthyroidism?              Case Study-21

Ans: Propylthiouracil (PTU) and methimazole

Q4. What should be the Patient Counselling points?                  Case Study-21

·         Pt should be instructed to report any new symptoms occurring.

·         She should be counselled about the necessity of medication adherence.

·         Next optimal time to conceive is once a euthyroid state is reached, should consult the practitioner for the same.

·         Pre- pregnancy counselling for all patients with hyperthyroidism or a history of hyperthyroidism is imperative and use of contraception until the disease is controlled.

·         Prior to conception, ablative therapy (radioiodine or surgery) or medical therapy may be offered when the thyroid gland is overactive.

·         May experience the previous symptoms of hyperthyroidism along with new symptoms, consult the practitioner

·         Avoid caffeine; stress reduction therapy should be suggested to relieve the symptoms of anxiety, nervousness, poor conc. May occur due to the miscarriage.

·         It is normal to women to experience some Vaginal bleeding (light, menstrual-like bleeding) for several weeks after an abortion.

·         Some pain is normal after an abortion, as the uterus is contracting.

·         In this case, she should be specially suggested of the recommended interval to next interval to next pregnancy is at least 6 months (checking the hyperthyroid condition)

·         She should be counselled with care and one to one interactions and the family members should be counselled to maintain a better social environment for her.

ADDITIONAL BASIC POINTS TO CONSIDER IN COUNSELLING:

·         If case, vomiting occurs immediately after taking the dose, consume it after sometime. If another episode occurs, consult your physician. Don’t stop taking the medication [this further may lead to untreated condition , resulting in myxedema (emergency condition)]

Case Study - 22 Answers

1.     Are there any drug-drug interactions?          Case Study – 22

Ans:    Major:

Diltiazem + Propranolol ::  Increases the toxicity of other by unspecified mechanism. Can increase the risk of bradycardia.

**In this case, alternative drugs also have the major/serious interaction (alternatively: closely monitor)

            Moderate:

            Propranolol + Furosemide :: Decreases serum potassium.

            Propranolol + Diltiazem  ::  Both increase anti-hypertensive channel blocking.

            ** Monitor Closely                              *Minor interactions are also there.

2.     Is the dosing of medication for hyperthyroidism, according to standard treatment guidelines?          Case Study – 22

Ans:    Dose of carbimazole in hyperthyroidism; depends on the

FT4 levels. If the levels are raised 2 times higher than the upper limit of normal values then the dose is : 10-20 mg. So, correct in this case.

Given in split doses as duration of action is less than 24 hours, Twice a day is acceptable.

If HR >90 BPM Beta adrenergic blocker are considered, Propranolol dose :10-40 mg 3-4 times / day

For atrial fibrillation: diltiazem IV 15-20 mg is considered.

Maintenance dose is: 5-10 mg Carbimazole

3.     What should be Patient counselling in this case, regarding drug & disease?                Case Study – 22

Ans:  Drugs:

·         If case, vomiting occurs immediately after taking the dose, consume it after sometime. If another episode occurs, consult your physician. Don’t stop taking the medication.

            Disease:

Hyperthyroidism

·         It is a disorder in which thyroid hormones level are increased. So, they may cause some alarming symptoms related to increase in the metabolism of the body such as wt loss, over active bowel movement, heat intolerance, tremors, palpitations, nervousness, fatigue , weakness.

·         Patient should be advised to follow medication adherence, Otherwise may increase the risk of cardiovascular problems.

·         Pt should be informed that improvement in the symptoms may be seen in 3-4 weeks.

·         A sightly elevated liver function test are commonly seen in some hyperthyroidism pts.

·         Patient should be informed of side effects of ATDs and the necessity of informing the physician promptly if they should develop pruritic rash, jaundice, acolic stools or dark urine, arthralgias, abdominal pain, nausea, fatigue, fever, or pharyngitis. Preferably, this information should be in writing.

·         Before starting ATDs and at each subsequent visit, the patient should be alerted to stop the medication immediately and call their physician if there are symptoms suggestive of agranulocytosis or hepatic injury.

·         Patient should be informed about the condition like, Sometimes, after apparently successful treatment, the condition may return, and further treatment may be needed.

Hemorrhoids/ Anal fissure

·         May be advised to sit over Hot water bag/ tub/brick not more than 15 min.

Note: As much as one can tolerate

·          Keep a check on your regular bowel movement, avoid any straining.

·          Pt should be advised to Eat less, oily less spicy easily digested food items

Case Study -23 Answers

1.     What should be the further investigation ?                          Case Study- 23

Ans:     Females with primary ovarian insufficiency-related estrogen deficiency are

at risk of osteopenia, osteoporosis, and fracture, especially if hypoestrogenism occurs early in life and before accrual of peak bone mass. Therefore, Dual-energy X-ray absorptiometry has been recommended for the evaluation of bone mineral density in women diagnosed with primary ovarian insufficiency.

Flow-mediated brachial artery diameter for endothelial dysfunction as there are cardiovascular risks .

[** The woman has evidence of hypogonadotrophic hypogonadism- she has low oestradiol

levels associated with low gonadotrophin stimulation from the anterior pituitary. This

may be due to various pituitary or hypothalamic causes, but in this case clearly relates to

anorexia nervosa and possibly excessive exercise . The raised prolactin is consistent with

stress and does not need to be investigated further. At a BMI below 18 kg/m2, menstru-

ation tends to cease, returning once the BMI increases again.]

-Furthermore, CBC, Liver & renal function test should be

  monitored.

-Cognition, Mood, and Psychosocial Functioning

-Vasomotor Symptoms and Quality of Life

-Also include USG abdomen if needed!

2.     How, this patient would be managed ?                Case Study- 23

Ans: The combined oral contraceptive pill should be prescribed, which will prevent osteoporosis and bring on periods. However, this anorexia and primary ovarian insufficiency condition can be refractory to treatment, albeit pharmacologically induced.

Recommended dose:

* One of the estrogen options to be combined with one of the progestogen options.

Also, vitamin supplementation and calcium should be started (if evident)

3.     What Patient counselling points should be included in this case?      Case Study- 23

Ans: Following Counselling points will help in effective recovering:

·         Encouraging the woman to eat a more normal diet and to avoid exercising is the ideal management.

·         Explanation that her periods will return if she increases her BMI may possibly encourage her to put on weight.

·         Proper counselling should also be given to her parents, to help in stress reduction.

·         She should be advised to consult dietician & nutritionist also, to maintain regular food habits with minimal or moderate physical exercise.

·         Combined oral contraceptive pills are to be taken daily at approximately the same time each day.

·         Avoid taking them greater than 24 hours apart as this could affect efficacy. 

·         When you initiate the contraceptive pills you are not protected from pregnancy prevention in the first 7 days and an alternative method of birth control is recommended during this time period (not required in this case)

·         If she, miss a tablet, just take the missed tablet as soon as she remembers and the next tablet at the usual time (taking 2 tablets in 1 day). 

·         If she, miss 2 tablets in a row in the first or second week, then, take 2 tablets the day she remembers and 2 tablets the next day, then resume 1 per day.

·         The most common adverse effect of combined oral contraceptive pills is break through bleeding. 

·         She may feel of nausea, headaches, abdominal cramping, breast tenderness, and an increase in vaginal discharge or decreased libido. 

·         Nausea can be avoided by taking the medication at night before sleep.

Ref: 1. https://www.acog.org/en/Clinical/Clinical%20Guidance/Committee%20Opinion/Articles/2017/05/Hormone%20Therapy%20in%20Primary%20Ovarian%20Insufficiency

2. Oral contraceptive pills, NCBI, National Library of Medicine,USA

Case Study -24 Answers

1.      What should be the further investigation ?                  Case Study – 24

Ans: Further recommended investigations are :

Lipid panel , 2 hr OGTT, questionnaire for Depression, USG Abdomen (transabdominal)

2.      How, this patient would be managed ?                      Case Study – 24

Ans : For acne: First line: hormonal contraception, and topical cream (benzoyl

         peroxide/ tretinoin/adapalene/ antibiotic cream like clindamycin.

            ** depending upon the staging nd grading of acne

Obesity: First line is Lifestyle Modifications

Insulin resistance: First line METFORMIN (Dose 1500-2250 mg twice daily )

Hirsutism: First line:  Hormonal contraception with or without anti androgen therapy

Menstrual irregularities : Clomiphene 50-100mg / day

3.      What Patient counselling points should be included in this case?                      Case Study – 24

·         Encouraging the woman to eat a more normal diet and to avoid leg exercising is the ideal management.

·         Explanation that how increased BMI can also affect the normal menstrual cycle.

·           For Acne: Wash the face twice daily with medicated face wash and can spill water during day time.

·         Adapalene is UV, light sensitive, to be used at night. And wash your face before stepping out. It should be instructed that cold temperatures or wind may also increase skin irritation during drug therapy.

·         Benzoyl peroxide : May cause dry skin and peeling, so avoid using higher concentration. Should be advised to use a test dose for over-the-counter (OTC) products due to potential hypersensitivity reactions. Apply small amount to skin for 3 days and if no discomfort occurs, use product as directed.

·         Clomiphene citrate: Drug may cause decreased visual acuity. Patient should be instructed to report visual symptoms, such as blurred vision. Advise patient to take drug exactly as ordered, as administration is timing-sensitive.

Case Study - 25 Answers

1.     What should be the further investigation for this patient ?              Case Study – 25

Ans : The following further investigation should be suggested for her:

                                    i.             Thyroid profile

                                  ii.              HbA1C test

                               iii.              Diurnal epinephrine test

2.     Give the provisional diagnosis for this case.                                             Case Study – 25

     Ans : Provisional Diagnosis: Menopausal syndrome

3.     How, this patient would be managed ?                                Case Study – 25

Ans : Following management plan is recommended for the patient:

Estradiol low dose transdermal patch is to be started with min possible dose

of : 0.025 mg with size 5 cm ² patch. (Duration : 3 weeks of therapy + 1 week off)

An oral progestin is also added for only 10-14 days every month this will promote sleep.

( Dose : 10mg dydrogesterone OR Micronised progesterone capsule 100mg Frequency : at night 1 tab)

For glycine and glutamate:  glutathione 20mg is recommended Or NAC(n-Acetyl cysteine) if needed

For PEA:  vit B6 is advised daily, at lunch time.. 1tab, MVBC most preferably

4.     What Patient counselling points should be included in this case?                            Case Study – 25

Ans:    About disease:

  •     Give information to menopausal women and their family members (as appropriate) that includes:
  •     That a change in their menstrual cycle they may experience a variety of symptoms associated with menopause, including:

·         Vasomotor symptoms (for example, hot flushes and sweats)

·         Musculoskeletal symptoms (for example, joint and muscle pain)

·         Effects on mood (for example, low mood)

·         Urogenital symptoms (for example, vaginal dryness)

·         Sexual difficulties (for example, low sexual desire)

·         Benefits and risks of treatments for menopausal symptoms

·         Long-term health implications of menopause

·          Hormone replacement therapy (HRT) aids for Psychological symptoms , Consider HRT to alleviate low mood that arises as a result of the menopause

·         Non-pharmaceutical, for example cognitive behavioral therapy (CBT),  Consider CBT to alleviate low mood or anxiety that arise as a result of the menopause

About drugs: (general points)

Transdermal patch:

Application : It is to be applied to non hairy skin, below waist on upper quadrant of abdomen.

Replace the patch in every 3-4 days using a different site.

·         Gradually reducing HRT may limit recurrence of symptoms in the short term

·         Gradually reducing or immediately stopping HRT makes no difference to their symptoms in the longer term

**Long-term benefits and risks of hormone replacement therapy–

·          Venous thromboembolism:The risk of venous thromboembolism (VTE) is increased by oral HRT compared with baseline population risk

·          The risk of VTE associated with HRT is greater for oral than transdermal preparations

·          Consider transdermal rather than oral HRT for menopausal women who are at increased risk of VTE, including those with a BMI over 30 kg/m ²

·          Consider referring menopausal women at high risk of VTE (for example, those with a strong family history of VTE or a hereditary thrombophilia) to a haematologist for assessment before considering HRT

Lifestyle changes:

·         She should be suggested to exercise regularly

·         She can take cold water bath as required.

·         She should be counselled to avoid stress and enjoy day to day activity.

·         Advise her to deep her feet in mild-salted, tolerable cold water, to ease her in hot flashes.

·         Consult your gynecologist and ask her about vaginal wash products to prevent dryness, if needed.

·         isoflavones or black cohosh may relieve vasomotor symptoms advice to consume soy products, nuts raisins, legumes, sesame, peanuts, Meat, pork, chicken, egg , banana.

Note: Ensure that menopausal women and healthcare professionals involved in their care understand that there is no clear evidence for SSRIs or SNRIs to ease low mood in menopausal women who have not been diagnosed with depression. (NICE Guidelines)

1.        Indian Menopause Society ( https://indianmenopausesociety.org/prescription-writing-module-for-hormone-therapy/ )

2.        Essentials of medical Pharmacology, Tripathi.KD, 6 th edition.

3.        https://www.acc.org/latest-in-cardiology/ten-points-to-remember/2020/02/10/12/13/hormone-therapy-for-postmenopausal-women

4.        https://dailymed.nlm.nih.gov/dailymed/lookup.cfm?setid=c714974b-766f-42f2-a846-b0c1f5a60560

5.        https://www.jmidlifehealth.org/article.asp?issn=0976-7800;year=2013;volume=4;issue=2;spage=77;epage=106;aulast=Meeta%2C

Case Study -26 Answers

1.      How should the ovarian hormones she lacks be replaced?            Case Study- 26 

Ans: The patient should be advised to start daily transdermal estradiol (100

mcg/day) along with oral natural progesterone (200mg/day) for the last 12 days of each 28 day cycle. On this regimen, her symptoms should disappear and normal monthly uterine bleeding resume.

2.      What extra measures should she take for her osteoporosis while receiving treatment?              Case Study- 26 

Ans: She should  also be advised to get adequate exercise and increase her calcium

and vitamin D intake as treatment for her osteoporosis.

Clinical evidence:

Non-pharmacological approaches–

·         A balanced diet, adequate calcium and vitamin D intake, weight-bearing exercise, maintaining a healthy body weight and cessation of smoking and moderation of alcohol intake are primary goals in reducing fracture risk.

·         Calcium is essential for bone health, and there is evidence that calcium supplementation in older women reduces the risk of fracture.

·         The recommended nutritional intake (RNI) for calcium is 1000mg/day, and for vitamin D 800 IU/day 

**Higher calcium intake during growth and early adulthood is associated with higher peak bone mass.

**Based on recent concerns of a potential association between calcium supplement use and increased risk of myocardial infarction, calcium supplements should not be prescribed when dietary calcium intake is adequate (1000 – 1200 mg/day).

Case Study - 27 Answers

1.      What all investigations are needed for the further management?

·         Her 10-year risk of fracture FRAX is the risk assessment tools validated for use

·         She has been taking multivitamin with calcium so

·         Consider assessing serum 25-OH-D

·         dual-energy X-ray absorptiometry (DXA) and repeat DXA every 1 to 2 years until findings are stable

·         Additional tests for Clinical Identification of Vertebral Fracture :

2.      How this patient would be managed ?

·         Counsel patients to maintain adequate dietary intake

of calcium, to a total intake (including diet plus supplement, if needed) of 1,200 mg/day for women age ≥50 years

·         Pharmacologic therapy is strongly recommended for Patients with osteopenia or low bone mass and a history Of fragility fracture of the hip or spine

·          Pharmacologic therapy is strongly recommended for Patients with a T-score of −2.5 or lower in the spine, femoral neck, total hip, or 1/3 radius

Calcium: 1200mg/day is recommended so as per the patients need either 500mg twice daily OR 1000mg once daily is given with food items rich in calcium.. 

Vit D3: If serum25[OH]D deficiency is seen then according to individualize patient therapy is given as per this case: vit D3 5000IU / day for 8-12 weeks is recommended…. With the maintenance therapy of 1000-2000 IU/day

3.      How will you do the Patient counselling  in this case?

·         Counsel patients to maintain adequate dietary intake of calcium rich products.

·         Counsel patients to avoid or stop smoking.

·          Counsel patients to maintain an active lifestyle, including weight-bearing, balance, and resistance exercises

·          Provide counseling on reducing risk of falls, particularly among the elderly.

·         Daily sun bath is advised.

·          Consider referral for physical therapy, which may reduce discomfort, prevent falls, and improve quality of life.

·          avoiding use of tobacco And excessive use of alcohol;

·         This “bone healthy” lifestyle is Important for everyone, not only patients with osteopenia And osteoporosis.

·         Weight-bearing exercise includes walking, jogging, Tai Chi, stair climbing, and dancing, among other activities.

·         Muscle-strengthening exercise includes weight training and other resistive exercises.

·         Before initiating an exercise program in an individual with osteoporosis, a clinician’s evaluation is recommended.

·         Physical therapy plays an important role in the effort to mitigate sarcopenia and reduce risk of falls.

·         Measures for Prevention of Falls

·         Anchor rugs

·         Minimize clutter

·         Remove loose wires

·         Use nonskid mats

·         Install handrails in bathrooms, halls, and long stairways

·         Light hallways, stairwells, and entrances

·         Encourage patient to wear sturdy, low-heeled shoes

4.      Mention the indications for BMD testing.

·         All women 65 years of age or older

·         All postmenopausal women

·         With a history of fracture(s) without major trauma

·         With osteopenia identified radiographically

·         Starting or taking long-term systemic glucocorticoid therapy (≥3 months)

·         Other perimenopausal or postmenopausal women with risk factors for Osteoporosis if willing to consider pharmacologic interventions

·         Low body weight (<127 lb or body mass index <20 kg/m2)

·         Long-term systemic glucocorticoid therapy (≥3 months)

·         Family history of osteoporotic fracture

·         Early menopause

·         Current smoking

·         Excessive consumption of alcohol

·         Secondary osteoporosis

Case Study-28 Answers

1. what is the significance of t- score            case study – 28.

• The T-score on your bone density report shows how much your bone mass differs from the bone mass of an average healthy adult.

• DEXA accomplishes with only one-tenth of the radiation exposure of a standard chest x-ray and is considered the gold standard for osteoporosis screening

• Standard X-rays may show weakened bones. But at the point when bone weakness can be seen on standard X-rays, it may be too far advanced to treat. Bone densitometry testing can find decreasing bone density and strength at a much earlier stage when treatment can be beneficial

• The World Health Organization has established the following classification system for bone density:

1. If your T-score is –1 or greater: your bone density is considered normal.

2. If your T-score is between –1 and –2.5: you have low bone density, known as osteopenia, but not osteoporosis.

3. If your T-score is –2.5 or less: you have osteoporosis, even if you haven’t yet broken a bone.

2. How this patient would be managed ?                Case Study – 28

• A daily calcium intake of 1000 mg for men under 70 years is advised(only of dietary calcium is not sufficient enough). So 500 mg twice daily once on morning and other at night is ideal.

• Vit D starting from 800 IU/day (once daily OR 400 IU twice daily)

3. How will you do the Patient counselling in this case?       Case Study – 28

• to not involve yourself in smoking again

• Quit drinking alcohol

• Regular exercise helps make your bones stronger. • physical activity is reviewed and prescribed by an exercise professional, such as a physiotherapist or exercise physiologist. This is because some activities like jumping, running and twisting can be hazardous to weaker bones, particularly if you have had a fracture. • Other forms of exercise such as strength training can actually benefit the bones

• Foods high in calcium include milk products, leafy green vegetables, sardines, salmon, tofu, and almonds.

• You can get enough vitamin D from being in direct sunlight for 10 to 15 minutes, two or three times weekly.

Falls prevention program –

• falls are responsible for the majority of hip and spine fractures in older people.

• A falls prevention program can provide strategies to help you prevent falls occurring

Measures for Prevention of Falls

• Anchor rugs

• Minimize clutter

• Remove loose wires

• Use nonskid mats

• Install handrails in bathrooms, halls, and long stairways

• Light hallways, stairwells, and entrances

Sources Of Calcium :

• Milk & milk products -cheese, yogurt, ice cream, buttermilk

• Turnip greens, Spinach, Kale, Radish, Okra( lady Finger),bottle gourd

• Dry Beans, such as rajma, chole, chana, lobia, other kidney beans, black-eyed peas, kidney beans, black beans

• Kamal gatta(makhana)

• Water chestnuts/cresnuts(singhada)

• Nuts like peanuts, groundnuts, walnuts, cashew nuts ,almonds , and fruit seeds.

• Seeds like melon seeds, watermelon seeds etc

• Fruits like Custard Apple, Guavas, Banana, Jackfruits, Figs, Oragnges & chiku

Sources of Vit D :

1. Sunlight

2. Food – fatty fish (examples are mackerel, salmon and tuna), egg yolks and liver.

Calcium: take the tablet with meals… and better at night…

Vit D: with meals.. Together with calcium supplements.

Case Study - 29 Answers

1.     What findings are needed for the diagnosis of POAG?            Case Study  – 29

§   enlarged cup-to-disc ratio

§   Visual acuity loss

§   Family history of glaucoma

§   Race/ Ethnicity (African American)

2.     Explain the result of the above mentioned investigations of the ocular exam?    Case Study  – 29

1.      evidence of increased IOP ,

2.      optic nerve head abnormality

3.      open anterior chamber angle,

4.      visual field deficits and no history to suggest a secondary glaucoma (glaucoma due to an identifiable cause.

·         Visual field examination shows defects that are consistent with the state of the optic nerve.

·         Measurements of the nerve fiber layer over the optic nerve can confirm an abnormally thin nerve.

3.     How this patient would be managed ?        Case Study  – 29

Ans : **Damage is permanent to eyes—it cannot be reversed. But medicine and surgery help to stop further damage.

However, Latanoprost 0.005% eyedrops                    only once at bedtime

And, If further required,then,

                   Pilocarpine 1-4% eyedrops thrice a day    OR

                   Brimonidine tartarate 0.2% twice daily     OR

                   Dorzolamide 2% eyedrops   2-3 times a day can be advised.

**If patient is not controlled on 2 topical drugs, then consider alternative treatment options with either laser trabeculoplasty or glaucoma filtering surgery.

4.     Enlist the Patient counselling points in this case?        Case Study  – 29

Patient should be counselled as-

Counselling regarding administration of drops

·         It is extremely important to use your glaucoma eye drops exactly as your ophthalmologist tells you to. That includes taking every dose, every day.

·         But remember, glaucoma eye drops won’t cure glaucoma or improve your vision. They prevent your vision from getting worse. If you don’t use them as prescribed, you could lose your vision.

How to put drops In eyes:

Follow these steps to put in your eye drops:

·         Tilt your head back and look up

·         With one hand, pull your lower eyelid down and away from your eyeball — this makes a “pocket” for the drops

·         With the other hand, hold the eye drop bottle upside down with the tip just above the pocket

·         Squeeze the prescribed number of eye drops into the pocket 

·         For at least 1 minute, close your eye and press your finger lightly on your tear duct (small hole in the inner corner of your eye) — this keeps the eye drop from draining into your nose

·         to use more than 1 type of eye drop, like different drops for different eye conditions, wait at least 5 minutes between each type.

Follow these tips to protect your eyes from infection:

·         Wash your hands with soap and water before you use eye drops

·         Don’t touch the tip of the eye drop bottle with your hands

·         Don’t let the tip of the eye drop bottle touch your eye or eyelid

Storage of drops:

·         All eye drops have an expiration date, which refers to the shelf life of a drop which has not been opened.

·         opened Eye Drops can lose potency and even become contaminated.

·         Most eye drops are stored in a cool dry place and should not be used longer than one month after the bottle is opened, unless otherwise stated on the label.

·         Pt should be advised to mention the date of opening of drops on the label itself.

General points:

·         Do not drive or operate machinery if your glaucoma eye drops make you feel tired or drowsy.

·         Blurry vision, stinging, and redness may improve with time. But if the side effects still bother you, call your ophthalmologist.

·         Never suddenly quit taking your medicine unless your doctor tells you to.

Case Study -30 Answers

1. what is the diagnosis in this case          case study – 30.

Ans • Suspected acute angle closure glaucoma R eye

         • Narrow angle L eye

2. What is the mechanism of angle closure in an episode of acute angle closure glaucoma resulting from a pupillary block?

• The apposition of the pupil border against the lens obstructs aqueous humor flow through the pupil and creates a pressure gradient with increased pressure behind the iris. This moves the iris forward with subsequent apposition of the peripheral iris with the trabecular meshwork. • Decreased drainage by the trabecular meshwork causes increased pressure in the anterior chamber and pushes the iris against the lens. • A portion of vitreous humor moves anteriorly and around the lens, blocking the trabecular meshwork. • Increased aqueous humor drainage through the trabecular meshwork causes a decreased pressure in the anterior chamber, causing a pressure gradient that presses the iris forward and blocks the angle.

3. Mention in brief the surgical options for the disease?          Case Study – 30

Ans: laser iridotomy Is recommended as the first line treatment for all patients Laser peripheral iridotomy/ Surgical iridectomy Preferably, a laser peripheral iridotomy (LPI) is done to alleviate pupillary block. It allows the aqueous to bypass the pupil, providing an alternative route for outflow from posterior to anterior chambers of the eye. Surgical iridectomy may rarely be needed in case of failures of laser iridotomy. Laser peripheral iridotomy: Technique: The role and limitations and possible complications of laser iridotomy are explained to the patient. To reduce the risk of post laser IOP spike and inflammation, apraclonidine 1% or brimonidine 0.15/ 0.2% can be used either before or after the procedure. Alternatively, oral/ topical carbonic anhydrase inhibitors or topical glycerine (in case of corneal epithelial edema secondary to raised IOP) can be used in selected patients.          • It is preferable to reduce IOP to a safe level prior to the procedure. To reduce the risk of bleeding, selected patients on oral anticoagulants for systemic diseases should be counseled and may be asked      to stop their anticoagulants for a few days prior to the procedure.         • Usually iridotomy is recommended between 11-1 o’ clock beneath theeyelids avoiding the 12 o’ clock position. However, others prefer 3 and 9 o’ clock positions.         • PI is avoided at lid margins to reduce symptoms of glare formed by tear meniscus. Successful penetration is seen with a gush of pigments in anterior chamber with a visible deepening of anterior chamber. A minimum opening of 150-200 microns is aimed to ensure patency.        • Surgery is usually considered in case of failure of medical/ laser .        • Management for IOP control or progression of glaucoma despite maximum medical Management.        • Trabeculectomy alone or combined with cataract surgery.

Case Study - 31 Answers

Questions/Answers:

1.      What are the patient counselling points in these case?          Case Study-31

With one hand, pull your lower eyelid down and away from your eyeball — this makes a “pocket” for the drops

To use more than 1 type of eye drop, like different drops for different eye conditions, wait at least 5 minutes between each type.

Follow these tips to protect your eyes from infection :

·         Opened Eye Drops can lose potency and even become contaminated.

·         Wash your face 3-4 times per day with plain water

·         Soak your eyes with cotton filled of water if having any discomfort , it will ease in your painful eyes.

·         Don’t touch your eyes with bare hands

·         If accidently touching the infected eye then wash hands .. OR least possibly don’t touch the eyes which is non infected.

·         Avoid any extra strain on eyes with limited activity of gadgets

·         Do not drive or operate machinery if your  eye drops make you feel tired or drowsy.

2.      What is not a typical exam finding of conjunctivitis?                Case Study-31

·         Eyelid erythema

·         Red conjunctiva

·         Subepithelial corneal infiltrates

·         Anterior chamber cell

·         Mucous in the canthus

Case Study - 32 Answers

1. what are the possible differential diagnosis in this case      case study – 32.

Ans: Differential Diagnosis • Viral Conjunctivitis • Hyperacute Bacterial Conjunctivitis • Chlamydial Conjunctivitis • Allergic Conjunctivitis • Superficial Keratitis • Blepharitis • Episcleritis • Scleritis • Acute Angle-closure Glaucoma • Acute Anterior Uveitis

2. What is the role of Ofloxacin in this Patient’s case?

Ans: Ofloxacin was prescribed to replace tobramycin due to suspecting toxic reaction to Tobramycin. Patient was asked to instil one drop of ofloxacin into the left Eye q2h on the day, to reduce the dosage to qid the next day if the condition Improves, the right eye is to be treated with ofloxacin qid. A review appointment was Scheduled in one week time.

3. Why Tobramycin wasn’t readvised after the PCR result, is there any evidence suggestive for the change?   

Case study – 32.

Ans:        1. the patient had an adverse Reaction to the aminoglycoside, this was evident when she noted that the Discontinuation of tobramycin had reduced ocular hyperaemia, but resulted an Increase in ocular discharge.      2. PCR was performed to rule possible viral and Chlamydia infection. This was deemed necessary as her condition was unresponsive To prescribed therapy.      3. The ophthalmologist had a strong suspicion that the condition Was caused by a gram-negative bacteria due to the nature of ocular discharge, Therefore prescribed ofloxacin as it has a relatively strong antibacterial activity Against gram-negative organisms.

Case Study - 33 Answers

1.What should Leanne’s GP consider as a possible diagnosis?       Case Study- 33

Ans: Following points should be considered for the diagnostic purpose-

·         Take a detailed history from Leanne and her parents and explore the ‘trances’ because they have experienced and witnessed them. This should determine whether an epileptic seizure is likely to have occurred.

·         Diagnosis should not be based on the presence or absence of single features.

·         Consider a history of absence seizures. Leanne is the right age

and gender for this relatively common childhood epilepsy syndrome.

·         The frequent occurrence of the daydreams and the fact that they interrupt her activities are suspicious features of childhood-onset absence epilepsy.

·         The positive family history is supportive but not diagnostic of this epilepsy syndrome. Childhood-onset absence epilepsy is classified as an idiopathic (presumed genetic) generalised epilepsy.

·         Confirm the diagnosis in the surgery. Children with typical absence seizures will often experience one of their absences during hyperventilation (over-breathing). However, hyperventilation usually has to be performed well and for at least 3 minutes to induce an absence.

·         Refer Leanne to a general paediatrician with an interest in epilepsy or a paediatric neurologist to establish the diagnosis.

Ans: First, information should be given to Leanne and her parents about the reasons For further tests, and they should be carried out in a child-centred environment.

·         An electroencephalogram (EEG) should be arranged and Leanne should have This test soon after it has been requested. Because the paediatrician or Paediatric neurologist suspects that her seizures are epileptic in origin, the EEG Should be performed to support a diagnosis of epilepsy.

·         It should not be used in Isolation to make a diagnosis of epilepsy.

·         The healthcare professionals carrying Out the EEG should encourage Leanne to hyperventilate, because this is one of The provocation techniques always undertaken during an EEG.

·         The EEG is abnormal, as in the vast majority of children with childhood-onset Absence epilepsy. It ‘captures’ an absence seizure, particularly during Hyperventilation.

3. What should the discussion around medication include, and what medications may be Prescribed?   Case Study- 33

·         The discussion on anti-epileptic medication should include the different . Medications that are available, and specifically ethosuximide, sodium valproate And lamotrigine, the evidence base for using these medications.

·         Their Common and potentially unwanted side effects. Discussion should also include The likely outcome or prognosis of the epilepsy and specifically that it will go into Spontaneous remission (that is, it will ‘go away’).

·         The family should also be Referred to a paediatric epilepsy nurse who can provide information and Guidance on lifestyle and other non-medical issues.

·         Ethosuximide or sodium valproate should be offered as a first-line treatment.

medical case study questions and answers

  •   Interventions
  •   Drug Encyclopaedia
  •   Bookmarks
  •    
  • Sign Up Now Already have an account?
  • Search MedSchool
  •   Discover
  •   Guides History, exam, tests, drugs and interventions
  •   History
  •   Examination
  •   Investigations
  •   Drugs
  •   Encyclopaedia Diseases, symptoms, signs and drugs
  •   Diseases
  •   Symptoms
  •   Clinical Signs
  •   Test Findings
  •   Study Quizzes and flashcards
  •   Optimise
  •   Quizzes
  •   My Flashcards
  •   Calculators
  • Clinical Med Quiz

Clinical Medicine Quiz

Snapshot: initialising....

We have a new app!

Take the Access library with you wherever you go—easy access to books, videos, images, podcasts, personalized features, and more.

Download the Access App here: iOS and Android . Learn more here!

  • Remote Access
  • Save figures into PowerPoint
  • Download tables as PDFs

Pharmacology in Rehabilitation

APPENDIX C:  Answers to Case Study Questions

  • Download Chapter PDF

Disclaimer: These citations have been automatically generated based on the information we have and it may not be 100% accurate. Please consult the latest official manual style if you have any questions regarding the format accuracy.

Download citation file:

  • Search Book

Jump to a Section

Chapter 6 case study, chapter 7 case study.

  • CHAPTER 8 CASE STUDY
  • CHAPTER 9 CASE STUDY
  • CHAPTER 10 CASE STUDY
  • CHAPTER 11 CASE STUDY
  • CHAPTER 12 CASE STUDY
  • CHAPTER 13 CASE STUDY
  • CHAPTER 14 CASE STUDY
  • CHAPTER 15 CASE STUDY
  • CHAPTER 16 CASE STUDY
  • CHAPTER 17 CASE STUDY
  • CHAPTER 21 CASE STUDY
  • CHAPTER 22 CASE STUDY
  • CHAPTER 23 CASE STUDY
  • CHAPTER 24 CASE STUDY
  • CHAPTER 25 CASE STUDY
  • CHAPTER 26 CASE STUDY
  • CHAPTER 27 CASE STUDY
  • CHAPTER 29 CASE STUDY
  • CHAPTER 30 CASE STUDY
  • CHAPTER 31 CASE STUDY
  • CHAPTER 32 CASE STUDY
  • CHAPTER 33 CASE STUDY
  • CHAPTER 34 CASE STUDY
  • CHAPTER 35 CASE STUDY
  • CHAPTER 36 CASE STUDY
  • CHAPTER 37 CASE STUDY
  • CHAPTER 38 CASE STUDY
  • Full Chapter
  • Supplementary Content

Sedative-Hypnotic Drugs

What is the most likely reason for R.S.'s poor performance in the morning rehabilitation sessions?

The sedative-hypnotic (flurazepam) appeared to be producing a hangover-like effect, which limited the patient's cognitive skills during the early daily activities. Flurazepam is a benzodiazepine drug with a half-life of 2.3 hours and a duration of action of 7 to 8 hours. This drug, however, is metabolized in the liver to active metabolites with half-lives ranging from 30 to 200 hours. It seems likely that R.S. was continuing to experience sedative effects from these active metabolites well into the next morning.

What would be the likely solution?

The therapists can deal with this problem initially by reserving the early morning session for stretching and ROM activities and then gradually moving into upper-body strengthening. Activities that require more patient learning and comprehension can be done later in the morning or in the afternoon. The hangover-like problem should also be brought to the attention of the physician. In this case, the physician switched the hypnotic drug to zolpidem (Ambien), 10 mg administered at bedtime. Zolpidem has a half-life of 2.5 hours and a duration of action of 6 to 8 hours, so R.S. should still get the benefit of a full night's sleep. Because zolpidem is not metabolized to active metabolites, it is unlikely that it will continue to exert sedative-hypnotic effects into the next morning. Zolpidem also affects neuronal GABA receptors differently than benzodiazepines such as flurazepam. This difference might reduce the risk of problems, such as rebound insomnia, when it is time to discontinue the drug.

Antidepressant Drugs

How can the therapist reduce the risk of orthostatic hypotension during rehabilitation sessions?

To reduce orthostatic hypotension, the therapist decided to place the patient on the tilt table for the first day after imipramine was started and to monitor blood pressure regularly. The therapist had the patient perform weight shifting and upper-extremity facilitation activities while he was on the tilt table. The patient tolerated this well, so the therapist had him resume ambulation activities using the parallel bars on the following day. With the patient standing inside the bars, the therapist carefully watched for any subjective signs of dizziness or syncope in the patient (i.e., facial pallor, inability to follow instructions). Standing bouts were also limited in duration. By the third day, ambulation training continued with the patient outside the parallel bars, but the therapist made a point of having the patient's wheelchair close at hand in case the patient began to appear faint. These precautions of careful observation and short, controlled bouts of ambulation were continued throughout the remainder of the patient's hospital stay, and the therapist observed no incident of orthostatic hypotension during physical therapy.

Will clinicians notice an immediate improvement in J.G.'s mood after starting this?

Pop-up div Successfully Displayed

This div only appears when the trigger link is hovered over. Otherwise it is hidden from view.

Please Wait

U.S. flag

An official website of the United States government

The .gov means it's official. Federal government websites often end in .gov or .mil. Before sharing sensitive information, make sure you're on a federal government site.

The site is secure. The https:// ensures that you are connecting to the official website and that any information you provide is encrypted and transmitted securely.

  • Publications
  • Account settings
  • Browse Titles

NCBI Bookshelf. A service of the National Library of Medicine, National Institutes of Health.

StatPearls [Internet]. Treasure Island (FL): StatPearls Publishing; 2024 Jan-.

Cover of StatPearls

StatPearls [Internet].

Case study: 33-year-old female presents with chronic sob and cough.

Sandeep Sharma ; Muhammad F. Hashmi ; Deepa Rawat .

Affiliations

Last Update: February 20, 2023 .

  • Case Presentation

History of Present Illness:  A 33-year-old white female presents after admission to the general medical/surgical hospital ward with a chief complaint of shortness of breath on exertion. She reports that she was seen for similar symptoms previously at her primary care physician’s office six months ago. At that time, she was diagnosed with acute bronchitis and treated with bronchodilators, empiric antibiotics, and a short course oral steroid taper. This management did not improve her symptoms, and she has gradually worsened over six months. She reports a 20-pound (9 kg) intentional weight loss over the past year. She denies camping, spelunking, or hunting activities. She denies any sick contacts. A brief review of systems is negative for fever, night sweats, palpitations, chest pain, nausea, vomiting, diarrhea, constipation, abdominal pain, neural sensation changes, muscular changes, and increased bruising or bleeding. She admits a cough, shortness of breath, and shortness of breath on exertion.

Social History: Her tobacco use is 33 pack-years; however, she quit smoking shortly prior to the onset of symptoms, six months ago. She denies alcohol and illicit drug use. She is in a married, monogamous relationship and has three children aged 15 months to 5 years. She is employed in a cookie bakery. She has two pet doves. She traveled to Mexico for a one-week vacation one year ago.

Allergies:  No known medicine, food, or environmental allergies.

Past Medical History: Hypertension

Past Surgical History: Cholecystectomy

Medications: Lisinopril 10 mg by mouth every day

Physical Exam:

Vitals: Temperature, 97.8 F; heart rate 88; respiratory rate, 22; blood pressure 130/86; body mass index, 28

General: She is well appearing but anxious, a pleasant female lying on a hospital stretcher. She is conversing freely, with respiratory distress causing her to stop mid-sentence.

Respiratory: She has diffuse rales and mild wheezing; tachypneic.

Cardiovascular: She has a regular rate and rhythm with no murmurs, rubs, or gallops.

Gastrointestinal: Bowel sounds X4. No bruits or pulsatile mass.

  • Initial Evaluation

Laboratory Studies:  Initial work-up from the emergency department revealed pancytopenia with a platelet count of 74,000 per mm3; hemoglobin, 8.3 g per and mild transaminase elevation, AST 90 and ALT 112. Blood cultures were drawn and currently negative for bacterial growth or Gram staining.

Chest X-ray

Impression:  Mild interstitial pneumonitis

  • Differential Diagnosis
  • Aspiration pneumonitis and pneumonia
  • Bacterial pneumonia
  • Immunodeficiency state and Pneumocystis jiroveci pneumonia
  • Carcinoid lung tumors
  • Tuberculosis
  • Viral pneumonia
  • Chlamydial pneumonia
  • Coccidioidomycosis and valley fever
  • Recurrent Legionella pneumonia
  • Mediastinal cysts
  • Mediastinal lymphoma
  • Recurrent mycoplasma infection
  • Pancoast syndrome
  • Pneumococcal infection
  • Sarcoidosis
  • Small cell lung cancer
  • Aspergillosis
  • Blastomycosis
  • Histoplasmosis
  • Actinomycosis
  • Confirmatory Evaluation

CT of the chest was performed to further the pulmonary diagnosis; it showed a diffuse centrilobular micronodular pattern without focal consolidation.

On finding pulmonary consolidation on the CT of the chest, a pulmonary consultation was obtained. Further history was taken, which revealed that she has two pet doves. As this was her third day of broad-spectrum antibiotics for a bacterial infection and she was not getting better, it was decided to perform diagnostic bronchoscopy of the lungs with bronchoalveolar lavage to look for any atypical or rare infections and to rule out malignancy (Image 1).

Bronchoalveolar lavage returned with a fluid that was cloudy and muddy in appearance. There was no bleeding. Cytology showed Histoplasma capsulatum .

Based on the bronchoscopic findings, a diagnosis of acute pulmonary histoplasmosis in an immunocompetent patient was made.

Pulmonary histoplasmosis in asymptomatic patients is self-resolving and requires no treatment. However, once symptoms develop, such as in our above patient, a decision to treat needs to be made. In mild, tolerable cases, no treatment other than close monitoring is necessary. However, once symptoms progress to moderate or severe, or if they are prolonged for greater than four weeks, treatment with itraconazole is indicated. The anticipated duration is 6 to 12 weeks total. The response should be monitored with a chest x-ray. Furthermore, observation for recurrence is necessary for several years following the diagnosis. If the illness is determined to be severe or does not respond to itraconazole, amphotericin B should be initiated for a minimum of 2 weeks, but up to 1 year. Cotreatment with methylprednisolone is indicated to improve pulmonary compliance and reduce inflammation, thus improving work of respiration. [1] [2] [3]

Histoplasmosis, also known as Darling disease, Ohio valley disease, reticuloendotheliosis, caver's disease, and spelunker's lung, is a disease caused by the dimorphic fungi  Histoplasma capsulatum native to the Ohio, Missouri, and Mississippi River valleys of the United States. The two phases of Histoplasma are the mycelial phase and the yeast phase.

Etiology/Pathophysiology 

Histoplasmosis is caused by inhaling the microconidia of  Histoplasma  spp. fungus into the lungs. The mycelial phase is present at ambient temperature in the environment, and upon exposure to 37 C, such as in a host’s lungs, it changes into budding yeast cells. This transition is an important determinant in the establishment of infection. Inhalation from soil is a major route of transmission leading to infection. Human-to-human transmission has not been reported. Infected individuals may harbor many yeast-forming colonies chronically, which remain viable for years after initial inoculation. The finding that individuals who have moved or traveled from endemic to non-endemic areas may exhibit a reactivated infection after many months to years supports this long-term viability. However, the precise mechanism of reactivation in chronic carriers remains unknown.

Infection ranges from an asymptomatic illness to a life-threatening disease, depending on the host’s immunological status, fungal inoculum size, and other factors. Histoplasma  spp. have grown particularly well in organic matter enriched with bird or bat excrement, leading to the association that spelunking in bat-feces-rich caves increases the risk of infection. Likewise, ownership of pet birds increases the rate of inoculation. In our case, the patient did travel outside of Nebraska within the last year and owned two birds; these are her primary increased risk factors. [4]

Non-immunocompromised patients present with a self-limited respiratory infection. However, the infection in immunocompromised hosts disseminated histoplasmosis progresses very aggressively. Within a few days, histoplasmosis can reach a fatality rate of 100% if not treated aggressively and appropriately. Pulmonary histoplasmosis may progress to a systemic infection. Like its pulmonary counterpart, the disseminated infection is related to exposure to soil containing infectious yeast. The disseminated disease progresses more slowly in immunocompetent hosts compared to immunocompromised hosts. However, if the infection is not treated, fatality rates are similar. The pathophysiology for disseminated disease is that once inhaled, Histoplasma yeast are ingested by macrophages. The macrophages travel into the lymphatic system where the disease, if not contained, spreads to different organs in a linear fashion following the lymphatic system and ultimately into the systemic circulation. Once this occurs, a full spectrum of disease is possible. Inside the macrophage, this fungus is contained in a phagosome. It requires thiamine for continued development and growth and will consume systemic thiamine. In immunocompetent hosts, strong cellular immunity, including macrophages, epithelial, and lymphocytes, surround the yeast buds to keep infection localized. Eventually, it will become calcified as granulomatous tissue. In immunocompromised hosts, the organisms disseminate to the reticuloendothelial system, leading to progressive disseminated histoplasmosis. [5] [6]

Symptoms of infection typically begin to show within three to17 days. Immunocompetent individuals often have clinically silent manifestations with no apparent ill effects. The acute phase of infection presents as nonspecific respiratory symptoms, including cough and flu. A chest x-ray is read as normal in 40% to 70% of cases. Chronic infection can resemble tuberculosis with granulomatous changes or cavitation. The disseminated illness can lead to hepatosplenomegaly, adrenal enlargement, and lymphadenopathy. The infected sites usually calcify as they heal. Histoplasmosis is one of the most common causes of mediastinitis. Presentation of the disease may vary as any other organ in the body may be affected by the disseminated infection. [7]

The clinical presentation of the disease has a wide-spectrum presentation which makes diagnosis difficult. The mild pulmonary illness may appear as a flu-like illness. The severe form includes chronic pulmonary manifestation, which may occur in the presence of underlying lung disease. The disseminated form is characterized by the spread of the organism to extrapulmonary sites with proportional findings on imaging or laboratory studies. The Gold standard for establishing the diagnosis of histoplasmosis is through culturing the organism. However, diagnosis can be established by histological analysis of samples containing the organism taken from infected organs. It can be diagnosed by antigen detection in blood or urine, PCR, or enzyme-linked immunosorbent assay. The diagnosis also can be made by testing for antibodies again the fungus. [8]

Pulmonary histoplasmosis in asymptomatic patients is self-resolving and requires no treatment. However, once symptoms develop, such as in our above patient, a decision to treat needs to be made. In mild, tolerable cases, no treatment other than close monitoring is necessary. However, once symptoms progress to moderate or severe or if they are prolonged for greater than four weeks, treatment with itraconazole is indicated. The anticipated duration is 6 to 12 weeks. The patient's response should be monitored with a chest x-ray. Furthermore, observation for recurrence is necessary for several years following the diagnosis. If the illness is determined to be severe or does not respond to itraconazole, amphotericin B should be initiated for a minimum of 2 weeks, but up to 1 year. Cotreatment with methylprednisolone is indicated to improve pulmonary compliance and reduce inflammation, thus improving the work of respiration.

The disseminated disease requires similar systemic antifungal therapy to pulmonary infection. Additionally, procedural intervention may be necessary, depending on the site of dissemination, to include thoracentesis, pericardiocentesis, or abdominocentesis. Ocular involvement requires steroid treatment additions and necessitates ophthalmology consultation. In pericarditis patients, antifungals are contraindicated because the subsequent inflammatory reaction from therapy would worsen pericarditis.

Patients may necessitate intensive care unit placement dependent on their respiratory status, as they may pose a risk for rapid decompensation. Should this occur, respiratory support is necessary, including non-invasive BiPAP or invasive mechanical intubation. Surgical interventions are rarely warranted; however, bronchoscopy is useful as both a diagnostic measure to collect sputum samples from the lung and therapeutic to clear excess secretions from the alveoli. Patients are at risk for developing a coexistent bacterial infection, and appropriate antibiotics should be considered after 2 to 4 months of known infection if symptoms are still present. [9]

Prognosis 

If not treated appropriately and in a timely fashion, the disease can be fatal, and complications will arise, such as recurrent pneumonia leading to respiratory failure, superior vena cava syndrome, fibrosing mediastinitis, pulmonary vessel obstruction leading to pulmonary hypertension and right-sided heart failure, and progressive fibrosis of lymph nodes. Acute pulmonary histoplasmosis usually has a good outcome on symptomatic therapy alone, with 90% of patients being asymptomatic. Disseminated histoplasmosis, if untreated, results in death within 2 to 24 months. Overall, there is a relapse rate of 50% in acute disseminated histoplasmosis. In chronic treatment, however, this relapse rate decreases to 10% to 20%. Death is imminent without treatment.

  • Pearls of Wisdom

While illnesses such as pneumonia are more prevalent, it is important to keep in mind that more rare diseases are always possible. Keeping in mind that every infiltrates on a chest X-ray or chest CT is not guaranteed to be simple pneumonia. Key information to remember is that if the patient is not improving under optimal therapy for a condition, the working diagnosis is either wrong or the treatment modality chosen by the physician is wrong and should be adjusted. When this occurs, it is essential to collect a more detailed history and refer the patient for appropriate consultation with a pulmonologist or infectious disease specialist. Doing so, in this case, yielded workup with bronchoalveolar lavage and microscopic evaluation. Microscopy is invaluable for definitively diagnosing a pulmonary consolidation as exemplified here where the results showed small, budding, intracellular yeast in tissue sized 2 to 5 microns that were readily apparent on hematoxylin and eosin staining and minimal, normal flora bacterial growth. 

  • Enhancing Healthcare Team Outcomes

This case demonstrates how all interprofessional healthcare team members need to be involved in arriving at a correct diagnosis. Clinicians, specialists, nurses, pharmacists, laboratory technicians all bear responsibility for carrying out the duties pertaining to their particular discipline and sharing any findings with all team members. An incorrect diagnosis will almost inevitably lead to incorrect treatment, so coordinated activity, open communication, and empowerment to voice concerns are all part of the dynamic that needs to drive such cases so patients will attain the best possible outcomes.

  • Review Questions
  • Access free multiple choice questions on this topic.
  • Comment on this article.

Histoplasma Contributed by Sandeep Sharma, MD

Disclosure: Sandeep Sharma declares no relevant financial relationships with ineligible companies.

Disclosure: Muhammad Hashmi declares no relevant financial relationships with ineligible companies.

Disclosure: Deepa Rawat declares no relevant financial relationships with ineligible companies.

This book is distributed under the terms of the Creative Commons Attribution-NonCommercial-NoDerivatives 4.0 International (CC BY-NC-ND 4.0) ( http://creativecommons.org/licenses/by-nc-nd/4.0/ ), which permits others to distribute the work, provided that the article is not altered or used commercially. You are not required to obtain permission to distribute this article, provided that you credit the author and journal.

  • Cite this Page Sharma S, Hashmi MF, Rawat D. Case Study: 33-Year-Old Female Presents with Chronic SOB and Cough. [Updated 2023 Feb 20]. In: StatPearls [Internet]. Treasure Island (FL): StatPearls Publishing; 2024 Jan-.

In this Page

Bulk download.

  • Bulk download StatPearls data from FTP

Related information

  • PMC PubMed Central citations
  • PubMed Links to PubMed

Similar articles in PubMed

  • Review Palliative Chemotherapy: Does It Only Provide False Hope? The Role of Palliative Care in a Young Patient With Newly Diagnosed Metastatic Adenocarcinoma. [J Adv Pract Oncol. 2017] Review Palliative Chemotherapy: Does It Only Provide False Hope? The Role of Palliative Care in a Young Patient With Newly Diagnosed Metastatic Adenocarcinoma. Doverspike L, Kurtz S, Selvaggi K. J Adv Pract Oncol. 2017 May-Jun; 8(4):382-386. Epub 2017 May 1.
  • Review Breathlessness with pulmonary metastases: a multimodal approach. [J Adv Pract Oncol. 2013] Review Breathlessness with pulmonary metastases: a multimodal approach. Brant JM. J Adv Pract Oncol. 2013 Nov; 4(6):415-22.
  • A 50-Year Old Woman With Recurrent Right-Sided Chest Pain. [Chest. 2022] A 50-Year Old Woman With Recurrent Right-Sided Chest Pain. Saha BK, Bonnier A, Chong WH, Chenna P. Chest. 2022 Feb; 161(2):e85-e89.
  • Suicidal Ideation. [StatPearls. 2024] Suicidal Ideation. Harmer B, Lee S, Duong TVH, Saadabadi A. StatPearls. 2024 Jan
  • Review Domperidone. [Drugs and Lactation Database (...] Review Domperidone. . Drugs and Lactation Database (LactMed®). 2006

Recent Activity

  • Case Study: 33-Year-Old Female Presents with Chronic SOB and Cough - StatPearls Case Study: 33-Year-Old Female Presents with Chronic SOB and Cough - StatPearls

Your browsing activity is empty.

Activity recording is turned off.

Turn recording back on

Connect with NLM

National Library of Medicine 8600 Rockville Pike Bethesda, MD 20894

Web Policies FOIA HHS Vulnerability Disclosure

Help Accessibility Careers

statistics

PharmaDaily-logo

  • Login / Signup

PharmaDaily-logo

Medical Coding Tutorial

  • What is Medical Coding?
  • What are Medical Coding Systems?
  • Importance of Medical Coding
  • What are Medical Coding Guidelines?
  • What is Medical Decision-Making Level?
  • What is ICD-10 coding?
  • What is CPT Coding?
  • What is HCPCS coding?
  • What are HIPAA Regulations?
  • What is Protected Health Information?
  • Anatomy and Physiology Fundamentals
  • Human Anatomy: An Overview of Major Body Systems, Organs, and Tissues
  • Physiology Basics: Understanding Body Functions for Accurate Medical Coding
  • ICD-10-CM Coding Overview
  • Code Structure of ICD-10-CM Coding
  • Coding Conventions of Medical Coding
  • Coding Guidelines for ICD-10-CM
  • How to navigate the Code Sets?
  • Medical Coding Case Studies: Practice Coding Real-World Scenarios
  • Tough Medical Coding Case Studies
  • Current Procedural Terminology Coding System
  • CPT Coding Categories
  • What is Chief Complaint?
  • What is History of the Present Illness?
  • What are Surgical Modifiers?
  • What are Bundled Procedures?
  • Evaluation and Management (E/M) Codes in CPT coding system

CPT Coding: Case Studies

  • EM Coding: Case Studies
  • Inpatient E/M coding Case Studies
  • Outpatient E/M Coding Case Studies
  • Difference between inpatient and outpatient E/M coding
  • What is the HCPCS Coding?
  • What is DMPEOS Coding?
  • HCPCS Overview
  • What is Medical Coding Compliance?
  • Ethical Considerations in Medical Coding
  • Medical Coding Compliance and Fraud
  • Health Insurance Portability and Accountability Act (HIPAA)
  • Medical Coding Auditing and Documentation
  • What Medical Billing?
  • Revenue Cycle Management in Medical Billing
  • Medical Coding and Reimbursement Relationship
  • Medical Coding Claim Submission
  • What are Medical Terminologies?
  • Medical Language Basics for Improved Coding Accuracy
  • Medical Terminologies: Prefixes, Suffixes, and Root Words

Module 1: Introduction to Medical Coding

Module 2: anatomy and physiology fundamentals, module 3: icd-10-cm coding, module 4: cpt coding, module 5: hcpcs level ii coding, module 6: medical coding compliance and ethics, module 7: medical billing and reimbursement, module 8: medical terminology, module 10: medical coding interview questions & answers, join our community on telegram, join the biggest community of pharma students and professionals..

Join our Telegram Community.

These are a few CPT medical coding case study questions along with their correct answers for practice. Medical coding requires accurate code selection based on the provided scenario. Remember to consult the latest CPT codebook for the most up-to-date information. Let's begin:

Case Study 1: Evaluation and Management (E/M) Coding

Scenario: A 35-year-old female patient with complaints of chest pain and shortness of breath visits the primary care physician's office. The physician performs a comprehensive history and examination, and the medical decision-making is of high complexity.

Question 1: Assign the appropriate E/M code for this encounter.

Answer 1: CPT code 99205 (Office or other outpatient visit for the evaluation and management of a new patient, which requires a comprehensive history, comprehensive examination, and high-level medical decision-making) would be appropriate for this encounter.

Case Study 2: Surgical Coding

Scenario: A patient undergoes a laparoscopic cholecystectomy with cholangiography due to gallstones and biliary colic.

Question 2: Assign the appropriate CPT code for the laparoscopic cholecystectomy with cholangiography.

Answer 2: CPT code 47562 (Laparoscopy, surgical; cholecystectomy) should be assigned for the laparoscopic cholecystectomy. Additionally, CPT code 47563 (Cholangiography, intraoperative, when performed, and includes the use of a cholangiogram, real-time fluoroscopic guidance, and all associated radiological supervision and interpretation; intraoperative) should be assigned for the cholangiography.

Case Study 3: Pathology/Laboratory Coding

Scenario: A patient undergoes a comprehensive metabolic panel (CMP) and complete blood count (CBC) tests at a clinical laboratory.

Question 3: Assign the appropriate CPT codes for the CMP and CBC tests.

Answer 3: CPT code 80053 (Comprehensive metabolic panel) should be assigned for the CMP. CPT code 85025 (Complete (CBC), automated (Hgb, Hct, RBC, WBC, and platelet count)) should be assigned for the CBC.

Case Study 4: Radiology Coding

Scenario: A patient comes to the radiology department for an X-ray of the right ankle after twisting it during a sports activity.

Question 4: Assign the appropriate CPT code for the X-ray of the right ankle.

Answer 4: CPT code 73610 (Radiologic examination, ankle; 2 views) should be assigned for the X-ray of the right ankle.

Case Study 5: Emergency Department (ED) Coding

Scenario: A 25-year-old male patient arrives at the emergency department with a deep laceration on his left forearm. The physician performs wound exploration, extensive cleaning, and suturing.

Question 5: Assign the appropriate E/M code for this emergency department encounter.

Answer 5: CPT code 99284 (Emergency department visit for the evaluation and management of a patient, which requires a detailed history, detailed examination, and medical decision-making of moderate complexity) would be appropriate for this emergency department encounter.

Please note that medical coding is a vast field, and these case studies cover only a few scenarios. It is essential to continue studying and practicing with a wide range of real-world scenarios to gain expertise in CPT medical coding. Always verify the codes with the latest CPT codebook and any relevant official guidelines.

medical case study questions and answers

Book a FREE live class. NOW!

Fill your details and select a date for your live class

  • Open access
  • Published: 22 April 2024

Artificial intelligence and medical education: application in classroom instruction and student assessment using a pharmacology & therapeutics case study

  • Kannan Sridharan 1 &
  • Reginald P. Sequeira 1  

BMC Medical Education volume  24 , Article number:  431 ( 2024 ) Cite this article

354 Accesses

1 Altmetric

Metrics details

Artificial intelligence (AI) tools are designed to create or generate content from their trained parameters using an online conversational interface. AI has opened new avenues in redefining the role boundaries of teachers and learners and has the potential to impact the teaching-learning process.

In this descriptive proof-of- concept cross-sectional study we have explored the application of three generative AI tools on drug treatment of hypertension theme to generate: (1) specific learning outcomes (SLOs); (2) test items (MCQs- A type and case cluster; SAQs; OSPE); (3) test standard-setting parameters for medical students.

Analysis of AI-generated output showed profound homology but divergence in quality and responsiveness to refining search queries. The SLOs identified key domains of antihypertensive pharmacology and therapeutics relevant to stages of the medical program, stated with appropriate action verbs as per Bloom’s taxonomy. Test items often had clinical vignettes aligned with the key domain stated in search queries. Some test items related to A-type MCQs had construction defects, multiple correct answers, and dubious appropriateness to the learner’s stage. ChatGPT generated explanations for test items, this enhancing usefulness to support self-study by learners. Integrated case-cluster items had focused clinical case description vignettes, integration across disciplines, and targeted higher levels of competencies. The response of AI tools on standard-setting varied. Individual questions for each SAQ clinical scenario were mostly open-ended. The AI-generated OSPE test items were appropriate for the learner’s stage and identified relevant pharmacotherapeutic issues. The model answers supplied for both SAQs and OSPEs can aid course instructors in planning classroom lessons, identifying suitable instructional methods, establishing rubrics for grading, and for learners as a study guide. Key lessons learnt for improving AI-generated test item quality are outlined.

Conclusions

AI tools are useful adjuncts to plan instructional methods, identify themes for test blueprinting, generate test items, and guide test standard-setting appropriate to learners’ stage in the medical program. However, experts need to review the content validity of AI-generated output. We expect AIs to influence the medical education landscape to empower learners, and to align competencies with curriculum implementation. AI literacy is an essential competency for health professionals.

Peer Review reports

Artificial intelligence (AI) has great potential to revolutionize the field of medical education from curricular conception to assessment [ 1 ]. AIs used in medical education are mostly generative AI large language models that were developed and validated based on billions to trillions of parameters [ 2 ]. AIs hold promise in the incorporation of history-taking, assessment, diagnosis, and management of various disorders [ 3 ]. While applications of AIs in undergraduate medical training are being explored, huge ethical challenges remain in terms of data collection, maintaining anonymity, consent, and ownership of the provided data [ 4 ]. AIs hold a promising role amongst learners because they can deliver a personalized learning experience by tracking their progress and providing real-time feedback, thereby enhancing their understanding in the areas they are finding difficult [ 5 ]. Consequently, a recent survey has shown that medical students have expressed their interest in acquiring competencies related to the use of AIs in healthcare during their undergraduate medical training [ 6 ].

Pharmacology and Therapeutics (P & T) is a core discipline embedded in the undergraduate medical curriculum, mostly in the pre-clerkship phase. However, the application of therapeutic principles forms one of the key learning objectives during the clerkship phase of the undergraduate medical career. Student assessment in pharmacology & therapeutics (P&T) is with test items such as multiple-choice questions (MCQs), integrated case cluster questions, short answer questions (SAQs), and objective structured practical examination (OSPE) in the undergraduate medical curriculum. It has been argued that AIs possess the ability to communicate an idea more creatively than humans [ 7 ]. It is imperative that with access to billions of trillions of datasets the AI platforms hold promise in playing a crucial role in the conception of various test items related to any of the disciplines in the undergraduate medical curriculum. Additionally, AIs provide an optimized curriculum for a program/course/topic addressing multidimensional problems [ 8 ], although robust evidence for this claim is lacking.

The existing literature has evaluated the knowledge, attitude, and perceptions of adopting AI in medical education. Integration of AIs in medical education is the need of the hour in all health professional education. However, the academic medical fraternity facing challenges in the incorporation of AIs in the medical curriculum due to factors such as inadequate grounding in data analytics, lack of high-quality firm evidence favoring the utility of AIs in medical education, and lack of funding [ 9 ]. Open-access AI platforms are available free to users without any restrictions. Hence, as a proof-of-concept, we chose to explore the utility of three AI platforms to identify specific learning objectives (SLOs) related to pharmacology discipline in the management of hypertension for medical students at different stages of their medical training.

Study design and ethics

The present study is observational, cross-sectional in design, conducted in the Department of Pharmacology & Therapeutics, College of Medicine and Medical Sciences, Arabian Gulf University, Kingdom of Bahrain, between April and August 2023. Ethical Committee approval was not sought given the nature of this study that neither had any interaction with humans, nor collection of any personal data was involved.

Study procedure

We conducted the present study in May-June 2023 with the Poe© chatbot interface created by Quora© that provides access to the following three AI platforms:

Sage Poe [ 10 ]: A generative AI search engine developed by Anthropic © that conceives a response based on the written input provided. Quora has renamed Sage Poe as Assistant © from July 2023 onwards.

Claude-Instant [ 11 ]: A retrieval-based AI search engine developed by Anthropic © that collates a response based on pre-written responses amongst the existing databases.

ChatGPT version 3.5 [ 12 ]: A generative architecture-based AI search engine developed by OpenAI © trained on large and diverse datasets.

We queried the chatbots to generate SLOs, A-type MCQs, integrated case cluster MCQs, integrated SAQs, and OSPE test items in the domain of systemic hypertension related to the P&T discipline. Separate prompts were used to generate outputs for pre-clerkship (preclinical) phase students, and at the time of graduation (before starting residency programs). Additionally, we have also evaluated the ability of these AI platforms to estimate the proportion of students correctly answering these test items. We used the following queries for each of these objectives:

Specific learning objectives

Can you generate specific learning objectives in the pharmacology discipline relevant to undergraduate medical students during their pre-clerkship phase related to anti-hypertensive drugs?

Can you generate specific learning objectives in the pharmacology discipline relevant to undergraduate medical students at the time of graduation related to anti-hypertensive drugs?

A-type MCQs

In the initial query used for A-type of item, we specified the domains (such as the mechanism of action, pharmacokinetics, adverse reactions, and indications) so that a sample of test items generated without any theme-related clutter, shown below:

Write 20 single best answer MCQs with 5 choices related to anti-hypertensive drugs for undergraduate medical students during the pre-clerkship phase of which 5 MCQs should be related to mechanism of action, 5 MCQs related to pharmacokinetics, 5 MCQs related to adverse reactions, and 5 MCQs should be related to indications.

The MCQs generated with the above search query were not based on clinical vignettes. We queried again to generate MCQs using clinical vignettes specifically because most medical schools have adopted problem-based learning (PBL) in their medical curriculum.

Write 20 single best answer MCQs with 5 choices related to anti-hypertensive drugs for undergraduate medical students during the pre-clerkship phase using a clinical vignette for each MCQ of which 5 MCQs should be related to the mechanism of action, 5 MCQs related to pharmacokinetics, 5 MCQs related to adverse reactions, and 5 MCQs should be related to indications.

We attempted to explore whether AI platforms can provide useful guidance on standard-setting. Hence, we used the following search query.

Can you do a simulation with 100 undergraduate medical students to take the above questions and let me know what percentage of students got each MCQ correct?

Integrated case cluster MCQs

Write 20 integrated case cluster MCQs with 2 questions in each cluster with 5 choices for undergraduate medical students during the pre-clerkship phase integrating pharmacology and physiology related to systemic hypertension with a case vignette.

Write 20 integrated case cluster MCQs with 2 questions in each cluster with 5 choices for undergraduate medical students during the pre-clerkship phase integrating pharmacology and physiology related to systemic hypertension with a case vignette. Please do not include ‘none of the above’ as the choice. (This modified search query was used because test items with ‘None of the above’ option were generated with the previous search query).

Write 20 integrated case cluster MCQs with 2 questions in each cluster with 5 choices for undergraduate medical students at the time of graduation integrating pharmacology and physiology related to systemic hypertension with a case vignette.

Integrated short answer questions

Write a short answer question scenario with difficult questions based on the theme of a newly diagnosed hypertensive patient for undergraduate medical students with the main objectives related to the physiology of blood pressure regulation, risk factors for systemic hypertension, pathophysiology of systemic hypertension, pathological changes in the systemic blood vessels in hypertension, pharmacological management, and non-pharmacological treatment of systemic hypertension.

Write a short answer question scenario with moderately difficult questions based on the theme of a newly diagnosed hypertensive patient for undergraduate medical students with the main objectives related to the physiology of blood pressure regulation, risk factors for systemic hypertension, pathophysiology of systemic hypertension, pathological changes in the systemic blood vessels in hypertension, pharmacological management, and non-pharmacological treatment of systemic hypertension.

Write a short answer question scenario with questions based on the theme of a newly diagnosed hypertensive patient for undergraduate medical students at the time of graduation with the main objectives related to the physiology of blood pressure regulation, risk factors for systemic hypertension, pathophysiology of systemic hypertension, pathological changes in the systemic blood vessels in hypertension, pharmacological management, and non-pharmacological treatment of systemic hypertension.

Can you generate 5 OSPE pharmacology and therapeutics prescription writing exercises for the assessment of undergraduate medical students at the time of graduation related to anti-hypertensive drugs?

Can you generate 5 OSPE pharmacology and therapeutics prescription writing exercises containing appropriate instructions for the patients for the assessment of undergraduate medical students during their pre-clerkship phase related to anti-hypertensive drugs?

Can you generate 5 OSPE pharmacology and therapeutics prescription writing exercises containing appropriate instructions for the patients for the assessment of undergraduate medical students at the time of graduation related to anti-hypertensive drugs?

Both authors independently evaluated the AI-generated outputs, and a consensus was reached. We cross-checked the veracity of answers suggested by AIs as per the Joint National Commission Guidelines (JNC-8) and Goodman and Gilman’s The Pharmacological Basis of Therapeutics (2023), a reference textbook [ 13 , 14 ]. Errors in the A-type MCQs were categorized as item construction defects, multiple correct answers, and uncertain appropriateness to the learner’s level. Test items in the integrated case cluster MCQs, SAQs and OSPEs were evaluated with the Preliminary Conceptual Framework for Establishing Content Validity of AI-Generated Test Items based on the following domains: technical accuracy, comprehensiveness, education level, and lack of construction defects (Table  1 ). The responses were categorized as complete and deficient for each domain.

The pre-clerkship phase SLOs identified by Sage Poe, Claude-Instant, and ChatGPT are listed in the electronic supplementary materials 1 – 3 , respectively. In general, a broad homology in SLOs generated by the three AI platforms was observed. All AI platforms identified appropriate action verbs as per Bloom’s taxonomy to state the SLO; action verbs such as describe, explain, recognize, discuss, identify, recommend, and interpret are used to state the learning outcome. The specific, measurable, achievable, relevant, time-bound (SMART) SLOs generated by each AI platform slightly varied. All key domains of antihypertensive pharmacology to be achieved during the pre-clerkship (pre-clinical) years were relevant for graduating doctors. The SLOs addressed current JNC Treatment Guidelines recommended classes of antihypertensive drugs, the mechanism of action, pharmacokinetics, adverse effects, indications/contraindications, dosage adjustments, monitoring therapy, and principles of monotherapy and combination therapy.

The SLOs to be achieved by undergraduate medical students at the time of graduation identified by Sage Poe, Claude-Instant, and ChatGPT listed in electronic supplementary materials 4 – 6 , respectively. The identified SLOs emphasize the application of pharmacology knowledge within a clinical context, focusing on competencies needed to function independently in early residency stages. These SLOs go beyond knowledge recall and mechanisms of action to encompass competencies related to clinical problem-solving, rational prescribing, and holistic patient management. The SLOs generated require higher cognitive ability of the learner: action verbs such as demonstrate, apply, evaluate, analyze, develop, justify, recommend, interpret, manage, adjust, educate, refer, design, initiate & titrate were frequently used.

The MCQs for the pre-clerkship phase identified by Sage Poe, Claude-Instant, and ChatGPT listed in the electronic supplementary materials 7 – 9 , respectively, and those identified with the search query based on the clinical vignette in electronic supplementary materials ( 10 – 12 ).

All MCQs generated by the AIs in each of the four domains specified [mechanism of action (MOA); pharmacokinetics; adverse drug reactions (ADRs), and indications for antihypertensive drugs] are quality test items with potential content validity. The test items on MOA generated by Sage Poe included themes such as renin-angiotensin-aldosterone (RAAS) system, beta-adrenergic blockers (BB), calcium channel blockers (CCB), potassium channel openers, and centrally acting antihypertensives; on pharmacokinetics included high oral bioavailability/metabolism in liver [angiotensin receptor blocker (ARB)-losartan], long half-life and renal elimination [angiotensin converting enzyme inhibitors (ACEI)-lisinopril], metabolism by both liver and kidney (beta-blocker (BB)-metoprolol], rapid onset- short duration of action (direct vasodilator-hydralazine), and long-acting transdermal drug delivery (centrally acting-clonidine). Regarding the ADR theme, dry cough, angioedema, and hyperkalemia by ACEIs in susceptible patients, reflex tachycardia by CCB/amlodipine, and orthostatic hypotension by CCB/verapamil addressed. Clinical indications included the drug of choice for hypertensive patients with concomitant comorbidity such as diabetics (ACEI-lisinopril), heart failure and low ejection fraction (BB-carvedilol), hypertensive urgency/emergency (alpha cum beta receptor blocker-labetalol), stroke in patients with history recurrent stroke or transient ischemic attack (ARB-losartan), and preeclampsia (methyldopa).

Almost similar themes under each domain were identified by the Claude-Instant AI platform with few notable exceptions: hydrochlorothiazide (instead of clonidine) in MOA and pharmacokinetics domains, respectively; under the ADR domain ankle edema/ amlodipine, sexual dysfunction and fatigue in male due to alpha-1 receptor blocker; under clinical indications the best initial monotherapy for clinical scenarios such as a 55-year old male with Stage-2 hypertension; a 75-year-old man Stage 1 hypertension; a 35-year-old man with Stage I hypertension working on night shifts; and a 40-year-old man with stage 1 hypertension and hyperlipidemia.

As with Claude-Instant AI, ChatGPT-generated test items on MOA were mostly similar. However, under the pharmacokinetic domain, immediate- and extended-release metoprolol, the effect of food to enhance the oral bioavailability of ramipril, and the highest oral bioavailability of amlodipine compared to other commonly used antihypertensives were the themes identified. Whereas the other ADR themes remained similar, constipation due to verapamil was a new theme addressed. Notably, in this test item, amlodipine was an option that increased the difficulty of this test item because amlodipine therapy is also associated with constipation, albeit to a lesser extent, compared to verapamil. In the clinical indication domain, the case description asking “most commonly used in the treatment of hypertension and heart failure” is controversial because the options listed included losartan, ramipril, and hydrochlorothiazide but the suggested correct answer was ramipril. This is a good example to stress the importance of vetting the AI-generated MCQ by experts for content validity and to assure robust psychometrics. The MCQ on the most used drug in the treatment of “hypertension and diabetic nephropathy” is more explicit as opposed to “hypertension and diabetes” by Claude-Instant because the therapeutic concept of reducing or delaying nephropathy must be distinguished from prevention of nephropathy, although either an ACEI or ARB is the drug of choice for both indications.

It is important to align student assessment to the curriculum; in the PBL curriculum, MCQs with a clinical vignette are preferred. The modification of the query specifying the search to generate MCQs with a clinical vignette on domains specified previously gave appropriate output by all three AI platforms evaluated (Sage Poe; Claude- Instant; Chat GPT). The scenarios generated had a good clinical fidelity and educational fit for the pre-clerkship student perspective.

The errors observed with AI outputs on the A-type MCQs are summarized in Table  2 . No significant pattern was observed except that Claude-Instant© generated test items in a stereotyped format such as the same choices for all test items related to pharmacokinetics and indications, and all the test items in the ADR domain are linked to the mechanisms of action of drugs. This illustrates the importance of reviewing AI-generated test items by content experts for content validity to ensure alignment with evidence-based medicine and up-to-date treatment guidelines.

The test items generated by ChatGPT had the advantage of explanations supplied rendering these more useful for learners to support self-study. The following examples illustrate this assertion: “ A patient with hypertension is started on a medication that works by blocking beta-1 receptors in the heart (metoprolol)”. Metoprolol is a beta blocker that works by blocking beta-1 receptors in the heart, which reduces heart rate and cardiac output, resulting in a decrease in blood pressure. However, this explanation is incomplete because there is no mention of other less important mechanisms, of beta receptor blockers on renin release. Also, these MCQs were mostly recall type: Which of the following medications is known to have a significant first-pass effect? The explanation reads: propranolol is known to have a significant first pass-effect, meaning that a large portion of the drug is metabolized by the liver before it reaches systemic circulation. Losartan, amlodipine, ramipril, and hydrochlorothiazide do not have significant first-pass effect. However, it is also important to extend the explanation further by stating that the first-pass effect of propranolol does not lead to total loss of pharmacological activity because the metabolite hydroxy propranolol also has potent beta-blocking activity. Another MCQ test item had a construction defect: “A patient with hypertension is started on a medication that can cause photosensitivity. Which of the following medications is most likely responsible?” Options included: losartan, amlodipine, ramipril, hydrochlorothiazide, hydrochlorothiazide/triamterene. The explanation states that hydrochlorothiazide (HCTZ) can cause photosensitivity, which is increased sensitivity to sunlight that can cause skin rashes and sunburns. Losartan, amlodipine, ramipril, and HCTZ/triamterene are less likely to cause photosensitivity. However, it is irrational to claim that HCTZ/triamterene, one of the wrong options, is less likely to cause photosensitivity. The difficulty index of such test items is likely unacceptable in high-stake tests to make equitable psychometric decisions.

The responses from Sage Poe, Claude-Instant, and ChatGPT on guidance for standard-setting methods are listed in the electronic supplementary materials 13 – 15 . Sage Poe offered this information with the caveat that “the results of this simulation were purely hypothetical and do not reflect the actual performance of any real students”. The simulation estimates of examinees correctly answering test items ranged from 60 to 90% for the mechanism of action, 65–85% for pharmacokinetics, 60–80% for adverse reactions, and 75–90% for clinical indications of antihypertensives.

In contrast, Claude-Instant admitted that “Performing a full simulation with 100 medical students is beyond the scope of what I can do as an AI assistant. I can generate sample MCQ questions and answers, but I do not have access to a population of medical students to administer the questions. A true simulation that administers questions to real students and analyzes the results would require access to a database of medical students, a platform to deliver the questions, and code to analyze the responses. As an AI assistant, I lack these capabilities. While I cannot perform a full simulation, here are some considerations for administering MCQ questions to medical students: provide details on the purpose and instructions for the test to ensure students understand what is expected out of them; consider including some easier questions in addition to more challenging ones to boost students’ confidence and motivation; avoid ambiguous wording and ensure each question has only one unambiguously correct answer option; when analyzing results, look for questions factors that may have impacted performance, like unclear wording or answer options that are too similar. Focusing on weak questions can help improve future tests; compare performance across different student groups to identify areas where more teaching may be needed; provide detailed answer explanations and review sessions after the test to help students learn from any errors and reinforce correct concepts”. However, ChatGPT offered the simulation result estimates for the 20 MCQ on antihypertensive drugs: mechanism of action ranged from 67 to 92%, pharmacokinetics 63–86%, adverse effects 65–82%, and clinical indications 64–89%. Furthermore, it also stated that “Overall, the performance of the students was quite good, with most questions having a response rate of over 70%. However, there were some questions where the projected correct response rate was rather low, such as Question #5 (Mechanism of Action of Hydrochlorothiazide; see Electronic Supplementary Material 12 ) and Question 18 (Indications for Verapamil; see Electronic Supplementary Material 10 ). This may suggest areas where students need more focused education or review.”

We asked AI assistants to generate 20 integrated case cluster MCQs with 2 test items in each cluster with five options for undergraduate medical students in the pre-clerkship phase integrating pharmacology and physiology related to systemic hypertension with a case vignette and the responses by Sage Poe, Claude-Instant, and ChatGPT are listed in the electronic supplementary materials ( 16 – 18 ). In all instances, the test items generated had focused case descriptions in the form of a clinical vignette, and horizontal integration across the pathophysiology of hypertension and pharmacology of antihypertensive drugs. These test items mostly targeted the ‘knows (knowledge)’ or ‘knows how (competence)’ level on Miller’s pyramid and are suitable for assessing the clinical competence of pre-clerkship medical students, especially in an integrated PBL curriculum. Both the AI assistants generated excellent clinical vignettes and themes; however, most of the cluster MCQs by ChatGPT had “None of the above” as an option, which is often considered a test item construction flaw. Notwithstanding these limitations, case cluster integrated test items are valuable for learners to integrate their knowledge of different basic medical sciences and their application to clinical sciences. This integrated approach can be used for both instructional and student assessment purposes to make the course more meaningful. Indeed, one of the basic tenets of PBL is curriculum integration.

In the next query, we asked AI assistants to write integrated case cluster MCQs with 2 test items in each cluster with 5 options for undergraduate medical students at the time of graduation integrating pharmacology and physiology of hypertension with a case vignette. The responses are listed in the electronic supplementary materials 19 – 21 . Claude-Instant has provided only partial responses to both the queries. Overall, the information provided in case vignettes requires the learner to interpret the clinical information and make therapeutic decisions. Such test items are appropriate for assessing the minimum competency of medical school graduates to start internship or residency programs; these MCQs are generally comparable to test items used in USMLE Part 2. The evaluation of the integrated case cluster test items provided by all the AI platforms is summarized in Table  3 . In general, the AI outputs had at least one construction defect, and did not include all the core sub-topics to a sufficient degree.

When asked to simulate the above-generated test items to 100 undergraduate medical students at the time of graduation and provide a percentage of students who are likely to correctly answer each item, the AI assistant responded:

“ As an AI language model, I cannot simulate the responses of 100 undergraduate medical students likely to obtain the correct answer for each question. However, I can provide some general information about the difficulty level of each question based on the complexity of the concepts being tested and the level of integration between pharmacology and physiology ”.

AI assistant went on to clarify that questions that require knowledge of basic pharmacology principles, such as the mechanism of action of specific drugs, are likely to be easier for students to answer correctly. Test items that require an understanding of the physiological mechanisms underlying hypertension and correlating with symptoms are likely to be more challenging for students. The AI assistant sorted these test items into two categories accordingly. Overall, the difficulty level of the test item is based on the level of integration between pharmacology and pathophysiology. Test items that require an understanding of both pharmacological and physiological mechanisms are likely to be more challenging for students requiring a strong foundation in both pharmacology and physiology concepts to be able to correctly answer integrated case-cluster MCQs.

Short answer questions

The responses to a search query on generating SAQs appropriate to the pre-clerkship phase Sage Poe, Claude-Instant, and ChatGPT generated items are listed in the electronic supplementary materials 22 – 24 for difficult questions and 25–27 for moderately difficult questions.

It is apparent from these case vignette descriptions that the short answer question format varied. Accordingly, the scope for asking individual questions for each scenario is open-ended. In all instances, model answers are supplied which are helpful for the course instructor to plan classroom lessons, identify appropriate instructional methods, and establish rubrics for grading the answer scripts, and as a study guide for students.

We then wanted to see to what extent AI can differentiate the difficulty of the SAQ by replacing the search term “difficult” with “moderately difficult” in the above search prompt: the changes in the revised case scenarios are substantial. Perhaps the context of learning and practice (and the level of the student in the MD/medical program) may determine the difficulty level of SAQ generated. It is worth noting that on changing the search from cardiology to internal medicine rotation in Sage Poe the case description also changed. Thus, it is essential to select an appropriate AI assistant, perhaps by trial and error, to generate quality SAQs. Most of the individual questions tested stand-alone knowledge and did not require students to demonstrate integration.

The responses of Sage Poe, Claude-Instant, and ChatGPT for the search query to generate SAQs at the time of graduation are listed in the electronic supplementary materials 28 – 30 . It is interesting to note how AI assistants considered the stage of the learner while generating the SAQ. The response by Sage Poe is illustrative for comparison. “You are a newly graduated medical student who is working in a hospital” versus “You are a medical student in your pre-clerkship.”

Some questions were retained, deleted, or modified to align with competency appropriate to the context (Electronic Supplementary Materials 28 – 30 ). Overall, the test items at both levels from all AI platforms were technically accurate and thorough addressing the topics related to different disciplines (Table  3 ). The differences in learning objective transition are summarized in Table  4 . A comparison of learning objectives revealed that almost all objectives remained the same except for a few (Table  5 ).

A similar trend was apparent with test items generated by other AI assistants, such as ChatGPT. The contrasting differences in questions are illustrated by the vertical integration of basic sciences and clinical sciences (Table  6 ).

Taken together, these in-depth qualitative comparisons suggest that AI assistants such as Sage Poe and ChatGPT consider the learner’s stage of training in designing test items, learning outcomes, and answers expected from the examinee. It is critical to state the search query explicitly to generate quality output by AI assistants.

The OSPE test items generated by Claude-Instant and ChatGPT appropriate to the pre-clerkship phase (without mentioning “appropriate instructions for the patients”) are listed in the electronic supplementary materials 31 and 32 and with patient instructions on the electronic supplementary materials 33 and 34 . For reasons unknown, Sage Poe did not provide any response to this search query.

The five OSPE items generated were suitable to assess the prescription writing competency of pre-clerkship medical students. The clinical scenarios identified by the three AI platforms were comparable; these scenarios include patients with hypertension and impaired glucose tolerance in a 65-year-old male, hypertension with chronic kidney disease (CKD) in a 55-year-old woman, resistant hypertension with obstructive sleep apnea in a 45-year-old man, and gestational hypertension at 32 weeks in a 35-year-old (Claude-Instant AI). Incorporating appropriate instructions facilitates the learner’s ability to educate patients and maximize safe and effective therapy. The OSPE item required students to write a prescription with guidance to start conservatively, choose an appropriate antihypertensive drug class (drug) based on the patients’ profile, specifying drug name, dose, dosing frequency, drug quantity to be dispensed, patient name, date, refill, and caution as appropriate, in addition to prescribers’ name, signature, and license number. In contrast, ChatGPT identified clinical scenarios to include patients with hypertension and CKD, hypertension and bronchial asthma, gestational diabetes, hypertension and heart failure, and hypertension and gout (ChatGPT). Guidance for dosage titration, warnings to be aware, safety monitoring, and frequency of follow-up and dose adjustment. These test items are designed to assess learners’ knowledge of P & T of antihypertensives, as well as their ability to provide appropriate instructions to patients. These clinical scenarios for writing prescriptions assess students’ ability to choose an appropriate drug class, write prescriptions with proper labeling and dosing, reflect drug safety profiles, and risk factors, and make modifications to meet the requirements of special populations. The prescription is required to state the drug name, dose, dosing frequency, patient name, date, refills, and cautions or instructions as needed. A conservative starting dose, once or twice daily dosing frequency based on the drug, and instructions to titrate the dose slowly if required.

The responses from Claude-Instant and ChatGPT for the search query related to generating OSPE test items at the time of graduation are listed in electronic supplementary materials 35 and 36 . In contrast to the pre-clerkship phase, OSPEs generated for graduating doctors’ competence assessed more advanced drug therapy comprehension. For example, writing a prescription for:

(1) A 65-year- old male with resistant hypertension and CKD stage 3 to optimize antihypertensive regimen required the answer to include starting ACEI and diuretic, titrating the dosage over two weeks, considering adding spironolactone or substituting ACEI with an ARB, and need to closely monitor serum electrolytes and kidney function closely.

(2) A 55-year-old woman with hypertension and paroxysmal arrhythmia required the answer to include switching ACEI to ARB due to cough, adding a CCB or beta blocker for rate control needs, and adjusting the dosage slowly and monitoring for side effects.

(3) A 45-year-old man with masked hypertension and obstructive sleep apnea require adding a centrally acting antihypertensive at bedtime and increasing dosage as needed based on home blood pressure monitoring and refer to CPAP if not already using one.

(4) A 75-year-old woman with isolated systolic hypertension and autonomic dysfunction to require stopping diuretic and switching to an alpha blocker, upward dosage adjustment and combining with other antihypertensives as needed based on postural blood pressure changes and symptoms.

(5) A 35-year-old pregnant woman with preeclampsia at 29 weeks require doubling methyldopa dose and consider adding labetalol or nifedipine based on severity and educate on signs of worsening and to follow-up immediately for any concerning symptoms.

These case scenarios are designed to assess the ability of the learner to comprehend the complexity of antihypertensive regimens, make evidence-based regimen adjustments, prescribe multidrug combinations based on therapeutic response and tolerability, monitor complex patients for complications, and educate patients about warning signs and follow-up.

A similar output was provided by ChatGPT, with clinical scenarios such as prescribing for patients with hypertension and myocardial infarction; hypertension and chronic obstructive pulmonary airway disease (COPD); hypertension and a history of angina; hypertension and a history of stroke, and hypertension and advanced renal failure. In these cases, wherever appropriate, pharmacotherapeutic issues like taking ramipril after food to reduce side effects such as giddiness; selection of the most appropriate beta-blocker such as nebivolol in patients with COPD comorbidity; the importance of taking amlodipine at the same time every day with or without food; preference for telmisartan among other ARBs in stroke; choosing furosemide in patients with hypertension and edema and taking the medication with food to reduce the risk of gastrointestinal adverse effect are stressed.

The AI outputs on OSPE test times were observed to be technically accurate, thorough in addressing core sub-topics suitable for the learner’s level and did not have any construction defects (Table  3 ). Both AIs provided the model answers with explanatory notes. This facilitates the use of such OSPEs for self-assessment by learners for formative assessment purposes. The detailed instructions are helpful in creating optimized therapy regimens, and designing evidence-based regimens, to provide appropriate instructions to patients with complex medical histories. One can rely on multiple AI sources to identify, shortlist required case scenarios, and OSPE items, and seek guidance on expected model answers with explanations. The model answer guidance for antihypertensive drug classes is more appropriate (rather than a specific drug of a given class) from a teaching/learning perspective. We believe that these scenarios can be refined further by providing a focused case history along with relevant clinical and laboratory data to enhance clinical fidelity and bring a closer fit to the competency framework.

In the present study, AI tools have generated SLOs that comply with the current principles of medical education [ 15 ]. AI tools are valuable in constructing SLOs and so are especially useful for medical fraternities where training in medical education is perceived as inadequate, more so in the early stages of their academic career. Data suggests that only a third of academics in medical schools have formal training in medical education [ 16 ] which is a limitation. Thus, the credibility of alternatives, such as the AIs, is evaluated to generate appropriate course learning outcomes.

We observed that the AI platforms in the present study generated quality test items suitable for different types of assessment purposes. The AI-generated outputs were similar with minor variation. We have used generative AIs in the present study that could generate new content from their training dataset [ 17 ]. Problem-based and interactive learning approaches are referred to as “bottom-up” where learners obtain first-hand experience in solving the cases first and then indulge in discussion with the educators to refine their understanding and critical thinking skills [ 18 ]. We suggest that AI tools can be useful for this approach for imparting the core knowledge and skills related to Pharmacology and Therapeutics to undergraduate medical students. A recent scoping review evaluating the barriers to writing quality test items based on 13 studies has concluded that motivation, time constraints, and scheduling were the most common [ 19 ]. AI tools can be valuable considering the quick generation of quality test items and time management. However, as observed in the present study, the AI-generated test items nevertheless require scrutiny by faculty members for content validity. Moreover, it is important to train faculty in AI technology-assisted teaching and learning. The General Medical Council recommends taking every opportunity to raise the profile of teaching in medical schools [ 20 ]. Hence, both the academic faculty and the institution must consider investing resources in AI training to ensure appropriate use of the technology [ 21 ].

The AI outputs assessed in the present study had errors, particularly with A-type MCQs. One notable observation was that often the AI tools were unable to differentiate the differences between ACEIs and ARBs. AI platforms access several structured and unstructured data, in addition to images, audio, and videos. Hence, the AI platforms can commit errors due to extracting details from unauthenticated sources [ 22 ] created a framework identifying 28 factors for reconstructing the path of AI failures and for determining corrective actions. This is an area of interest for AI technical experts to explore. Also, this further iterates the need for human examination of test items before using them for assessment purposes.

There are concerns that AIs can memorize and provide answers from their training dataset, which they are not supposed to do [ 23 ]. Hence, the use of AIs-generated test items for summative examinations is debatable. It is essential to ensure and enhance the security features of AI tools to reduce or eliminate cross-contamination of test items. Researchers have emphasized that AI tools will only reach their potential if developers and users can access full-text non-PDF formats that help machines comprehend research papers and generate the output [ 24 ].

AI platforms may not always have access to all standard treatment guidelines. However, in the present study, it was observed that all three AI platforms generally provided appropriate test items regarding the choice of medications, aligning with recommendations from contemporary guidelines and standard textbooks in pharmacology and therapeutics. The prompts used in the study were specifically focused on the pre-clerkship phase of the undergraduate medical curriculum (and at the time of their graduation) and assessed fundamental core concepts, which were also reflected in the AI outputs. Additionally, the recommended first-line antihypertensive drug classes have been established for several decades, and information regarding their pharmacokinetics, ADRs, and indications is well-documented in the literature.

Different paradigms and learning theories have been proposed to support AI in education. These paradigms include AI- directed (learner as recipient), AI-supported (learner as collaborator), and AI-empowered (learner as leader) that are based on Behaviorism, Cognitive-Social constructivism, and Connectivism-Complex adaptive systems, respectively [ 25 ]. AI techniques have potential to stimulate and advance instructional and learning sciences. More recently a three- level model that synthesizes and unifies existing learning theories to model the roles of AIs in promoting learning process has been proposed [ 26 ]. The different components of our study rely upon these paradigms and learning theories as the theoretical underpinning.

Strengths and limitations

To the best of our knowledge, this is the first study evaluating the utility of AI platforms in generating test items related to a discipline in the undergraduate medical curriculum. We have evaluated the AI’s ability to generate outputs related to most types of assessment in the undergraduate medical curriculum. The key lessons learnt for improving the AI-generated test item quality from the present study are outlined in Table  7 . We used a structured framework for assessing the content validity of the test items. However, we have demonstrated using a single case study (hypertension) as a pilot experiment. We chose to evaluate anti-hypertensive drugs as it is a core learning objective and one of the most common disorders relevant to undergraduate medical curricula worldwide. It would be interesting to explore the output from AI platforms for other common (and uncommon/region-specific) disorders, non-/semi-core objectives, and disciplines other than Pharmacology and Therapeutics. An area of interest would be to look at the content validity of the test items generated for different curricula (such as problem-based, integrated, case-based, and competency-based) during different stages of the learning process. Also, we did not attempt to evaluate the generation of flowcharts, algorithms, or figures for generating test items. Another potential area for exploring the utility of AIs in medical education would be repeated procedural practices such as the administration of drugs through different routes by trainee residents [ 27 ]. Several AI tools have been identified for potential application in enhancing classroom instructions and assessment purposes pending validation in prospective studies [ 28 ]. Lastly, we did not administer the AI-generated test items to students and assessed their performance and so could not comment on the validity of test item discrimination and difficulty indices. Additionally, there is a need to confirm the generalizability of the findings to other complex areas in the same discipline as well as in other disciplines that pave way for future studies. The conceptual framework used in the present study for evaluating the AI-generated test items needs to be validated in a larger population. Future studies may also try to evaluate the variations in the AI outputs with repetition of the same queries.

Notwithstanding ongoing discussions and controversies, AI tools are potentially useful adjuncts to optimize instructional methods, test blueprinting, test item generation, and guidance for test standard-setting appropriate to learners’ stage in the medical program. However, experts need to critically review the content validity of AI-generated output. These challenges and caveats are to be addressed before the use of widespread use of AIs in medical education can be advocated.

Data availability

All the data included in this study are provided as Electronic Supplementary Materials.

Tolsgaard MG, Pusic MV, Sebok-Syer SS, Gin B, Svendsen MB, Syer MD, Brydges R, Cuddy MM, Boscardin CK. The fundamentals of Artificial Intelligence in medical education research: AMEE Guide 156. Med Teach. 2023;45(6):565–73.

Article   Google Scholar  

Sriwastwa A, Ravi P, Emmert A, Chokshi S, Kondor S, Dhal K, Patel P, Chepelev LL, Rybicki FJ, Gupta R. Generative AI for medical 3D printing: a comparison of ChatGPT outputs to reference standard education. 3D Print Med. 2023;9(1):21.

Azer SA, Guerrero APS. The challenges imposed by artificial intelligence: are we ready in medical education? BMC Med Educ. 2023;23(1):680.

Masters K. Ethical use of Artificial Intelligence in Health Professions Education: AMEE Guide 158. Med Teach. 2023;45(6):574–84.

Nagi F, Salih R, Alzubaidi M, Shah H, Alam T, Shah Z, Househ M. Applications of Artificial Intelligence (AI) in Medical Education: a scoping review. Stud Health Technol Inf. 2023;305:648–51.

Google Scholar  

Mehta N, Harish V, Bilimoria K, et al. Knowledge and attitudes on artificial intelligence in healthcare: a provincial survey study of medical students. MedEdPublish. 2021;10(1):75.

Mir MM, Mir GM, Raina NT, Mir SM, Mir SM, Miskeen E, Alharthi MH, Alamri MMS. Application of Artificial Intelligence in Medical Education: current scenario and future perspectives. J Adv Med Educ Prof. 2023;11(3):133–40.

Garg T. Artificial Intelligence in Medical Education. Am J Med. 2020;133(2):e68.

Matheny ME, Whicher D, Thadaney IS. Artificial intelligence in health care: a report from the National Academy of Medicine. JAMA. 2020;323(6):509–10.

Sage Poe. Available at: https://poe.com/Assistant (Accessed on. 3rd June 2023).

Claude-Instant: Available at: https://poe.com/Claude-instant (Accessed on 3rd. June 2023).

ChatGPT: Available at: https://poe.com/ChatGPT (Accessed on 3rd. June 2023).

James PA, Oparil S, Carter BL, Cushman WC, Dennison-Himmelfarb C, Handler J, Lackland DT, LeFevre ML, MacKenzie TD, Ogedegbe O, Smith SC Jr, Svetkey LP, Taler SJ, Townsend RR, Wright JT Jr, Narva AS, Ortiz E. 2014 evidence-based guideline for the management of high blood pressure in adults: report from the panel members appointed to the Eighth Joint National Committee (JNC 8). JAMA. 2014;311(5):507–20.

Eschenhagen T. Treatment of hypertension. In: Brunton LL, Knollmann BC, editors. Goodman & Gilman’s the pharmacological basis of therapeutics. 14th ed. New York: McGraw Hill; 2023.

Shabatura J. September. Using Bloom’s taxonomy to write effective learning outcomes. https://tips.uark.edu/using-blooms-taxonomy/ (Accessed on 19th 2023).

Trainor A, Richards JB. Training medical educators to teach: bridging the gap between perception and reality. Isr J Health Policy Res. 2021;10(1):75.

Boscardin C, Gin B, Golde PB, Hauer KE. ChatGPT and generative artificial intelligence for medical education: potential and opportunity. Acad Med. 2023. https://doi.org/10.1097/ACM.0000000000005439 . (Published ahead of print).

Duong MT, Rauschecker AM, Rudie JD, Chen PH, Cook TS, Bryan RN, Mohan S. Artificial intelligence for precision education in radiology. Br J Radiol. 2019;92(1103):20190389.

Karthikeyan S, O’Connor E, Hu W. Barriers and facilitators to writing quality items for medical school assessments - a scoping review. BMC Med Educ. 2019;19(1):123.

Developing teachers and trainers in undergraduate medical education. Advice supplementary to Tomorrow’s Doctors. (2009). https://www.gmc-uk.org/-/media/documents/Developing_teachers_and_trainers_in_undergraduate_medical_education___guidance_0815.pdf_56440721.pdf (Accessed on 19th September 2023).

Cooper A, Rodman A. AI and Medical Education - A 21st-Century Pandora’s Box. N Engl J Med. 2023;389(5):385–7.

Chanda SS, Banerjee DN. Omission and commission errors underlying AI failures. AI Soc. 2022;17:1–24.

Narayanan A, Kapoor S. ‘GPT-4 and Professional Benchmarks: The Wrong Answer to the Wrong Question’. Substack newsletter. AI Snake Oil (blog). https://aisnakeoil.substack.com/p/gpt-4-and-professional-benchmarks (Accessed on 19th September 2023).

Brainard J. November. As scientists face a flood of papers, AI developers aim to help. Science, 21 2023. doi.10.1126/science.adn0669.

Ouyang F, Jiao P. Artificial intelligence in education: the three paradigms. Computers Education: Artif Intell. 2021;2:100020.

Gibson D, Kovanovic V, Ifenthaler D, Dexter S, Feng S. Learning theories for artificial intelligence promoting learning processes. Br J Edu Technol. 2023;54(5):1125–46.

Guerrero DT, Asaad M, Rajesh A, Hassan A, Butler CE. Advancing Surgical Education: the Use of Artificial Intelligence in Surgical Training. Am Surg. 2023;89(1):49–54.

Lee S. AI tools for educators. EIT InnoEnergy Master School Teachers Conference. 2023. https://www.slideshare.net/ignatia/ai-toolkit-for-educators?from_action=save (Accessed on 24th September 2023).

Download references

Author information

Authors and affiliations.

Department of Pharmacology & Therapeutics, College of Medicine & Medical Sciences, Arabian Gulf University, Manama, Kingdom of Bahrain

Kannan Sridharan & Reginald P. Sequeira

You can also search for this author in PubMed   Google Scholar

Contributions

RPS– Conceived the idea; KS– Data collection and curation; RPS and KS– Data analysis; RPS and KS– wrote the first draft and were involved in all the revisions.

Corresponding author

Correspondence to Kannan Sridharan .

Ethics declarations

Ethics approval and consent to participate.

Not applicable as neither there was any interaction with humans, nor any personal data was collected in this research study.

Consent for publication

Not applicable.

Competing interests

The authors declare no competing interests.

Additional information

Publisher’s note.

Springer Nature remains neutral with regard to jurisdictional claims in published maps and institutional affiliations.

Electronic supplementary material

Below is the link to the electronic supplementary material.

Supplementary Material 1

Rights and permissions.

Open Access This article is licensed under a Creative Commons Attribution 4.0 International License, which permits use, sharing, adaptation, distribution and reproduction in any medium or format, as long as you give appropriate credit to the original author(s) and the source, provide a link to the Creative Commons licence, and indicate if changes were made. The images or other third party material in this article are included in the article’s Creative Commons licence, unless indicated otherwise in a credit line to the material. If material is not included in the article’s Creative Commons licence and your intended use is not permitted by statutory regulation or exceeds the permitted use, you will need to obtain permission directly from the copyright holder. To view a copy of this licence, visit http://creativecommons.org/licenses/by/4.0/ . The Creative Commons Public Domain Dedication waiver ( http://creativecommons.org/publicdomain/zero/1.0/ ) applies to the data made available in this article, unless otherwise stated in a credit line to the data.

Reprints and permissions

About this article

Cite this article.

Sridharan, K., Sequeira, R.P. Artificial intelligence and medical education: application in classroom instruction and student assessment using a pharmacology & therapeutics case study. BMC Med Educ 24 , 431 (2024). https://doi.org/10.1186/s12909-024-05365-7

Download citation

Received : 26 September 2023

Accepted : 28 March 2024

Published : 22 April 2024

DOI : https://doi.org/10.1186/s12909-024-05365-7

Share this article

Anyone you share the following link with will be able to read this content:

Sorry, a shareable link is not currently available for this article.

Provided by the Springer Nature SharedIt content-sharing initiative

  • Medical education
  • Pharmacology
  • Therapeutics

BMC Medical Education

ISSN: 1472-6920

medical case study questions and answers

IMAGES

  1. case studies questions & answers

    medical case study questions and answers

  2. 💐 Case study answers format. Sample Case Study Questions and Answers

    medical case study questions and answers

  3. Case Study with Answers

    medical case study questions and answers

  4. Case study 15.pdf

    medical case study questions and answers

  5. FREE 11+ Medical Case Study Templates in PDF

    medical case study questions and answers

  6. HIV Case Study DONE

    medical case study questions and answers

VIDEO

  1. Case 3

  2. मानसिक बीमारी कुछ ही दिनो में ठीक करे

  3. CBSE class 10 maths : Case study based questions with solution in Malayalam

  4. Case study 5 :- Loose Motion

  5. Case:8 Medical Case Study #casestudy#case#diagnosis#medicalconditions#clinicalcase#infant#disorders

  6. Family case study

COMMENTS

  1. USMLE Step 1: Clinical Case Questions

    Clincal Case Practice Question 1. A 24-year-old woman presents with a fever and myalgias. She experienced brief, self-limited diarrhea 24 hours after attending a barbecue two weeks earlier. She remained asymptomatic until the day prior to presentation when she developed a fever of 39.4 C (103 F), conjunctivitis, and severe muscle pain.

  2. Clinical cases Archives

    Hyperglycaemia case study with questions and answers - for doctors and medical students exams, finals and OSCEs.

  3. Clinical Cases • LITFL Medical Blog • Case Collection

    The LITFL Clinical Case Collection includes over 250 Q&A style clinical cases to assist ' Just-in-Time Learning ' and ' Life-Long Learning '. Cases are categorized by specialty and can be interrogated by keyword from the Clinical Case searchable database. Search by keywords; disease process; condition; eponym or clinical features….

  4. Educational Case: A 57-year-old man with chest pain

    This is an educational case report including multiple choice questions and their answers. For the best educational experience we recommend the interactive . ... Oxford Medical Case Reports, Volume 2016, Issue 4, April 2016, ... both, or neither. the ISIS-2 (second international study of infarct survival) collaborative group. BMJ. 1998; 316 (7141):

  5. Faculty Case Studies

    When students take the Test Bank questions, they will not get feedback on correct answers. Students and faculty should review test results and correct answers together. The case studies are contained in 4 categories: Family (13 case studies), Fundamentals and Mental Health (14 case studies) and Medical Surgical (20 case studies).

  6. Next Generation NCLEX Case Study Sample Questions

    NGN Case Study Sample Questions and Answers. First, let's take a look at our case study summary below: Case Study Summary: A 68-year-old male is admitted with shortness of breath. He reports difficulty breathing with activity, lying down, or while sleeping. He states that in order to "breathe easier," he has had to sleep in a recliner for ...

  7. 25: Case Study Answers

    Read chapter 25 of Pathophysiology of Disease: An Introduction to Clinical Medicine, 7e online now, exclusively on AccessMedicine. AccessMedicine is a subscription-based resource from McGraw Hill that features trusted medical content from the best minds in medicine.

  8. Dementia case study with questions and answers

    Dementia case study with questions and answers. Common dementia exam questions for medical finals, OSCEs and MRCP PACES. The case below illustrates the key features in the assessment of a patient with dementia or undiagnosed memory decline. It works through history, examination and investigations - click on the plus symbols to see the answers ...

  9. New Case Based Questions for Internal Medicine Board Review

    NEJM Knowledge+ Internal Medicine Board Review already contains more than 1600 case-based questions on the most relevant and important topics in medicine today. We're adding another 50 case-based questions now (and 50 more in December 2016) to further expand that knowledge base. Most of the 50 new questions we've added relate to the ...

  10. ABG Examples (ABG exam questions for medical students and PACES)

    ABG Examples (ABG exam questions for medical students OSCEs and MRCP PACES) Below are some brief clinical scenarios with ABG results. Try to interpret each ABG and formulate a differential diagnosis before looking at the answer. Question 1. You are called to see a 54 year old lady on the ward.

  11. Diabetes Case Studies : Real Problems, Practical Solutions

    Diabetes Case Studies: Real Problems, Practical Solutions. Edited by: Boris Draznin, MD, PhD, Cecilia C. Low Wang, MD, FACP ... Rosalind Franklin University of Medicine and Science/Chicago Medical School, North Chicago, IL. 2 Endocrinology Section, Department of Medicine, Captain James A. Lovell Health Care Center, North Chicago, IL. Search for ...

  12. Top 100 Ultrasound • LITFL • Ultrasound Library Clinical cases

    LITFL 100+ Ultrasound quiz. Clinical cases and self assessment problems from the Ultrasound library to enhance interpretation skills through Ultrasound problems. Preparation for examinations. Each case presents a clinical scenario; a series of questions; clinical images and finally some pearls to highlight the key learning points.

  13. OSCE Cases

    Each scenario allows you to work through history taking, investigations, diagnosis and management. You might also be interested in our bank of 1000+ OSCE Stations. A collection of interactive medical and surgical OSCE cases (clinical case scenarios) to put your history, examination, investigation, diagnostic and management skills to the test.

  14. Appendix C: Answers to Case Study Questions

    Read chapter Appendix C of Pharmacology in Rehabilitation, 5e online now, exclusively on F.A. Davis AT Collection. F.A. Davis AT Collection is a subscription-based resource from McGraw Hill that features trusted content from the best minds in PT.

  15. Answers to the Clinical Case Study Questions : Home Healthcare Now

    Answers to clinical case study questions: PDF Only. Answers to the Clinical Case Study Questions. Thobaben, Marshelle RN, MS, FNP; Anderson, Linda RN, MSN. Home Healthcare Nurse: The Journal for the Home Care and Hospice Professional: January 1993 - Volume 11 - Issue 1 - p 61. Buy

  16. Review Questions

    QUESTION ONE. Answer: A & C. Rationale. Visual changes, recurrent infections, and pruritis are all complications of Type 2 Diabetes Mellitus. Although polyuria, polyphagia, and polydipsia are known as the classic symptoms for Type 1 Diabetes Mellitus, they are also present in Type 2 Diabetes Mellitus. Nausea, hypotension and mental confusion ...

  17. Clinical Case Studies with Answers

    c) Endocrine System Case Studies with answers: Diabetes: Case Study 17 and Case Study - 18. Hypothyroidism : Case Study -19 and Case Study -20. Other Thyroid Disorders : Case Study -21 and Case Study -22. Oral Contraceptive Use: Case Study- 23 and Case Study- 24. Hormone Replacement Therapy: Case Study- 25 and Case Study- 26.

  18. Respiratory Assessment Case Study (Questions) Flashcards

    A) Ask the client to cough before repeating the assessment. B) Document the normal finding on the assessment record. C) Question the client about a recent history of rib fractures. D) Stop the assessment and measure the client's vital signs. B) Document the normal finding on the assessment record.

  19. Clinical Medicine Quiz

    Clinical Medicine Quiz. There are 2523 questions available in this quiz.

  20. Internal Medicine Cases & Quizzes

    Internal Medicine Cases & Quizzes. Hematologic Complications After Femoral Fracture Surgery During the course of her postoperative hospital stay, a gradual decline in her platelet count is noted ...

  21. APPENDIX C: Answers to Case Study Questions

    Read chapter APPENDIX C of Pharmacology in Rehabilitation online now, exclusively on F.A. Davis PT Collection. F.A. Davis PT Collection is a subscription-based resource from McGraw Hill that features trusted content from the best minds in PT.

  22. Case Study: 33-Year-Old Female Presents with Chronic SOB and Cough

    History of Present Illness: A 33-year-old white female presents after admission to the general medical/surgical hospital ward with a chief complaint of shortness of breath on exertion. She reports that she was seen for similar symptoms previously at her primary care physician's office six months ago. At that time, she was diagnosed with acute bronchitis and treated with bronchodilators ...

  23. CPT Coding: Case Studies

    Answer 3: CPT code 80053 (Comprehensive metabolic panel) should be assigned for the CMP. CPT code 85025 (Complete (CBC), automated (Hgb, Hct, RBC, WBC, and platelet count)) should be assigned for the CBC. Case Study 4: Radiology Coding. Scenario: A patient comes to the radiology department for an X-ray of the right ankle after twisting it ...

  24. Artificial intelligence and medical education: application in classroom

    Artificial intelligence (AI) tools are designed to create or generate content from their trained parameters using an online conversational interface. AI has opened new avenues in redefining the role boundaries of teachers and learners and has the potential to impact the teaching-learning process. In this descriptive proof-of- concept cross-sectional study we have explored the application of ...